Вы находитесь на странице: 1из 206

2015

IBPS Clerk & PO


Preliminary Exam
Quantitative Aptitude
EBook Questions With
Explained
Pages: 206 Price: 45/- Website: www.todaysprint.com

Jenisys Systems Pvt ltd


4/30/2015

Compound Interest
1. The height of a tree increase every year 1/8 times .If the present height of the tree is 64 cm, then
what will be its height after 2 years?
(a) 76 cm (b) 80 cm (c) 81 cm

(d) 84 cm

(e) None of these

Solution: (c) .
Initial height = 64
First year = 1/8(64) = 8
So 8+64 = 72cm
Second year = 1/8(72) = 9
So 9+72 = 81cm

2. The Simple Interest on a certain sum for 2 years is Rs.120 and the compound interest is Rs. 129. Find
the rate of interest
(a ) 14% (b) 15 %

(c ) 12 %

(d ) 12 % (e ) Cannot be determined

Solution ; (b).
Interest for first year in Simple Interest and Compound Interest are same
So we need to blind first year interest.
Simple interest for every year will be same
= Simple Interest for 2 years = 120
1 year = 120/2 = 60
So first year interest of Compound Interest = 60
The difference between simple interest and compound interest for 2nd year is a. So find a is what % of
60
= 9/60 X 100 = 15%

3. At what percentage annual compound interest rate, a certain sum amounts to its 27 times in 3 Years?
Jenisys Systems Pvt ltd www.todaysprint.com

| www.ibpsexamguru.in

( a) 100%

(b ) 150 %

(c ) 75%

(d ) 200% (e) 900%

Solution: (d).
Let the principal be P
N=3
Amount = 27P
= 27P = P ( 1+ R/100)3
While solving
(3)3 = ( 1+R/100)3
Therefore 3 = 1 + R/100
=R/100 = 3-1
So R = 200%

4. Find the least number of complete years in which a sum of money put out at 40% compound interest
will be more than double.
(a) 3yr (b) 4 yr (c) 5yr (d) 8 yr (e) None of these
Solution: (a).
Let the principal be P
Then P(1+40/100)n > 2P
While solving the above relation , we get
(140/100)n >2 => (7/5)n >2
= (1.4)n > 2
So n = 3
By trail sub the values for n
When n = 3
We get (1.4)3 is >2
Therefore number of years is 3
Jenisys Systems Pvt ltd www.todaysprint.com | www.ibpsexamguru.in

5. What amount will be received on a sum of Rs.15000 in 1 yrs at 10% per annum, if interest is
compounded half yearly?
(a) 4996.5 (b) 19996.5 (c) 23999.5 (d) 21609.25 (e) None of these
Solution: (5).
P = 15000
N = 1 years N = 3/2
R = 10%, p.a. (for year = 5%)
Since the interest is compounded
yearly the formula will be
Amount = P (1+ R/100)2N
= 15000 ( 1 + 5/100) 2 3/2
= 15000 ( 105/100)3
= 15000 x 21/20 x 21/20 x 21/20
= 138915/8
= 17364.375

PARTNERSHIP
6. A and B invest in the ratio of 3:5 respectively. After 6 months C enters the business with the
investment of the capital equal to that of B. What will be the ratio of the profits of A, B and C at the end
of the year?
a) 6 : 10 : 5

b) 3 : 5 : 5

c) 3 : 5 : 2

d) 6 : 2 : 3

e) None of these

Solution: (a).
Let investment of A = 3x
Therefore investment of B = 5x
Investment of C = 5x
As we know,
Jenisys Systems Pvt ltd www.todaysprint.com | www.ibpsexamguru.in

Ratio of profits = Ratio of investments


Therefore As share : Bs share : Cs share
= 3x * 12 : 5x *12 :5x * (12- 6)
= 3x * 12 :5x * 12 : 5x * 6 = 6:10:5

7. Ramesh and Priya started a business initially with Rs. 5100 and Rs.6600, respectively. Investments
done by both the persons are for different time periods .If the total profits is Rs. 5460, what is the profit
of Ramesh?
a) Rs.1530 b) Rs.1600 c) 1400

d) data inadequate

e) None of the above

Solution: (d).
Time is not given in the question. so, we cannot find the profit of Ramesh. Hence, the data is
inadequate.

8. A, B and C together start a business. B invests ______of the total capital while investment s of A and C
are equal. If the annual profit on this investment is Rs. 33600. Find the difference between the profits of
B and C.
a) Rs.8400

b) Rs.7200

c) 6000

d) Rs.9600

e) None of these

Solution: (a).
Given,
Investment of B = 1/6 of total capital
Therefore investments of A and C each
= ( 1- 1/6) of total capital
= x 5/6 of total capital
= 5/12 of total capital
Now, As share : Bshare: Cs share
= 5/12 : 1/6 : 5/12
= 5 : 2 :5

Jenisys Systems Pvt ltd www.todaysprint.com | www.ibpsexamguru.in

Let As share = 5x
Bs share = 2x
Cs share = 5x
According to the question,
5x + 2x + 5x = 33600
Therefore 12x = 33600
Therefore x = 33600/12 = 2800
Therefore difference in the profits of B and C
= 5x - 2x = 3x = 3 x 2800 = Rs.8400

9. A and B together start a business by investing in the ratio of 4:3. If 9% of the total profit goes to
charity and As share is Rs. 1196, find the total profit.
a) Rs.2300

b) 4435

c) 2093

d) RS.2700

e) None of the above

Solution: (a).
Let total profit = x
Paid to charity = 9% of x = 9x/100
Therefore balance profit = x 9x/100 = 91x /100
Therefore As share = 4/(4+3) x 91x /100 = 4/7 x 91x /100
According to the question,
4/7 * 91x /100 = 1196
Therefore x = (1196 x 7x100) / 4 * 91 = Rs.2300
Hence, total profit = Rs.2300

10. A,B and C invested capitals in the ratio of 4 : 6 : 9 . At the end of the business term, they received the
profit in the ratio of 2 : 3 : 5. Find the ratio of the time for which they contributed their capitals.
a) 1:1 : 9

b) 2: 2 : 9

c) 10 : 10 : 9

d) 9 : 9 :10

Jenisys Systems Pvt ltd

www.todaysprint.com | www.ibpsexamguru.in

Solution: (d).
Here, P1 :P2 : P3 = 2 : 3 : 5 (Profit ratio)
And x1 : x2 :x3 = 4 : 6 : 9 ( investments ratio)
According to the rule,
Required ratio
= P1/x1 : P2/x2 : P3/x3
= 2/4 : 3/6 : 5/9
= : : 5/9 = 9 : 9 : 10

Most important 5 Ratio questions


11. 4 years ago Rahuls age was 3/4 times that of Ravi .Four years hence Rahuls age will be 5/6 times
that of Ravi. What is the present age of Rahul?
1) 20 years

2) 15 years

3) 24 years

4) 16 years

5) None of these

Solution: (4). Let us consider present year as 2014 so 4 years ago is 2010 and 4 years hence from 2014
is 2018. So the gap between 2010 and 2018 is 8 years. So the equation becomes
(3x+8) / (4x+8) = 5/6 then x= 4
Present age of Rahul is 3x+4 = 3(4) + 4 = 16 years
12. Mr.Sundaram owned 950 gold coins all of which he distributed amongst his three sons Lakshman,
Arun and Nagaraj . Lakshman gave 25 gold coins to his wife, Arun donated 15 gold coins and Nagaraj
made jewellery out of 30 gold coins. The new respective ratio of the coins left with them was 20:73:83.
How many gold coins did Arun receive from Mr.Sundaram?
1) 350

2) 400

3) 380

4) 415

5)None of these

Solution: (3). Total coins spent by the sons is 70 (25+15+30).


So remaining coins is 880 (950 - 70) .
Given ratio is corresponding to remaining coins.
So 20x + 73x + 83x = 880. Then x = 5.
So Arun received 73x+15 = 73(5) + 15 = 380. So he received 380 gold coins.
Jenisys Systems Pvt ltd www.todaysprint.com | www.ibpsexamguru.in

13. Abi invested in three schemes A,B and C . The amounts in the Ratio of 2:3:4 respectively. If the
schemes offered interest at 20 p.c.p.a. , 16 p.c.p.a and 15 p.c.p.a. respectively .What will be respective
ratio of the amounts after 1 year?
1) Cannot be determined

2) 10:8:5

3) 8:10:5

4) 15:14:12

5) None of these

Solution: (5). Let us consider the amount invested in three schemes A,B and C is 2x, 3x, 4x and assume
x = 100. Then they have 200,300,400.
As per interest rates Scheme A gets 20% which is 40 ( 200 x 20/100)
Similarly B gets 16% which is 48 ( 300 x 16/100)
Similarly C gets 15% which is 60 ( 400 x 15/100
So amount after 1 year is 240,348,460
then the ratio becomes 240:348:460 which is 60:87:115.
So answer is none of these

14. A sum of money divided among A, B, C and D in the ratio of 4:5:7:11 respectively .If the share of C is
Rs.1351 then what is the total amount of money A and D together?
1) Rs. 2895

2) Rs.2316

3) Rs.2565

4) Rs.2123

5) None of these

Solution: (1). The ratio of A,B,C,D is 4x, 5x, 7x, 11x


It is given that share of C is 1351 then C = 7x = 1351, so solving x = 193.
Now question is A+D
Then A+D = 4x+11x = 15x = 15 x 193 = 2895. This is the share of A+D.

15. 38% of first number is 52% of second number .What is respective ratio of the first number to the
second number.
1) Cannot be determined

2) 16:9

3) 5:4

4) 26:19

5) None of these

Solution: (4) .let us consider first number as N1 and second number as N2


It is given 38% of N1 = 52% of N2
Jenisys Systems Pvt ltd www.todaysprint.com | www.ibpsexamguru.in

38% x N1 = 52% x N2
N1 / N2 = 52% / 38%
N1 / N2 = 52/38 further solving 26/19.
So N1:N2 = 26:19

(Discount)
16. A shopkeeper allows a discount of 10% to his customers and still gains 20%, the marked price of the
article which costs Rs.450, is
1) Rs.600

2) Rs.540

3) Rs.660

4) Rs.580

5) None of these

17. The marked price of an article is Rs.500. It is sold on two successive discounts of 20% and 10%. The
selling price of that article is
1) Rs.350

2) Rs.375

3) Rs.360

4) Rs.400

5) None of these

18. If a shopkeeper sold a book with 20% profit after giving a discount of 10% on marked price. The ratio
of cost price and marked price of the book is
1) 6:5

2) 5:6

3) 3:4

4) 2:3

5) None of these

19. What is the maximum percentage discount (approximately) that a merchant can offer on his marked
price, so that he ends up selling at no profit or loss, if he initially marked his goods up by 40%?
1) 60%

2) 28.5%

3) 33.5%

4) No discount

5) None of these

20. In order that there may be a profit of 20% after allowing a discount of 10% on the marked price, the
cost price of an article has to be increased by
1) 30%

2) 33%

3) 33 1/3%

4) 33 2/3%

5) None of these

(Time & Work)

Jenisys Systems Pvt ltd www.todaysprint.com | www.ibpsexamguru.in

21. A and B together can do a piece of work in 12 days, while B alone can finish it in 30 days. A alone can
finish the work in
1) 15days

2) 18 days

3) 20 days

4) 25 days

5) None of these

Solution: (3).
(A+B) s 1 days work = 1/12
Bs 1 days work = 1/30
Therefore As 1 dayss work = 1/12 1/30
= 5-2 / 60 = 1/20

22. A and B can do a piece of work in 72 days. B and C can do it in 120 days. A and C can do it in 90 days.
In what time can A alone do it?
1) 80 days

2) 100 days

3) 120 days

4) 150 days

5) None of these

Solution: (3).
( A+B)s 1 dayss work = 1/72
( B+C)s 1 days work = 1/120
(A+C)s 1 days work = 1/90
2((A+B+C)s 1 days work
= 1/72 + 1/120 + 1/90
Therefore ( A+B+C)s 1 days work
= (5+3+4) / (360 x 2) = (12/360 x 2 ) = 1/60
Therefore As 1 dayss work = (A+B+C)s 1 days work ( B+C)s 1 days work
= 1/60 1/120 = 2 1 /120 = 1/120
Therefore A alone can finish the work in 120 days.

23. Acan do a piece of work in x days and B can do the same work 3x days. To finish the work together
they take 12 days. What is the value of x?
1) 8

2) 10

3) 12

4) 16

5) None of these

Jenisys Systems Pvt ltd www.todaysprint.com | www.ibpsexamguru.in

10

Solution: (4).
As 1 days work A = 1/x
Bs 1 days work B = 1/3x
( A+B)s 1 days work
= 1/x + 1/3x = 4/3x
And given one day work of both A and B = 1/12
= 4/3x = 1/12 = 3x = 48 = x = 16

24. A can do a piece of work in 10 days and B in 20 days . They begin together but A leaves 2 days before
the completion of the work. The whole work will be done in
1) 8 days

2) 7 2/3

3) 7 days

4) 6 days

5) None of these

Solution: (1).
Here, a = 10 , b = 20 and x = 2
Therefore required time = ( a + x) *b / (a + b)
= (10 + 2) *20 / 30 = 8 days

25. A and B can complete a job in 24 days working together. A alone can complete it in 32 days. Both of
them worked together for 8 days and then A left. The number of days B will take to complete the
remaining job is
1)16 2)32 3)64 4)128 5)None of these
Solution: (3).
Let B will take x days to complete the remaining job .
According to the question,
1/A + 1/B = 1/24 and 1/A = 1/32
Therefore 1/B = 1/24 1/32 = 1/96 = B = 96 days
According to the question,
8 ( 1/A + 1/B ) + x * 1/B = 1
Jenisys Systems Pvt ltd www.todaysprint.com | www.ibpsexamguru.in

11

= 8 x 1/24 + x/96 = 1
= 1/3 + x/96 = 1
x/96 = 1 1/3
therefore x = 2*96 /3 = 64
Hence, B complete the remaining job in 64 days.

26. 0.1 x 0.1 + 0.01 x 0.01 / 0.0101


1) 0.0101

2) 0.01

3) 0.1

4) 1

5) None of these

Solution: (4)
0.1 x 0.1 + 0.01 x 0.01 / 0.0101
= 0.01 + 0.0001 / 0.0101
= 0.0101 / 0.0101
=1

27. if 4x+1 - 4 x = 24 , the value of (2x)x is


1) 3

2) 3

3) 33

4) 9

5) None of these

Solution: (3). 4x+1 4x = 24


4x ( 4 -1 ) = 24
4x x 3 = 24
4x = 8
(22)x = 23
Therefore 2x = 3
X = 3/2
(2x)x = ( 2 x 3/2)3/2
= 3 3/2 = 31 x 3 = 33

Jenisys Systems Pvt ltd www.todaysprint.com | www.ibpsexamguru.in

12

=3 3

28. ( 0.6 x 0.6 + 0.6) 6 = ?


1) 0.16

2) 0.46

3) 0.37

4) 0.42

5) None of these

Solution: (1). (0.6x0.6+0.6) 6


= (0.36 + 0.6 ) 6
= 0.96 6
= 96 600
= 0.16

29. ( (81)0.81 x 9 ) / ( (81)0.99 x 90.64 ) = ?


1) 9

2) 3

3) 0

4) 1

5) None of these

Solution: (4). [ ( (81)0.81 x 9 ) / ( (81)0.99 x 9 0.64 ) ]


= [ ((92)0.81 x 91 ) ] / [ (92)0.99 x 9 0.64 )]
= (9 1.62 x 91) / (91.98 x 90.64) = 92.62 / 92.62
=1

30. P varies inversely as q+3, if p = 1, then Q = 3, when Q = 1, then p = ?


1) 1/3

2) 2/3

3) 3/2

4) 1/6

5) None of these

Solution:(3). P 1/Q+3
Therefore p = k x ( 1/Q+3 )
When p = 1, Q = 3
1 = k x 1/3+3
1= k x 1/6
6=k

Jenisys Systems Pvt ltd www.todaysprint.com | www.ibpsexamguru.in

13

Therefore when Q = 1
P = 6 x 1/ 1+3 = 6x = 6/4 = 3/2

31. To Maintain a speed of 264 km/hr with a wheel of diameter 28m. How many revolutions per minute
should the wheel rotate?
1) 500

2) 50

3) 3000

4) 40

5) None of these

Solution: (2). Given Diameter of wheel = 28m


Hence radius =14m
So circumference of wheel = 2 r
= 2 x 22/7 x 14
= 88m.
Therefore Distance covered in 1 revolution is 88m
Required number of revolutions / per minute (rpm) = 264 km / 60 x 88m
= 264 x 1000 / 60 x 88
= 50 rpm

32. If the breadth of rectangle is increased by 25% then by what % length be decreased to maintain
same area?
1) 20%

2) 25%

3) 30%

4) 50%

5) None of these

Solution:( 1).Tricky Approach


Let us consider length and breath as 10 cm each ,
So initial Area = l x b = 10 x 10 = 100 cm
If breath increased by 25% then b = 10 + 2.5 = 12.5
To maintain same area l x b = 100
l x 12.5 = 100
l = 100/12.5
Jenisys Systems Pvt ltd www.todaysprint.com | www.ibpsexamguru.in

14

=8
So 2 cm reduced in length to maintain same area
Then
(Reduced / initial) x 100 = 2/10 x 100 = 20 % .

33. A wire is bent to form a square of perimeter 110cm. It is opened and bent to form a circle, the
diameter of circle will be ________
1) 70

2) 35

3) 1225

4) 35/2

5) None of these

Solution:( 2).Perimeter of square = circumference of circle


110 = 2 r
110 = 2 x 22/7 x r
(110 x 7 ) / (2 x 22) = r
35 / 2 = radius
Then Diameter = radius x 2
= 35/2 x 2 = 35cm

34. The length and breadth of rectangular garden are 12 and 3 respectively . Find the length of the
diagonal of a square garden having the same area as that of the rectangular garden.
1) 62

2) 6

3) 12

4) 122

5) None of these

Solution: (1).Area of rectangle garden = area of square garden


= 12 x 3
= 36
Therefore side of a square = square root of 36 = 6
Then diagonal of the square = side x square root of 2
= 6 square root of 2.

Jenisys Systems Pvt ltd www.todaysprint.com | www.ibpsexamguru.in

15

35. The area of the ring between two concentric circles, whose circumferences are 44cm and 88 cm is
___
1) 462

2) 472

3) 482

4) 492

5) None of these

Solution: (1).Circle 1 : 2r = 44cm


2 x 22/7 x r = 44
r= 7cm
area = r2 = 22/7 x 7 x 7 = 154 cm2
Circle 2;
2r = 88
2 x 22/7 x r = 88
r = 14
Area = r2 = 22/7 x 14 x 14
= 616 cm2
Area Difference circle 2 & Circle 1
= 616 154
= 462 cm2

LCM & HCF


36. A fruit vendor has to arrange some number of fruits in rows. When he put five fruits in each row he
was left with one fruit . Then he tried six fruits in a row, then 8 , then 9, then 12. But always he was left
with one fruit . so he thought for a while and put 13 fruits in a row and then he was left with none . what
is the smallest number of fruits ?
1)3601 2)6301 3)2601 4)4601 5)None of these
Solution: (1).when 5 or 6 or 8 or 9 or 12 fruits are arranged in rows, one fruit is left.
LCM ( 5,6,8,9,12 ) = 360
Then the number of fruits is 360x + 1 .
It is given , number of fruits perfectly divisible by 13
Jenisys Systems Pvt ltd www.todaysprint.com | www.ibpsexamguru.in

16

Then (360x + 1) / 13
Substitute x = 1
= (360 x 1 +1 ) / 13
X = 2,3,4 , ..
X = 10
(360 x 10 + 1) / 13
is perfectly divisible
= 3601 / 13 = perfectly divisible
Alternately :
Among the options only first option is perfectly divisible by 13

37. What is the LCM of (m+1) (m+5) (m-3)2 and (m-3) (m+1)3 (m+5)2
1).(m+5) (m+1)(m-3)

2).(m+5)2 (m+1)3 (m-3)2

3).(m+5)(m+1)

4).(m+5)2 (m+1) (m-3)

5)None of these
Solution: (2). LCM [ (m+1) (m+5) (m-3)2, (m-3)(m+1)3(m+5)2 ]
Consider all greater powers
Ans = (m+5)2 (m+1)3 (m-3)2

38. If the LCM of A & B is C . Find their HCF.


1) (ABC)2

2) ABC / AB

3) AB / C

4)Cannot be determined

5)none of these

Solution: (3). First Number = A


Second Number = B
Given LCM (A,B) = C
LCM x HCF = A x B

Jenisys Systems Pvt ltd www.todaysprint.com | www.ibpsexamguru.in

17

C x HCF = A x B
HCF = AxB / C hence answer is AB / C

39. Find the least number of tiles required to lay the floor of length 150 cm and width 125 cm .
1) 40

2) 20

3) 30

4) 25

5) None of these

Solution: (4).This sum is based on equal distribution then HCF (150,125) = 30


Hence Answer is 25.

40. Find the greatest number of 5 digits which when divided by 3,5,8,12 have 2 as remainder .
1) 99999

2) 99958

3) 99960

4) 99962

5) None of these

Solution ; (5). LCM (3,,5,8,12) = 120


Then 120x+2 = max.5 digits
When x = 833
Then 120 x 833 +2
99960 + 2
Answer : 99962
(Basics of Numbers)

41. What is the place value of 6 in 63214178 ?


1) 6

2) 6 x 103

3) 6 x 107

4) 6 x 108

5) None of these

Solution: (3).6 is at the place value of crore


Therefore required place value = 6 x 10000000
= 6 x 107

42. Find the face value of 8 in 7382146.

Jenisys Systems Pvt ltd www.todaysprint.com | www.ibpsexamguru.in

18

1) 8

2) 8 x 102

3) 8 x 104

4) 83

5)None of these

Solution: (1). Face value is value of the digit itself


Therefore required face value of 8 is 8

43. Find the sum of face and place value of 9 in 738924.


1) 8991

2) 9009

3) 909

4) 891

5) None of these

Solution: (3). Place value of 9 = 900


Face value of 9 = 9
Then sum of place & face value = 900 + 9
= 909

44. Product of 1st Natural number and 1st Prime number is _______
1) 1

2) 2

3) 3

4) 5

5)None of these

Solution: (2). 1st Natural number = 1


1st Prime number = 2 (1 is not prime no )
Product of 1st natural & 1st prime number = 1 x 2 = 2.

45. 3/5 is rational whereas (square root of 3) / (square root of 5 ) is :


1) Rational numbers 2) Irrational number 3) Prime number 4) Natural number 5) None of these
Solution: (2). Square root of 3 and square root of 5 are irrational .
(Square root of 3) / (Square root of 5) is irrational .

Time & Distance


46. A train running at 30 km/hr takes 24 seconds to cross a platform. It takes 8 seconds to pass a man
walking towards it at 6 km/hr. Find the length of the train and of the platform.
1) 200 metres

2) 120 metres

3) 220 metres

4) 320 metres

5) None of these

Jenisys Systems Pvt ltd www.todaysprint.com | www.ibpsexamguru.in

19

Solution: (5). Distance = length of Train + Length of Platform


Convert all units to metres & Seconds
Platform crossed in 24 seconds
Then Distance = length of train + length of platform
= speed x time
= 30 km X 24 sec
= 30 X 5/18 x 24
Length of Train & Platform = 200
Man crossed in 8 seconds
Distance = length of train
= speed x time (speed = train +man)
= 36 x 5/18 x 8
Length of train = 80 m
Then platform = 200 80 = 120 m

47. Two trains are moving in the same direction at 75 km/hr and 48 km/hr. The faster train crosses a
man sitting in the slower in 14 seconds. Find the length of the faster train .
1) 105 metres 2) 115 metres 3) 125 metres 4) 120 metres 5) None of these
Solution: (1).Same direction = relative speed
= faster slower
= 75 48
Relative speed
Time to cross
Length of faster train
= 27 k/hr
= 14 seconds
Jenisys Systems Pvt ltd www.todaysprint.com | www.ibpsexamguru.in

20

= speed x time
= 27 km x 14 sec
= 27 x 5/18 x 14
= 105 metre

48. A 175 metre long train crosses a 35 metre platform in 12 seconds. What is the speed of the train in
Km/hr?
1)42 2)64 3)63 4)59 5)None of these
Solution (3). Speed = Distance / Time
[ Distance = Train +platform = 175 + 35 = 210 m ]
= 210 m / 12 sec
= 210/12
x 18/5 km/hr
= 63 km/hr

49. A train covers a distance of 180 km in 4 hours. Another train covers the same distance in one hour
less. What is the difference in the distance covered by these two trains in one hour?
1) 45 km

2) 40 km

3) 15 km

4) 9 km

5) None of these

Solution: (3).Distance covered by the first train in 1 hour


= 180 / 4 = 45 km
Distance covered by the second train in 1 hour = 180/3 = 60km
Therefore required difference = ( 60 - 45)km = 15km

50. A train takes 3 hours to run from one station to another . If it reduces its speed by 12 km/hr, it takes
45 minutes more for the journey . The distance between the station is
1) 220 km

2) 210 km

3) 180 km

4) 160 km

5) None of these

Solution: (3).let the distance between the stations be x km


Jenisys Systems Pvt ltd www.todaysprint.com | www.ibpsexamguru.in

21

Original speed = x/3. km/hr


New speed = [ (x/3 ) 12) km/hr
New time = 3 hrs + 45 minutes = 3 hrs = 15/4 hrs
Therefore [ (x/3) 12 ] * 15/4 = x
Or, (5/4) x - 45 =x
Or ,x/4 = 45
Or, x = 180 km

Simplification in Fractions
51. Convert 0.72 into a vulgar fraction
1) 16/25

2) 17/25

3) 18/25

4) 19/25

5) None of these

Solution: (3). 0.72 = 72/100 = 18/25

52. 4/10 + 9/100 + 8/1000 = ?


1) 498 2) 4.98 3)49.8 4) 0.498 5)None of these
Solution:(4). 4/10 + 9/100 + 8/1000 = 0.4 + 0.09 + 0.008 = 0.498

53. Express 0.555 . as a fraction


1) 5/6 2) 5/9 3) 5/7 4) 5/12 5)None of these
Solution: (2). x = 0.555.
10 * x = 10x = 5.555 ..
10x - x = 5.555 .. - 0.555 ..
9x = 5
x = 5/9

Jenisys Systems Pvt ltd www.todaysprint.com | www.ibpsexamguru.in

22

54. 7/8 , 8/9 , 9/10, 10/11, 11/12


which is the largest ?
1) 11/12

2) 10/11

3) 9/10

4) 7/8

5)None of these

Solution: (1).Difference in numerator and denomination are same in all fractions .


Then increasing order is 7/8, 8/9, 9/10, 10/11, 11/12
Therefore largest is 11/12

55. 0.33 .+ 0.44. = ?


1) 3/9

2) 4/9

3) 6/11

4) 2/9

5) None of these

Solution: (5). x = 0.33..+ 0.44


Therefore x = 0.77 ..
10x = 7.77
10x - x = 7
9x = 7
x = 7/9

Simplification in Powers
56. 289 = 17x/5 , then x = ?
1) 16

2) 8

3) 10

4) 2/5

5) None of these

Solution: (3). 289 = 17 x/5


172 = 17 x/5
2 = x/5
10 = x

57. 4x - 4x-1 = 48 , then xx = ?

Jenisys Systems Pvt ltd www.todaysprint.com | www.ibpsexamguru.in

23

1) 9

2) 27

3) 64

4) 16

5) None of these

Solution (2).
4x 4x-1 = 48
4 x-1 ( 4 - 1) = 48
4 x-1 * 3 = 48
4x-1 = 16
4 x-1 = 42
x-1 = 2
x=3
then xx = 33 = 27

58. Evaluate 5 x (0. 00032)2/5


1) 1/5

2) 1/25

3) 1/125

4) 1/625

5) None of these

Solution: (1).
5 x ( 0.00032) 2/5 = 5 (32 / 100000)2/5 = 5 ( 25 / 105 ) 2/5
= 5 [ ( 2/10)5 ] 2/5
= 5 x (2/10)5 x 2/5
= 5 x (1/5)2 = 5 x (1/25) = 1/5

59. Simplify (1024) 3/5


1) 128

2) 64

3) 32

4) 512

5) None of these

Solution (2).
(1024)3/5 = (210)3/5 = 210 x 3/5 = 26 = 64

60. Simplify (256) -3/4

Jenisys Systems Pvt ltd www.todaysprint.com | www.ibpsexamguru.in

24

1) 1/128

2) 1/64

3) 1/32

4) 1/96

5) None of these

Solution: (2).
1/ (256)3/4
= 1/ (44)3/4 = 1/43 = 1/64

Word problems on Numbers


61. In a park, there are some cows and some ducks. If total number of heads in the park are 68 and
number of their legs together is 198, then find the number of ducks in the park.
1) 37

2) 47

3) 57

4) 27

5) None of these

Solution: (1).Here, L =198 and H = 68


So, number of cows = (L 2H) / 2 = (198-2 x 68) / 2
= (198 136)/2 = 62/2 = 31
Therefore number of ducks = Number of heads number of cows
= 68 31 = 37

62. Five times of a positive integer is equal to 3 less than twice the square of that number. Find the
number.
1) 3

2) 13

3) 23

4) 33

5) None of these

Solutions : (1). Let the number be x


According to the question ,
5x=2x2 3
= 2x2 5x 3 = 0
= (x-3) (2x+1) = 0
X=3, -
Thus, the required number is 3.

Jenisys Systems Pvt ltd www.todaysprint.com | www.ibpsexamguru.in

25

63. Find the maximum number of trees which can be planted 20 m apart on the two sides of a straight
road 1760 m long.
1) 174

2) 176

3) 180

4) 178

5) None of these

Solution: (4).Number of trees that can be planted on one side of road = 1760/20 + 1 = 88 + 1 = 89
Therefore trees on the both sides = 2 x 89 = 178

64. There are two examination halls P and Q. If 10 students shifted P to Q , then the number of students
will be equal in both the examination halls. If 20 students shifted from Q to P, then the students of P
would be doubled to the students of Q. The number of students would be in P and Q, respective are
1) 60, 40

2) 70,50

3) 80,60

4) 100,80

5) None of these

Solution:( 4). Use the option to solve the sum.

65. In a three-digit number, the digit in the units place is four times the digit in the hundreds place. If
the digit in the units place and the tens place are interchanged, the new number so formed is 18 more
than the original number . If the digit in the hundreds place is one-third of the digit in the tens place,
then what is 25% of the original number?
1) 67

2) 84

3) 137

4) Couldnt be determined

5) None of these

Solution:(1).Let hundreds digit = x


Then units digit = 4x and tens digit = 3x
Number = 100x + 30x + 4x = 134x
Again, hundreds digit = x
Tens digit = 4x and units digit = 3x
Number = 100x + 40x + 3x = 143x
According to the question,
143x 134x = 18 = 9x = 18
X=2
Therefore original number = 134x = 134 x 2 = 268
25% of original number = 268 * 25/100 = 67
Jenisys Systems Pvt ltd www.todaysprint.com | www.ibpsexamguru.in

26

Average

66. The mean temperature of Monday to Wednesday was 37C and of Tuesday to Thursday was 34C. If
the temperature on Thursday was 4/5 that of Monday , the temperature of Thursday was
1) 35.5C

2) 34C

3) 36.5C

4) 36C

5) None of these

Solution:(4).Temperature of (Mon + Tue +Wed)


= 37 x 3 = 111 degree Celsius
Temperature of ( Tue+Wed+Thu)
=34 x 3 = 102 degree Celsius
Temperature of (Mon Thu)
= 111 degree celsius - 102 degree Celsius = 9 degree Celsius
Temperature of [ Mon 4/5 (Mon)] = 9
Temperature of Monday = 9 x 5 = 45 degree Celsius
Therefore Temperature of Thursday
= 45 X 4/5 = 36 degree Celsius

67. The average of the test scores of a class of m students is 70 and that of n students is 91. When the
scores of both the classes are combined, the average is 80. What is n/m?
1) 11/10

2) 13/10

3) 10/13

4) 10/11

5) None of these

Solution: (4).According to the question,


70m + 91n = 80 (m+n)
= 70m + 91n = 80m + 80n
= 10m = 11n
n/m = 10/11

Jenisys Systems Pvt ltd www.todaysprint.com | www.ibpsexamguru.in

27

68. If 47a + 47b = 5452, what is the average of a and b?


1) 116

2) 23.5

3) 96

4) 58

5)None of these

Solution: (4).
47 (a+b) = 5452
a+b = 5452 / 47 = 116
Therefore Average value
= (a+b) / 2 = 116/2
= 58

69. The average age of 5 sisters is 20 yr. If the age of the youngest sister be 4 yr, what was the average
age of the group of sisters at the birth of the youngest sister?
1) 25 year

2) 15 year

3) 18 year

4) 20 year

5) None of these

Solution: (4)
Total age of 5 sisters = 20 x 5 = 100 year
4 year ago, total sum of ages
= 100 (5 x 4) = 100 20 = 80 yr
But at that time ( 4 year ago), there were 4 sisters in the group
Therefore average age at that time (4 yr ago)
= 80/4 = 20 year

70. A cricketer played 80 innings and scored an average of 99 runs. His score in the last inning is zero
run. To have an average of 100 at the end, his score in the last innings should have been
1) 10 runs

2) 1 run

3) 60 runs

4) 80 runs

5) None of these

Solution: (4)
Let the required runs be x
According to the question
Jenisys Systems Pvt ltd www.todaysprint.com | www.ibpsexamguru.in

28

( 80 x 99 + x) / 80 = 100
= 7920 + x = 8000
X=80

Percentage
71. The population of a city is 250000. It is increasing at the rate of 2% every year. The growth in the
population after 2 yr is ?
1) 2500

2) 10000

3) 252000

4) 10100

5) None of these

Solution: (4).
Population after 2 year
= P ( 1 + R/100 )2 = 250000 ( 1 + 2/100 )2
= 250000 x 51/50 x 51/50 = 260100
Therefore Growth = 260100 250000 = 10100

72. A jogger desires to run a certain course in less time than he usually takes. By what per cent must
be increase his average running speed to accomplish the goal?
1) 50%

2) 20%

3) 25%

4) 33 1/3 %

5) None of these

Solution: (4).
xt = x 3/4t = x = 3/4x = x = 4/3 x
Thus, he has to increase his speed by
4/3 x- x / x
* 100% i.e. 33 1/3 %

73. The prices of two articles are as 3:4 . If the price of the first article is increased by 10% and that of
the second by Rs.4, one original ratio remains the same. The original price of the second article is
1) Rs.40

2) Rs.10

3) Rs.30

4) Rs.35

5) None of these

Solution: (1).
Jenisys Systems Pvt ltd ww.todaysprint.com | www.ibpsexamguru.in

29

Let cost prices of two articles be 3x and 4x , respectively . then,


(110% of 3 x) / (4x +4)
1.1x = x+1
0.1x = 1
x= 10
Thus, cost price of the second article is 4 x 10 = Rs. 40

74. An alloy of gold and silver weights 50g. It contains 80% gold. How much gold should be added to the
alloy, so that percentage of gold is increased to 90?
1) 50g

2) 60g

3) 30g

4) 40g

5) None of these

Solution: (1).
Gold in 50g of alloy
= 80 x 50/100 = 40g
Let x g gold must be added
Now, according to the question.
(40 +x) / (50 + x ) = 90/100
= 100 ( 40+x) = 90 (50+x)
= 10 ( 40 + x) = 9 ( 50+x )
= 400 + 10x = 450 + 9x
x = 450-400 = x = 50g
Thus, 50 g of gold must be added to make it 90%

75. 1 L of water is added to 5 L of alcohol and water solution containing 40% alcohol strength. The
strength of alcohol in the new solution will be
1) 30%

2) 33 1/3 %

3) 33 2/3%

4) 33%

5) None of these

Solution: (2).Quantity of alcohol in 5 L of solution


Jenisys Systems Pvt ltd ww.todaysprint.com | www.ibpsexamguru.in

30

= 40/100 x 5 = 2L
Quantity of alcohol in 6 L of solution = 2L
Therefore Strength of alcohol in new solution
= ( 2/6 x 100) % = 33 1/3%

Profit and Loss


76. By selling 32 oranges for Rs.30 a man loses 25% . How many oranges should be sold for Rs.24 so as
to gain 20% in the transaction?
1) 16

2) 24

3) 32

4) 28

5) None of the above

Solution (1).
Let the cost price be Rs. x
SP of 1 orange = Rs. 30/32 = Rs. 15/16
According to the question,
75x /100 = 15/16
Therefore, x = 15*100 / 75*16 = Rs. 5/4
SP of 1 orange with 20% profit
= Rs. (5/4 * 120/100) = Rs. 3/2
Therefore, In Rs. 3/2, the number of oranges sold = 1
Therefore, In Rs. 24, the number of oranges sold
= 2/3 * 24 = 16

77. The selling price of 20 articles is equal to the cost price of 22 articles. The gain percentage is
1) 12%

2) 9%

3) 10%

4) 11%

5)None of these

Solution (3) .Here, a = 22 and b = 20, then


Therefore gain % = (a b) / b * 100%
= 22 20 / 20 * 100% = 10%
Jenisys Systems Pvt ltd www.todaysprint.com | www.ibpsexamguru.in

31

78. If the cost price of 23 toys is equal to selling price of 20 toys, then the gain or loss percentage is
1) 12

2) 14

3) 15

4) 12

5)None of these

Solution .(3)
Here, a = 23, b = 20
Therefore Required Percentage Profit
= (a b / b) * 100%
= (23 20) / 20 * 100% = 15%

79. The cost price of 24 apples is same as the selling price of 18 apples. The percentage of gain is
1) 12 %

2) 14 2/3%

3) 16 2/3%

4) 33 1/3%

5) None of these

Solution (4).
Here, 6 Apples are gained over 18 apples.
Therefore Gain % = ((6/18) * 100) % = 33 1/3 %

80. A dishonest dealer sells his goods at 10% loss on cost price but uses 20% less weight. What is his
profit or loss per cent ?
1) 12% loss

2) 22.5% gain

3) 13.9% loss

4) 12.5% gain

5) None of the above

Solution . (4) .
Here, a = 10% and b = 20%
According to the formula,
Required answer = [( ba )/ (100 b) * 100 ] %
= (20 10) / (100 20) * 100 %
= 10 / 80 * 100 % = 12.5 % gain.

Simple Interest
Jenisys Systems Pvt ltd www.todaysprint.com | www.ibpsexamguru.in

32

81. A sum becomes two times in 5 year at a certain rate of interest . Find the time in which the same
amount will be 8 times at the same rate of interest?
1) 35 year 2) 45 year 3) 55 year 4) 25 year 5) None of these
Solution: (1).
Here, n = 2 , m=8,
T1 = 5,
T2 = ?
Therefore T2 = ( (m 1) / (n 1) ) X T1 = ( (8 - 1) / ( 2 1 ) X 5 = 35 year

82. A certain sum in certain time becomes Rs.500 at the rate of 8 % per annum Simple Interest and the
same sum amounts to Rs.200 at the rate of 2% per annum Simple Interest in the same duration. Find the
sum and time.
1) Rs.100 and 50yr 2) Rs.200 and 25 yr 3) Rs.300 and 30 yr 4) Rs.400 and 35 year 5) None of these
Solution: (1). Here, R1 = 8% , R2 = 2% , A1 = Rs.500, A2 = Rs.200
Now, according to the formula,
P = ( A2R1 A1R2) / ( R1 R2) = (200 x 8 500 x 2 ) / (8 - 2) = (1600 1000) / 6
= 600/6
= Rs. 100
And
Time, T = (A1 A2) / (A2R1 A1R2) X 100
= ( 500 200 ) / ( 200 x 8 500 x 2 ) x 100
= 300 / 600 x 100
= 50 year

83. Simple interest for a sum of Rs.1550 for 2 year is Rs.20 more than the simple interest for Rs.1450 for
the same duration. Find the rate of interest.
1) 10%

2) 20%

3) 25%

4) 15%

5) None of these

Jenisys Systems Pvt ltd www.todaysprint.com | www.ibpsexamguru.in

33

Solution: (1). Given that, P1 = Rs. 1550, T1 = 2, P2 = Rs.1450, T2 = 2


According to the question,
= [ (P1 x R x T1) / 100 ] - [ (P2 x R X T2 ) / 100 ] = 20
= [ ( 1550 x 2 x R ) / 100 ] - [ ( 1450 x 2 x R ) / 100 ] = 20
= 200R / 100 = 20
R = 10%

84. 2/3 part of my sum is lent out at 3%, 1/6 part is lent out at 6% and remaining part is lent out at 12% .
All the three parts are lent out at simple interest. If the annual income is Rs.25, what is the sum?
1) Rs.500

2) Rs.650

3) Rs. 600

4) Rs. 450

5) None of the these

Solution: (1).
Let entire sum = P
According to the question,
2/3 P X 3% + 1/6 P X 6% + [ 1- ( 2/3 + 1/6) ] P x 12%
= 2P/3 X 3/100 + P/6 x 6/100 + [ 1 (4+1) / 6 ] 12P / 100 = 25
= 2P / 100 + P/100 + 2P/100 = 25
= 5P = 2500, P = 500

85. The simple interest on a sum of money is 1/16 of the principal and the number of years is equal to
the rate per cent annum. Find the rate per cent .
1) 2 %

2) 3 %

3) 4 %

4) 9 %

5) None of these

Solution: (1). Let Principal = P , Time = T and rate = T


According to the question,
( P X T X T ) / 100 = P / 16
(since time and rate are equal )
= T2 = 100 / 16 or T = 10/4 = 5/2 = 2 %
Jenisys Systems Pvt ltd www.todaysprint.com | www.ibpsexamguru.in

34

(Ratio)

86. If 30 % of A = 20% of B, then find the value of A : B.


1) 1:3

2) 3:2

3) 3:1

4) 2:3

5)None of the above

Solution: ( 4).
Given that , 30% of A = 20 % of B
A/B = 20/30 = 2/3 = A : B = 2:3

87. If 1/x : 1/y : 1/z = 2:3:5 , then determine x:y:z.


1) 6:15:10

2) 3:15:10

3) 15:3:10

4) 15:10:6

5) None of the above

Solution: (4) .
Let 1/x = 2K, 1/y = 3K and 1/z = 5K
Then , x = 1/2K, y = 1/3K and z = 1/ 5K
Therefore x:y:z = 1/2K : 1/3K : 1/5K
= : 1/3 : 1/5 = 15 : 10 : 6

88. If a/3 = b/8 , then ( a+3) : (b+8) is equal to


1) 3:8

2) 8:3

3) 5:8

4) 3:5

5) None of the above

Solution: (1).
Let a/3 = b/8 = K
Then, a = 3K, b = 8 K
Therefore (a+3) / (b+8) = (3K +3 ) / ( 8K + 8) = 3(K+1) / 8 (K+1) = 3/8
Therefore (a+3) : (b+8) = 3:8

Jenisys Systems Pvt ltd www.todaysprint.com | www.ibpsexamguru.in

35

89. If xy =36, then which of the following is correct?


1) x:9 = 4:y

2) 9:x = 4:y

3) x:17 = y:7

4) x:6 = y:6

5) none of the above

Solution: (1).
xy = 36
therefore xy = 4 x 9, x/9 = 4/y ; x :9 = 4 : y

90. If 2A = 3B = 4C, then find A:B:C


1) 2:3:4

2) 4:3:2

3) 6:4:3

4) 3:4:6

5) None of the above

Solution: (3).
Given , 2A = 3B = 4C
Now, 2A = 3B

(Alligation)
91. In what proportion must a grocer mix wheat at Rs.2.04 per kg and Rs.2.88 per kg so as to make a
mixture of worth Rs. 2.52 per kg?
1) 2:3 2) 3:2 3) 5 :3 4) 3:4 5) None of the above
Solution: (4).
According to the rule of allegation,
Cheaper price (Rs.2.04)
Dearer price (Rs.2.88)
Mean price ( Rs.2.52)
( 2.88 2.52 ) = Rs.0.36
(2.52 2.04) = Rs. 0.48
Therefore required ratio = 0.36 : 0.48 = 3 : 4
A/B = 3/2
Or A : B = 3:2
Jenisys Systems Pvt ltd www.todaysprint.com | www.ibpsexamguru.in

36

= ( 3 x 2) : ( 2 x 2) = 6 :4
Again, 3B = 4C
B/C = 4/3 Or B : C = 4:3
Therefore A : B : C = 6 : 4 : 3

92. The ratio milk and water mixture of four containers are 5:3:, 2;1, 3:2 and 7:4 , respectively . In which
container, is the quantity of milk relative to water minimum?
1) First

2) Second

3) Third

4) Fourth

5) None of these

Solution: (3).
Milk in first container = 5/8 = 0.625
Milk in second container = 2/3 = 0.66
Milk in third container = 3/5 = 0.6
Milk in fourth container = 7/11 = 0.636
So, it is clear that quantity of milk relative to water is minimum in third container

93. A merchant has 2000 kg of rice, one part of which he sells at 36% profit and the rest at 16% profit.
He gains 28% on the whole. Find the quantity sold at 16%
1) 400 kg

2) 300 kg

3) 900kg 4) 800 kg

5) None of the these

Solution:(4).
According to the rule of allegation,
Part I ( 16% )
Part II (36% )
Mean value (28% )
( 36 28 ) = 8
( 28 16 ) = 12
Part - I (16% ) : Part II (36%) = 8:12 = 2:3
Jenisys Systems Pvt ltd www.todaysprint.com | www.ibpsexamguru.in

37

Therefore Quantity sold at 16% profit


= [ a / (a+b) ] x total quantity
= 2/5 x 2000 = 800 kg

94. 300 g of salt solution has 40% salt in it. How much salt should be added to make it 50% in the
solution?
1) 40g

2) 60g

3) 70 g

4) 80 g

5) None of these

Solution: (2).
40 % is salt in 300 g of salt solution.
Then, quantity of salt = (40 x 300) / 100 = 120g
Now, by the condition in the question,
( 120 + x ) / (300 +x ) = 50 / 100
= (120 + x ) / ) 300 + x ) =
= 240 + 2x = 300 + x
Therefore x = 60 g

95. In a mixture of 60 L the ratio of acid and water is 2:1. If the ratio of acid and water is to be 1:2, then
the amount of water ( in litres ) to be added to the mixture is
1) 55

2) 60

3) 50

4) 45

5) None of these

Solution: (2)
Quantity of acid in the mixture = 2/3 x 60 = 40 L
Quantity of water in the mixture = 1/3 x 60 = 20 L
Let required quantity of water be x L.
According to the question,
40 / (20 + x) = = 80 = 20 +x
x = 60 L
Jenisys Systems Pvt ltd www.todaysprint.com | www.ibpsexamguru.in

38

(Alligation)
96. In what proportion must a grocer mix wheat at Rs.2.04 per kg and Rs.2.88 per kg so as to make a
mixture of worth Rs. 2.52 per kg?
1) 2:3

2) 3:2

3) 5 :3

4) 3:4

5) None of the above

Solution: (4).
According to the rule of allegation,
Cheaper price (Rs.2.04)
Dearer price (Rs.2.88)
Mean price ( Rs.2.52)
( 2.88 2.52 ) = Rs.0.36
(2.52 2.04) = Rs. 0.48
Therefore required ratio = 0.36 : 0.48 = 3 : 4

97. The ratio milk and water mixture of four containers are 5:3:, 2;1, 3:2 and 7:4 , respectively . In which
container, is the quantity of milk relative to water minimum?
1) First

2) Second

3) Third

4) Fourth

5) None of these

Solution: (3).
Milk in first container = 5/8 = 0.625
Milk in second container = 2/3 = 0.66
Milk in third container = 3/5 = 0.6
Milk in fourth container = 7/11 = 0.636
So, it is clear that quantity of milk relative to water is minimum in third container

98. A merchant has 2000 kg of rice, one part of which he sells at 36% profit and the rest at 16% profit.
He gains 28% on the whole. Find the quantity sold at 16%
1) 400 kg

2) 300 kg

3) 900kg

4) 800 kg

5) None of the these

Jenisys Systems Pvt ltd www.todaysprint.com | www.ibpsexamguru.in

39

Solution:(4).
According to the rule of allegation,
Part I ( 16% )
Part II (36% )
Mean value (28% )
(36 28) = 8
(28 16) = 12
Part - I (16% ) : Part II (36%) = 8:12 = 2:3
Therefore Quantity sold at 16% profit
= [ a / (a+b) ] x total quantity
= 2/5 x 2000 = 800 kg

99. 300 g of salt solution has 40% salt in it. How much salt should be added to make it 50% in the
solution?
1) 40g

2) 60g

3) 70 g

4) 80 g

5) None of these

Solution: (2).
40 % is salt in 300 g of salt solution.
Then, quantity of salt = (40 x 300) / 100 = 120g
Now, by the condition in the question,
( 120 + x ) / (300 +x ) = 50 / 100 = (120 + x ) / ) 300 + x ) =
= 240 + 2x = 300 + x
Therefore x = 60 g

100. In a mixture of 60 L the ratio of acid and water is 2:1. If the ratio of acid and water is to be 1:2, then
the amount of water ( in litres ) to be added to the mixture is
1) 55

2) 60

3) 50

4) 45

5) None of these

Jenisys Systems Pvt ltd www.todaysprint.com | www.ibpsexamguru.in

40

Solution: (2)
Quantity of acid in the mixture = 2/3 x 60 = 40 L
Quantity of water in the mixture = 1/3 x 60 = 20 L
Let required quantity of water be x L.
According to the question,
40 / (20 + x) = = 80 = 20 +x
x = 60 L

(Time & Work)


101. Aarti can do a piece of work in 6 days. In how many days will she complete the three time of work
of same type ?
1) 18 days

2) 21 days

3) 3 days

4) 6 days

5) None of the above

Solution: ( 1).
We have the important relation, more work , more time ( days)
Since A piece of work can be done in 6 days
Therefore three times of work of same type can be done in 6 x 3 = 18 days

102. A, B and C can complete a work in 2 h. If A does the job alone in 6 h and B in 5 h , how long will it
take for C to finish the job alone ?
1) 5 h 2) 7 h 3) 9 h 4) 4 h 5) None of the above
Solution: (2).
Let C alone can finish the job in x h.
According to the question,
Work done by A , B and C in 1 h =
= 1/6 + 1/5 + 1/x =
= 1/x = - 1/6 1/5 = ( 15 -5-6) / 30
Jenisys Systems Pvt ltd www.todaysprint.com | www.ibpsexamguru.in

41

= 4/30 = 2/15
Therefore x = 7 h

103. A and B together can complete a work in 3 days. They started together but after 2 days, B left the
work. If the work is completed after 2 more days, B alone could do the work in how many days ?
1) 5

2) 6

3) 7

4) 10

5) None of the above

Solution: (2)
( A+B )s 2 days work = 2 x 1/3 = 2/3
Remaining work = 1 2/3 = 1/3
A will complete 1/3 work in 2
A will complete 1 work in 6
A s 1 days work = 1/6
Bs 1 days work = 1/3 - 1/6 = 1/6
Therefore B will take 6 days to complete the work alone

104. P and Q can finish a work in 30 days. They worked at it for 10 days and then Q left . The remaining
work is done by P alone in 20 more days. How long will P take to finish the work alone?
1) 30 days

2) 20 days

3) 60 days

4) 50 days

5) None of the above

Solution: (1)
( P + Q ) s 10 days work = 1/30 x 10 = 1/3
Remaining work = ( 1 1/3 ) = 2/3
2/3 work is done by P in 20
Therefore whole work is done by P in
20 x 3/2 = 30 days

105. If 5 boys take 7 h to pack 35 toys, how many boys can pack 65 toys in 3 h ?
Jenisys Systems Pvt ltd www.todaysprint.com | www.ibpsexamguru.in

42

1) 26

2) 39

3) 45

4) 65

5) None of the above

Solution: (5).
Given , M1 = 5, M2 = ?, T1 = 7, T2 = 3,
W1 = 35 and W2 = 65
According to the formula ,
M1T1W2 = M2T2W1
= 5 x 7 x 65 = M2 x 3 x 35
Therefore M2 = ( 5 x 7 x 65 ) / ( 35 x 3) = 65 / 3 boys
(Time & Distance)

106. If speed of 3 1/3 m/s is converted to km/h, then it would be


1) 8 km/h

2) 9 km/h

3) 10 km/h

4) 12 km/h

5) None of these

Solution: (4).
Since 1 m/s = 18/5 km/h
Therefore 3 1/3 m/s = 10/3 m/s = 10/3 x 18/5 km/h
= 12 km/h

107. Two trains A and B travel from points X to Y and the ratio of the speeds of A to that of B is 2:7. Find
the ratio of time taken by A and B to reach from X to Y.
1) 2:5

2) 3:5

3) 3:8

4) 7:2

5) None of these

Solution: (4).
We know that speed is inversely proportional to time .
Given that,
( Speed of A ) : (Speed of B) = 2: 7
Therefore ( Time taken by A ) : ( Time taken by B)
= : 1/7 = 7:2
Jenisys Systems Pvt ltd www.todaysprint.com | www.ibpsexamguru.in

43

108. A thief is noticed by a policeman from a distance of 200 m. The thief starts running and the
policeman, chases him . The thief and the policeman run at the rate of 10 km/h and 11 km/h ,
respectively. The distance between them after 6 min will be
1) 100 m 2) 180 m 3) 150 m 4) 125 m 5) None of these
Solution: (1).
Relative speed of policeman with respect to thief = ( 11 -10 ) = 1 km/h
Now, relative distance covered by policeman in 6 min
= Speed x Time = 1 x 6/60
= 1/10 km = 100m
= The distance between the policeman and thief after 6 min = ( 200 100) = 100m

109. John started from A to B and Vinod from B to A . If the distance between A and B is 125 km and
they meet at 75 km from A, what is the ratio of Johns speed to that of Vinods speed?
1) 2:3

2) 3:2

3) 4:3

4) 5:4

5) None of these

Solution: (2).
Johns speed : Vinods speed
= 75 : ( 125 75 )
= 75 : 50 = 3 : 2

110. A is twice as fast as B and B is thrice as fast as C. The journey covered by C in 56 min will be
covered by A in
1) 5 1/3 min

2) 2 1/3 min

3) 7 1/3 min

4) 9 1/3 min

5) None of these

Solution: ( 4).
Let time taken by A = y
Let speed of C = x
Then, speed of B = 3x
Jenisys Systems Pvt ltd www.todaysprint.com | www.ibpsexamguru.in

44

Therefore speed of A = 6x
Now, ratio of speeds of A and C
= Ratio of time taken by C and A
6x : x = 56 :y
= 6x /x
= 56/y
Therefore y = 56/6 = 9 2/6
9 1/3 min

(Linear Equation)
111. Deepak has some hens and some goats. If the total number of animal heads is 90 and the total
number of animal feet is 248, what is the total number of goats Deepak has ?
1) 32

2) 36

3) 34

4) Cannot be determined

5) None of these

Solution: (3).
Let hens = H, goats = G
According to the questions,
H + G = 90
2H + 4G = 248
. (i)
..(ii)
On multiplying Eq. (i) by 2 and subtracting from Eq. (ii), we get
2H + 2G = 180
2H + 4G = 248
-

+-

____________
-2G = - 68
Jenisys Systems Pvt ltd www.todaysprint.com | www.ibpsexamguru.in

45

G = 34

112. The cost of 21 pencils and 9 clippers is Rs.819. What is the total cost of 7 pencils and 3 clippers
together?
1) Rs.204

2) Rs.409

3) Rs.273

4) Rs.208

5) None of these

Solution: (3).
Let cost of 1 pencil and 1 clipper be p and c, respectively.
Now, according to the question,
21p + 9c = Rs. 819
= 3 ( 7p + 3c ) = Rs.819
= 7p + 3c = Rs. 273
Cost of 7 pencils and 3 clippers = Rs.273

113. In a rare coin collection, there is one gold coin for every three non-gold coins. 10 more gold coins
are added to the collection and the ratio of gold coins to non-gold coins would be 1:2. Based on the
information; the total number of coins in the collection now becomes.
1) 90

2) 80

3) 60

4) 50

5) None of these

Solution: (1).
Let the number of gold coins initially be x and the number of non-gold coins be y.
According to the question,
3x = y
When 10 more gold coins, total number of gold coins become x+ 10 and the number of non-gold coins
remain the same at y
Now, we have 2 (10+x) = y
Solving these two equations, we get
x=20 and y = 60
Total number of coins in the collection at the end is equal to
Jenisys Systems Pvt ltd www.todaysprint.com | www.ibpsexamguru.in

46

= x+10+y = 20 + 10 + 60 = 90

114. In an examination, a student scores 4 marks for every correct answer and losses 1 mark for every
wrong answer. A student attempted all the 200 questions and scored 200 marks. Find the number of
questions, he answered correctly.
1) 82

2) 80

3) 68

4) 60

5)None of these

Solution: (2).
Let the number of correct answers be x and number of wrong answer be y .
Then , 4x y = 200 ..(i)
And
x + y = 200 (ii)
On adding Eqs. (i) and (ii), we get
4x - y = 200
x + y = 200
__________
5x = 400
X= 80

115. If 3x + y = 81 and 81x - y = 3, then what is the value of x ?


1) 17/16

2) 17/8

3) 17/4

4) 15/4

5) None of these

Solution: (2).
Given , 3x+y = 81
= 3x+y = 34
= x +y = 4
And 81x y = 3 or (34)x - y = 31
=xy=
Jenisys Systems Pvt ltd www.todaysprint.com | www.ibpsexamguru.in

47

..(i)
..(ii)
On solving the Eqs.(i) and (ii) , we get
2x = 17/4 = x = 17/8
(Permutation Combination Probability)

116. There is a 7-digit telephone number with all different digits. If the digit at extreme right and
extreme left are 5 and 6 respectively , find how many such telephone numbers are possible?
1) 120

2) 100000

3) 6720

4) 30240

5) None of the above

Solution. (3) .
There is a 7-digit telephone number but extreme right and extreme left positions are fixed. I e. 6 XXXXX
5
Required number of ways
= 8 X 7 X 6 X 5 X 4 = 6720

117. In a meeting between two countries, each country has 12 delegates. All the delegates of one
country shake hands with all delegates of the other country. Find the number of handshakes possible?
1) 72

2) 144

3) 288

4) 234

5) None of the above

Solution. (2). Total number of hand shakes


= 12 X 12 = 144

118. In how many different ways, 5 boys and 5 girls can sit on a circular table, so that the boys and girls
are alternate?
1) 2880

2) 2800

3) 2680

4) 2280

5) None of these

Solution. (1).
Consider a circular table : After fixing up one boy on the table ,the remaining can be arranged in 4! ways,
but boys and girls have to be alternate. There will be 5 places, one place each between two boys. These
are 5 place can be filled by 5 girls in 5! ways
Jenisys Systems Pvt ltd www.todaysprint.com | www.ibpsexamguru.in

48

Hence , by the principles of multiplication, the required number of ways = 4! X 5! =2880.

119. A committee of 5 members is going to be formed from 3 trainees, 4 professors and 6 research
associates. How many ways can they be selected, if
(i). in committee, there are 2 trainees and 3 research associates?
1) 15

2) 45

3) 60

4) 9

5) None of these

Solution: (i). (1) Required number = 3 C 2 X 6 C 3


=
=

x
x

= 60

120. There are 4 professors and 1 research associate or 3 trainees and 2 professors?
1) 12

2) 13

3) 24

4) 52

5) None of the above

Solution: (1). Required number = 4C4 X 6C1 + 3 C3 X 4C2


=

= 1 X 6 + 1 X 6 =12

(Probability)

121. What is the probability that a card drawn at random from a pack of 52 cards is either a king or a
spade?
1) 17/52

2) 4/13

3) 3/13

4) 13/52

5) None of these

Solution:(2)
Required probability =3/52 + 13/52 = 16/52 = 4/13
(Note : why 13/52 because there are 13 spades and why 3/52 instead of 4/52 (there are four
kings)because one king is already counted in spades).

Jenisys Systems Pvt ltd www.todaysprint.com | www.ibpsexamguru.in

49

122. If three unbiased coins are tossed simultaneously, then the probability of exactly two heads, is
1) 1/8

2) 2/8

3) 3/8

4) 4/8 5) none of the above

Solution: (3).
n(S) = 23 =8
Let E = event of getting exactly two heads
={ (H,H,T), (H,T,H), (T,H,H) }
= n(E) = 3
Therefore Required probability =3/8

123. When two dice are rolled, what is the probability that the sum of the numbers appeared on them is
11?
1) 1/6

2) 1/18

3) 1/9

4) 1

5) none of the above

Solution: (2) .
n(S) =36
n(E) = {(5,6),(6,5)} =2
Therefore P(E) = n(E) / n(S) = 2/36 = 1/18

124. A basket contains three blue and four red balls. If three balls are drawn at random from the basket,
what is the probability that all the three balls are either blue or red?
1) 1

2) 1/7

3) 3/14

4) 3/28

5) None of the above

Solution: (2)
Probability to be a blue = 3 C3 / 7 C3 47
Therefore Required Probability = 3C3 / 7C3 = 4 C3 / 7C3 = 5/35 = 1/7

125. A number is selected at random from the set { 1,2,3,,50}. The probability that it is a
prime, is
1) 0.1

2) 0.2

3) 0.3

4) 0.7

5) None of the above

Jenisys Systems Pvt ltd ww.todaysprint.com | www.ibpsexamguru.in

50

Solution:(3)
n (S) =50
Prime numbers are = 2,3,5,7,11,13,17,19,23,29,31,37,41,43,47
Therefore n(E)=15
P(E) = 15/50 = 3/10 =0.3

(Probability)

126. What is the probability that a card drawn at random from a pack of 52 cards is either a king or a
spade?
1) 17/52

2) 4/13

3) 3/13

4) 13/52

5) None of these

Solution:(2)
Required probability =3/52 + 13/52 = 16/52 = 4/13
(Note : why 13/52 because there are 13 spades and why 3/52 instead of 4/52 (there are four
kings)because one king is already counted in spades).

127. If three unbiased coins are tossed simultaneously, then the probability of exactly two heads, is
1) 1/8

2) 2/8

3) 3/8

4) 4/8

5) None of the above

Solution: (3).
n(S) = 23 =8
Let E = event of getting exactly two heads
={ (H,H,T), (H,T,H), (T,H,H) }
= n(E) = 3
Therefore Required probability =3/8

128. When two dice are rolled, what is the probability that the sum of the numbers appeared on them is
11?
Jenisys Systems Pvt ltd www.todaysprint.com | www.ibpsexamguru.in

51

1) 1/6

2) 1/18

3) 1/9

4) 1

5) None of the above

Solution: (2) .
n(S) =36
n(E) = {(5,6),(6,5)} =2
Therefore P(E) = n(E) / n(S) = 2/36 = 1/18

129. A basket contains three blue and four red balls. If three balls are drawn at random from the basket,
what is the probability that all the three balls are either blue or red?
1) 1

2) 1/7

3) 3/14

4) 3/28

5) None of the above

Solution: (2)
Probability to be a blue = 3 C3 / 7 C3 47
Therefore Required Probability = 3C3 / 7C3 = 4C3 / 7C3 = 5/35 = 1/7

130. A number is selected at random from the set { 1,2,3,,50}. The probability that it is a
prime, is
1) 0.1

2) 0.2

3) 0.3

4) 0.7

5) None of the above

Solution:(3)
n (S) =50
Prime numbers are = 2,3,5,7,11,13,17,19,23,29,31,37,41,43,47
Therefore n(E)=15
P(E) = 15/50 = 3/10
=0.3

(Quadrilateral)
131. The area of a rectangular field is 15 times the sum of its length and breadth. If the length of that
field is 40 m, what is the breadth of that field?
1) 24 m

2) 25m

3) 28 m

4) 32 m

5) None of these

Jenisys Systems Pvt ltd www.todaysprint.com | www.ibpsexamguru.in

52

Solution (1).
Length of rectangle = 40m
Let breadth of = x
Then according to the question,
(40 + x) = 15 = 40 X x
600 + 15x = 40x = 25x = 600
x = 24m

132. A ground 100 x 80 m2 has, two cross roads in its middle. The road parallel to the length is 5 m wide
and the other road is 4 m wide, both roads are perpendicular to each other. The cost of laying the bricks
at the rate of Rs.10 per m2, on the roads, will be
1) 7000

2) 8000

3) 9000

4) 1000

5) None of these

Solution: (2).
Area to be paved with bricks
= 5 x 100 + 4 x 80 4 x 5 = 800m2
So cost of lying bricks = 800 x 10 = Rs. 8000

133. Find the distance between the two parallel sides of a trapezium if the area of the trapezium is 500
sq m and the two parallel sides are equal to 30 m and 20 m, respectively.
1) 20 cm

2) 15 cm

3) 18cm

4) 25 cm

5) None of the above

Solution: (1).
According to the question ,
Area = (30 + 20) x h 50 h = 500x 2
h = 20cm

Jenisys Systems Pvt ltd ww.todaysprint.com | www.ibpsexamguru.in

53

134. Floor of a square room of side 10 m is to be completely covered with square tiles, each having
length 50 cm. The smallest number of tiles needed is
1) 200

2) 300

3) 400

5) 500

5) None of the above

Solution: (3).
Area of square room = (10) =100sq m
= 100 x (100) sq cm
= 100 x 100 x 100 sq cm
Now, area of tile =(50) = 50 x 50 sq cm
Therefore Number of tiles needed = ___________
__________ = 400
Hence, 400 tiles will be needed

135. ABCD is a rhombus with diagonals AC and BD. Then, which one among the following is correct?
1) AC and BD bisect each other but not necessarily perpendicular to each other.
2) AC and BD are perpendicular to each other but not necessarily bisect each other
3) AC and BD bisect each other and perpendicular to each other
4) AC and BD neither bisect each other nor perpendicular to each other.
Solution: (3).
ABCD is a rhombus
Therefore AB = BC = CD = DA
And diagonals bisect each other at right angles
Option (3) is correct

(Triangle)
136. Find the area of a triangle whose sides measure 8 cm, 10cm and 12 cm .
Jenisys Systems Pvt ltd ww.todaysprint.com | www.ibpsexamguru.in

54

1) 8 square root of 63 sq.cm.


2) 5 square root of 63 sq.cm.
3) 6 square root of 53 sq.cm.
4)7 square root of 93 sq.cm.
5) None of these
Solution: (2).
Given that,
A = 8 cm, b= 10 cm and c = 12 cm
We know, that
S = (a+b+c)/2
= (8+10+12) / 2
= 30/2 = 15 cm
Therefore (s-a) = (15 -8 ) = 7cm
(s-b) = (15-10) = 5 cm
(s-c) = ( 15-12) = 3 cm
Therefore Area = square root of s(s - a) (s b) ( s c)
= square root of 15 x 7 x 5x 3
= square root of 1575
= square root of 25 x 63
= 5 square root of 63 sq.cm.

137. The three sides of a triangle are 15, 25 and x units. Which one of the following is correct?
1) 10< x < 40

2) 10 x 40

3) 10 x <40

4) 10 < x 40

5) None of these

Solution: (1).
In a triangle ,

Jenisys Systems Pvt ltd www.todaysprint.com | www.ibpsexamguru.in

55

Sum of two sides is always greater than 3rd side i.e.,


x<25 +15 = 40 .(i)
Difference of two sides is always less than
3rd side i.e., 25 15 = 10 < x (ii)
Form Eqs. (i) and (ii) , we get
10 < x < 40

138. The sides of a triangle area in the ratio of 1/3 : : 1/5 and its perimeter is 94 cm . Find the length of
the smallest side of the triangle.
1) 18 cm

2) 22.5 cm

3) 24 cm

4) 27 cm

5) None of these

Solution: (3).
Given ratio = 1/3 : : 1/5
= 20:15:12
Let lengths of the sides be 20x, 15x and 12x.
Then, according to the question,
20x + 15x + 12x = 94 => 47x = 94
X = 94 / 47 =2
Smallest side = 12x = 12 * 2 = 24 cm.

139. The area of an equilateral triangle is 4 square root of 3 sq.cm. Find the length of each side of the
triangle.
1) 3 cm

2) 2 square root of 2 cm 3) 2 square root of 3 cm

4) 4 cm

5) None of these

Solution: (4).
Area of equilateral triangle = square root of 3/4 a2
= 4 square root of 3 = square root of 3 / 4 a2
a2 = 16 [ a = side ]
Jenisys Systems Pvt ltd www.todaysprint.com | www.ibpsexamguru.in

56

therefore a = square root of 16 = 4 cm

140. The sides of a right angled triangle are equal to three consecutive numbers expressed in
centimeters. What can be the area of such a triangle ?
1) 6 cm2

2) 8cm2

3) 10cm2

4) 12cm2

5) None of these

Solution: (1).
Since, the triangle is right angled.
Therefore all the three consecutive sides must satisfy Pythagoras theorem
Therefore 3, 4 and 5 are the sides of triangle which satisfy Pythagoras theorem.
Therefore ( 52 = 42 + 32 )
Therefore Area of triangle = x 4 x 3 = 6 cm2.

(Circle)
141. The area of a sector of a circle of radius 36 cm is 72* 22/7 cm2. The length of the corresponding are
of the sector is ?
1) 22/7 cm

2) 2 * 22/7 cm

3) 3 * 22/7 cm

4) 4 * 22/7 cm

5) None of these

Solution: (4).
Given area of sector = 72 * 22/7 cm2
= (72 x 360) / (36 x36) = = 20
Now length of arc = (22/7 * r ) /180
= ( 22/7 * 36 * 20) / 180
= 4 22/7 cm

142. If the radius of a circle is increased by 6% find the percentage increase in its area .
1) 15%

2) 12.36%

3) 8.39%

4) 17%

5) None of these

Solution: (2).
Jenisys Systems Pvt ltd www.todaysprint.com | www.ibpsexamguru.in

57

Given that a = 6
according to the formula, percentage increase in area
= [ 2a + (a /100)] %
= [ 2 x 6 + (36 /100) ] %
= [ 12+ 0.36]% = 12.36%

143. The circumferences of two circles are in the ratio 2 :3. what is the ratio of their areas?
1) 2 : 3

2) 4 : 9

3) 1 : 3

4) 8 : 27

5) None of these

Solution: (2).
Let the radii of two circles are r1 and r2 respectively.
Given,
Circumstances of 1 circle / circumstances of 2 circle = 2/3
2 * 22/7 * r1 / 2 * 22/7 * r2
=2/3 = r1 / r2 = 2/3
=[ r1 / r2 ] = 4 /9
Therefore Area of 1 circle / area of 2

circle = ( - * r1)2 / (22/7 * r2)2

= [ r1 / r2 ] =4/9

144. The radius of a circle is so increased that its circumstances increased by 5% . The area of the circle,
then increases by
1) 12.5%

2) 10.25%

3) 10.5%

4) 11.25%

5) None of these

Solution: (2).
Increase in circumstances of circle = 5%
Therefore Increase in radius is also 5%.
Now, increase in area of circle
[ 2a + (a /100) ]%
Jenisys Systems Pvt ltd www.todaysprint.com | www.ibpsexamguru.in

58

where ,a = increase in radius


= [ 2 x 5 + (5x5) / 100 ] % = 10.25%

145. The area of a circle is increased by 22 sq cm when its radius is increased by 1 cm. Find the original
radius of the circle .
1) 6 cm

2) 3.2 cm

3) 3 cm

4) 3.5 cm

5) None of these

Solution: (3).
Let original radius be r.
Then , according to the question
22/7 (r + 1) 22/7 * r = 22
X [(r + 1) 2r]2 = 22
22/7 x (r +1 + r) (r + 1 r )= 22
2r + 1 = 7 = 2r = 6
r = 6/2 = 3cm

(Circle)

146. The area of a sector of a circle of radius 36 cm is 72* 22/7 cm2. The length of the corresponding are
of the sector is ?
1) 22/7 cm

2) 2 * 22/7 cm

3) 3 * 22/7 cm

4) 4 * 22/7 cm

5) None of these

Solution: (4).
Given area of sector = 72 * 22/7 cm2
= (72 x 360) / (36 x36) = = 20
Now length of arc = (22/7 * r ) /180
= ( 22/7 * 36 * 20) / 180
= 4 22/7 cm
Jenisys Systems Pvt ltd www.todaysprint.com | www.ibpsexamguru.in

59

147. If the radius of a circle is increased by 6% find the percentage increase in its area .
1) 15%

2) 12.36%

3) 8.39%

4) 17%

5) None of these

Solution: (2).
Given that a = 6
according to the formula, percentage increase in area
= [ 2 x 6 + (36 /100) ] %
= [ 12+ 0.36]% = 12.36%

148. The circumferences of two circles are in the ratio 2 :3. what is the ratio of their areas?
1) 2 : 3

2) 4 : 9

3) 1 : 3

4) 8 : 27

5) None of these

Solution: (2).
Let the radii of two circles are r1 and r2 respectively.
Given,
Circumstances of 1 circle / circumstances of 2 circle = 2/3
2 * 22/7 * r1 / 2 * 22/7 * r2
=2/3 = r1 / r2 = 2/3
=[ r1 / r2 ] = 4 /9
Therefore Area of 1 circle / area of 2 circle = (- * r1)2 / (22/7 * r2)2
= [ r1 / r2 ] =4/9

149. The radius of a circle is so increased that its circumstances increased by 5% . The area of the circle,
then increases by
1) 12.5%

2) 10.25%

3) 10.5%

4) 11.25%

5) None of these

Solution: (2).
Increase in circumstances of circle = 5%
Jenisys Systems Pvt ltd www.todaysprint.com | www.ibpsexamguru.in

60

Therefore Increase in radius is also 5%.


Now, increase in area of circle
[ 2a + (a2 /100) ]%
where ,a = increase in radius
= [ 2 x 5 + (5x5) / 100 ] % = 10.25%

150. The area of a circle is increased by 22 sq cm when its radius is increased by 1 cm. Find the original
radius of the circle .
1) 6 cm

2) 3.2 cm

3) 3 cm

4) 3.5 cm

5) None of these

Solution: (3).
Let original radius be r.
Then , according to the question,
22/7 (r + 1) 22/7 * r = 22
X [(r +1) r ] = 22
22/7 x (r +1 + r) (r + 1 r )= 22
2r + 1 = 7 = 2r = 6
r = 6/2 = 3cm

Profit & Loss

151. A merchant fixed the selling price of his articles at Rs.700after adding 40% profit to the cost price.
As the sale was very low at this price level, he decided to fix the selling price at 10% profit. Find the new
selling price.
1) Rs.450

2) Rs.490

3) Rs.500

4) Rs.550

5) None of these

Solution: (4).
Let cost price = Rs.x
According to the question,
Jenisys Systems Pvt ltd www.todaysprint.com | www.ibpsexamguru.in

61

x * (100+40)/100
=700
x = (700 x 100)/140
=500
Therefore New selling price = [500 x (100 + 10)/100]
= 5 x 110 = 550 = Rs.550

152. A merchant has 1000 kg of sugar, part of which he sells at 8% profit and the rest at 18% profit. He
gains 14% on the whole. The quantity sold at 18% profit is
1) 500 kg

2) 600kg

3) 400kg

4) 640 kg

5) None of these

Solution: (2).
Let the sugar sold at 8% gain = x
Therefore sugar sold at 18% gain = ( 1000 x )
Let CP of sugar = Rs.y per kg
Total CP = Rs.1000y
Therefore [(108/100) * xy ] + 118/100 (1000-x)y
= 114/100 *1000y
= 108xy + 118000y 118xy = 114000y
= 10x = 4000
Therefore x = 400
Therefore Quantity sold at 18% profit
= 1000 400 = 600 kg

153. By selling an umbrella for Rs.30, a shopkeeper gains 20%. During a clearance sale, the shopkeeper
allows a discount of 10% of the marked price. His gain percentage during the sale season is :
1) 7

2) 7.5

3) 8

4) 9

5) None of these

Jenisys Systems Pvt ltd www.todaysprint.com | www.ibpsexamguru.in

62

Solution: (3).
Given, selling price of an umbrella = Rs.30
Profit percentage = 20 %
Therefore Cost price of an umbrella
= ( 30 x 100) / 120
= Rs.25
During the clearance sale, selling price of an umbrella
= (30 x 90)/100
= Rs.27
Therefore required profit percentage = (27- 25)/25 * 100 = 8%

154. Cost of a packet of coffee powder and a litre of milk are Rs.20 and Rs.30, respectively. 10 cups of
coffee is made with one packet coffee powder and for each cup 200 ml of milk is used.If coffee is sold
at25% profit, the selling price of each cup of coffee is
1) Rs.12.50

2) Rs.6.25

3) Rs.8

4) Rs.10

5) None of these

Solution: (4).
Cost of coffee powder used in one cup = 20/10 = Rs.2
Cost of milk used in one cup
= 30/100 x 200 = Rs.6
Therefore cost of each cup of coffee
= 2+6 = Rs.8
To gain 25% profit, sale price of each cup of coffee = 125% of 8 = Rs.10

(Quadrilateral)
155. A dealer bought 80 cricket bats for Rs.50 each. He sells 20 of them at a gain of 5%.What must be
the gain percentage of the remaining bats, so as to get 10% gain on the whole?
1) 3 2/11 %

2) 12 %

3) 11 2/3 %

4) Rs.3350

5) None of these

Jenisys Systems Pvt ltd www.todaysprint.com | www.ibpsexamguru.in

63

Solution: (3).
Let required percentage profit = x%
According to the question,
10% of ( 80 * 50 )
= 5 % of (20 * 50) + x% of ( 60 * 50)
= ( 80 * 50 * 10) / 100
= ( 20 * 50 * 5)/100 + (60 * 50 * x)/100
= 80 = 10 + 6x
x= 70/6 = 11 2/3%
(Quadrilateral)

156. The diagonal of a square is 4 2 cm.The diagonal of another square whose area is double that of the
first square is
1) 8 cm

2) 82 cm

3) 42 cm

4) 6 cm

5) None of these

Solutions: (1).
Diagonal of a square = 2 a [ a = side]
42=2a
a = 4 cm
Now, area of square = a2 = (4)2 = 16
Side of a square whose area is 2 x 16
a21 = 32
a1 = 32 = a1 = 4 2
Now, diagonal of new square = 2a = 2 x 4 2 = 8 cm.

157. The diagonal of two squares are in the ratio of 3:2. Find the ratio of their areas.
1) 9:4

2) 9:2

3) 9:5

4) 9:7

5) None of these

Jenisys Systems Pvt ltd ww.todaysprint.com | www.ibpsexamguru.in

64

Solution: (1).
Let the diagonals of the squares be 3x and 2x.
Therefore ratio of their areas = (3x)2 / (2x)2 = 9/4
= 9:4

158. The perimeter of two squares is 12 cm and 24 cm. The area of the bigger square is how many times
that of the smaller?
1) 2 times

2) 3 times

3) 4 times

4) 5 times

5) None of these

Solution: (3)
We know that,
Perimeter of square = 4 x side
Therefore 4 x a = 12 [for smaller square]
a=3
Therefore area of smaller square = 3 x 3 = 9 cm2 .(i)
Now, 4 x b = 24
[for bigger square]
b=6
therefore area of bigger square
= 6 x 6 = 36 cm2 = 4 x 9 cm2
= 4 x Area of smaller square [ from Eq.(i)]
Hence, area of bigger square is 4 times that of smaller square.

159. Diagonals of a rhombus are 1 m and 1.5 m in length. The area of the rhombs is
1) 0.75 m2

2) 1.5 m2

3) 1.5 m2

4) 0.375m2

5) None of these

Solution: (1)
Area of rhombus = x d1 x d2
Jenisys Systems Pvt ltd www.todaysprint.com | www.ibpsexamguru.in

65

= x 1 x 1.5 = 0.75 m2

160. Find the distance between the two parallel sides of a trapezium if the area of the trapezium is 500
sq m and the two parallel sides are equal to 30 m and 20m, respectively.
1) 20 cm

2) 15 cm

3) 18 cm

4) 25 cm

5) None of these

Solution: (1)
According to the question,

Area = (30+20) x h
= 50h = 500 x 2
Therefore h = 20 cm
(Discount)

161. Two successive discounts of 20% and 20% are equivalent to a single discount of
1) 42%

2) 40%

3) 36%

4) 34%

5) None of these

Solution: (3).
Given, r1 = 20% and r2 = 20%
Therefore single discount equal to r1 and r2
= ( r1 +r2 [(r1 x r2) / 100 ] )%
= (20 + 20 [(20 x 20) /100] )
= 40 4 = 36%

162. A shopkeeper earns a profit of 12% on selling a book at 10% discount on the printed price. The ratio
of the cost price and the printed price of the book is
Jenisys Systems Pvt ltd www.todaysprint.com | www.ibpsexamguru.in

66

1) 45:56

2) 8:11

3) 47:56

4) 3:4

5)None of these

Solution: (1)
Let the CP of book = Rs.x
Then, SP of book = ( 100 +12) * x / 100
= 112x /100
Now, the printed price = Rs.y
Then, after discount, the SP
= (100 - 10) * y / 100
= 90y/100
Since, both SP are same
Then, 112x/100
=90y/100
= x/y = 45/56 = 45:56

163. A manufacturer marked an article at Rs.50 and sold it allowing 20% discount . If his profit was 25% ,
then the cost price of the article was
1) Rs.40

2) Rs.35

3) Rs.32

4) Rs.30

5) None of these

Solution: (3).
Since marked price of an article = Rs.50
Therefore SP of an article = (50 x (100 20)) / 100
= 50 *80 / 100 = Rs.40
Hence, cost price of an article = (40 x 100) / (100 + 25)
= (40 x100) / 125 = Rs.32

164. By selling an article at 3/4th of the marked price, there is a gain of 25% . The ratio of the marked
price and the cost price is
Jenisys Systems Pvt ltd www.todaysprint.com | www.ibpsexamguru.in

67

1) 5:3

2) 3:5

3) 3:4

4) 4:3

5) None of these

Solution: (1).
Let MP of an article = Rs.x
Therefore SP of an article = Rs. x
and CP of an article = (3x/4) * (100 / 100+25)
= 3x/4 * 100/125 = Rs.3x/5
Required ratio = x: 3x/5 = 5:3

165. A retailer offers the following discount schemes for buyers on an article .
I. Two successive discounts of 10%.
II. A discount of 12% followed by a discount of 8%.
III. Successive discounts of 15% and 5%.
IV. A discount of 20%.
The selling price will be minimum under the scheme
1) I

2) II

3) III

4) IV

5) None of these

Solution: (4).
I. Equivalent single discount to 10% and 10%
= ( 10 + 10 ( 10 x 10)/100 )% = 19%
II. Equivalent single discount to 12% and 8%
= 12+8 (12*8)/100
= 20 - 0.96 = 19.04%
III. Equivalent single discount to 15% and 5%
= 15 + 5 ( 15 x 5)/100
= 20 0.75 = 19.25%
IV. Equivalent single discount to 20% = 20%

Jenisys Systems Pvt ltd www.todaysprint.com | www.ibpsexamguru.in

68

So, the selling price will be minimum under the scheme IV as in this scheme, the discount is maximum.

(Simple Interest)
166. A sum of money amounts to Rs.2240 at 4% per annum simple interest in 3 year. The interest on the
same sum for 6 months at 3.5% per annum is
1) Rs.30

2) Rs.50

3) Rs.35

4) Rs.150

5) None of these

Solution: (3)
If the sum be Rs. P ,then
2240P = 2240 = + P 2240 = P
Therefore P= x = Rs.2000
Now, required interest .SI= = 2000 - = Rs.35

167. A certain sum at simple interest amounts to Rs.1350 in 5 year and to Rs.1620 in 8 year. What is the
sum?
1) Rs.700

2) Rs.800

3) Rs.900

4) Rs.1000

5) None of these

Solution: (3 ).
Given A1 = Rs 1350, A2= Rs. 1620
T1= 5yr and T2 = 8yr
Let principal amount be Rs P.
Therefore In time = 8 5 =3yr
Simple interest will be
1620-1350 =Rs .270
Therefore R =
=

R = 10%

Jenisys Systems Pvt ltd www.todaysprint.com | www.ibpsexamguru.in

69

Therefore P =

= Rs. 900

168. The simple interest on a sum of money at 8% per annum for 6 year is half the sum. What is the
sum?
1) Rs.4800

2) Rs.6000

3) Rs.8000

4) Data is inadequate

5) None of these

Solution: (4 ) Let Sum = P


Then according to the question
SI =
Therefore = P =
Therefore It is clear that data is inadequate

169. A sum of Rs.1550 was lent partly at 5% and partly at 8% per annum simple interest . The total
interest received after 4 yr was Rs.400. The ratio of the money lent at 5% to that lent at 8% is
1) 16:15

2) 17:15

3) 16:13

4) 16:19

5) None of these

Solution: ( 1) Let the sum lent at 5% = P


Therefore Sum lent at 8% = (1550 P)
Then
= 20 P 32 P +1550 x 32=40000
= -12P + 49600 = 40000
=

-12P = -9600

Therefore

P = Rs. 800

Sum lent at 8% = 1550- 800=Rs. 750


Therefore required ratio =800:750 =16:15

Jenisys Systems Pvt ltd www.todaysprint.com | www.ibpsexamguru.in

70

170. Neeta borrowed some money at the rate of 6% per annum for the first 3yr, at the rate of 9% per
annum for the next 5 year and at the rate of 13% per annum for the period beyond 8 year. If she pays a
total interest of Rs.8160 at the end of 11 year, how much money did she borrow?
1) Rs.12000

2) Rs.10000

3) Rs.8000

4) Data is inadequate

5) None of these

Solution: ( 3) Let the sum borrowed = P


Then , according to the question
+

= 8160

= 8160

= 102P/100 = 8160
P=
Therefore P= Rs.8000.

Average
171. 3 year ago, the average age of a family of 5 members was 17 year. Even after the birth of a child,
the average is same today. Find out the age of child.
1) 1 yr

2) 3 yr

3) 2 yr

4) 2yr

5) None of these

Solution (4).
3 yr ago, total age of 5 members of the family =17 5 =85 yr
Present age of all the members of the family =85 + 3 5 = 100 yr
Let the age of the child = x yr
Present average age of family =
( At present total number of members = 5 +1 =6 )
17= 102 = 100 + x
Therefore x = 2 yr

Jenisys Systems Pvt ltd www.todaysprint.com | www.ibpsexamguru.in

71

172. The average age of three boys is 15 yr.If the ratio of their ages is 3:5:7, what is the age of the oldest
boy?
1) 7yr

2) 14yr

3) 20yr

4) 21yr

5) None of these

Solution (4 ).
Total age of all the three boys = 3 15 = 45 yr
Ratio of ages = 3:5:7
Therefore Age of the oldest boy =
x 45 =
21 yr

173. A person bought some oranges worth Rs.36 from each of the five markets at Rs.1, Rs.1.50, Rs.1.80,
Rs.2 and Rs.2.25 per orange, respectively. What is the average price of an orange?
1) Rs.1.91

2) Rs.2.00

3) Rs.1.58

4) Rs.1.80

5) None of these

3. Solution ( 3 ).
Number of the oranges bought by the person
= + + + + =36+ 24 +20 + 18 +16 = 144
Total expenditure = 36 x 5 = Rs.180
Thus ,average price of each orange = Rs.
= Rs.1.58

174. If the average of the ages of Rakesh and Mohan is 15, average of the ages of Mohan and Ramesh is
12 and the average of the ages of Rakesh and Ramesh is 13, then the age of Mohan, is
1) 16

2) 13

3) 14

4) 12

5) None of these

Solution (3 ).
Let ages of Rakesh, Mohan and Ramesh be R, M, and r, respectively.
Then , R + M = 15 x 2 = 30 ----- (i)
M + r = 12 x 2 =24 -----(ii)
Jenisys Systems Pvt ltd www.todaysprint.com | www.ibpsexamguru.in

72

r + R = 13x2 = 26 -----(iii)
Adding Eqs . (i) , (ii) and (iii) we get 2 ( R + M + r ) = 30 + 24 + 26 = 80
R + M + r = 40
Subtracting Eq (iii) from Eq. (iv), we get
M = 40 26 = 14

175. Nine friends have a dinner in a hotel. Eight of them spent Rs.12 each on their meals and the ninth
spent Rs.16 more than the average expenditure of all the nine. Find out the total money spent by them.
1) Rs.126

2) Rs.135

3) Rs.111

4) Rs.141

5) None of these

Solution ( 1 ).
Let average expenditure of 9 persons = x
According to the question 12 * 8 + ( x + 16 ) = 9 x
8x = 112 x =
= 14
Therefore , Total money = 9x = 9 * 14 = Rs.126

Percentage
176. A number increased by 137 % and the increment is 33. The number is
1) 27

2) 22

3) 24

4) 25

5) None of these

Solution: (3).
Let the number be x.
Then, 137.5% of x = 33
Therefore x = (33 x 100) /137.50
= 24

177. As salary is 20% less than Bs salary. Then Bs salary is more than As salary by
Jenisys Systems Pvt ltd www.todaysprint.com | www.ibpsexamguru.in

73

w.todaysprincmz

1) 33 %

2) 16 2/3 %

3) 20%

4) 25%

5) None of these

Solution: (4).
Given, a = 20%
Required percentage = a / (100 a) X 100%
= 20 / (100-20) X 100%
= 20/80 X 100% = 25%

178. If the price of petrol is increased by 20%, by what percentage should the consumption be
decreased by the consumer, if the expenditure on petrol remains unchanged?
1) 16 2/3 %

2) 6 2/3%

3) 8%

4) 15%

5) None of these

Solution: (1).
Here , a = 20%
Required percentage loss = a /(100 + a) X 100%
= 20 / (100+20) X 100%
= 20/120 x 100% = 16 2/3%

179. A water pipe is cut into two pieces. The longer piece is 70% of the length of the pipe. By how much
percentage is the longer piece longer than the shorter piece?
1) 140%

2) 400/3 %

3) 40%

4) All of the these

5) None of these

Solution: (2).
Increase in percentage of longer pipe compared to shorter pipe
= 70 - 30 / 30 X 100%
= 40/30 X 100% = 400/3 %.

180. A man losses 20% of his money. After spending 25% of the remainder, he has Rs.480 left. What is
the amount of money he originally had?
1) Rs.600

2) Rs.720

3) Rs.800

4) Rs.840

5) None of these

Jenisys Systems Pvt ltd | www.ibpsexamguru.in

74

Solution: (3).
Let a man have Rs.x
Remainder money, after losses 20% of his money
= x 20% of x =x (x x20)/100
= 5x x / 5
= 4x/5
Remainder money, after spending 25% of his remainder money = 4x/5 25% of (4x /5)
= 4x/5 - (4x/5 X 25/100)
= 4x/5 - x/5 = 4x-x / 5
=3x/5
According to the question,
3x/5 =480
x = 480 X 5 /3
= Rs.800

(Profit & Loss)


181. An article is sold for Rs.300 at a profit of 20%. Had it been sold for Rs.235, the loss percentage
would have been
(a) 3%

(b) 5%

(c) 6%

(d) 16%

Solution: (c ) Let the cost price of article =.


Then 120% of x=300
= 300 Therefore x= =. 250
Now SP = 235
Then ,loss percentage = 100 =6%

Jenisys Systems Pvt ltd www.todaysprint.com | www.ibpsexamguru.in

75

182. Meena purchased two fans each at Rs.1200. She sold one fan at the loss of 5% and other at the
gain of 10% . Find total gain or loss percent.
(a) 1.2% loss

(b) 1.2% profit

(c) 2.5% profit

(d) 2.5% loss

Solution (c ).
Total CP= 2 x 1200= Rs. 2400
SP at 5% loss = 1200 = . 1140
SP at 10% gain = 1200 = . 1320
Total SP = 1140 + 1320 = Rs. 2460
Therefore Gain = 2460 - 2400 = Rs.60
Therefore Gain % = 100% = 2.5%

183. The difference between the CP and SP of an article is Rs.240. If the profit is 20%,the selling price is
(a) Rs.1440

(b) Rs.1400

(c) Rs.1240

(d) Rs.1200

Solution (a )
Given Gain % = 20
Difference between SP and CP = 240
Now ,gain % = 100 20 = 100
CP = 240 5
CP = 1200
Therefore SP = 1200 + 240 =1440

184. If the difference between the selling prices of an article at profit of 6% and 4% is Rs.3 ,then the cost
price of the article should be
(a) Rs.100

(b) Rs.150

(c) Rs.175

(d) Rs.200

Solution ( b).
Let CP = x
Jenisys Systems Pvt ltd www.todaysprint.com | www.ibpsexamguru.in

76

According to the question, =3 =3


Therefore x = . 150
Direct Approach
According to the question, (5% - 4% ) =3 = 2% =3
= (2 x 50)% =3 x 50
Therefore 100% = Rs.150

185. A person sold his watch for Rs.75 and got a percentage profit equal to the cost price . The cost
price of the watch is
(a) Rs.40

(b) Rs.45

(c) Rs.50

(d) Rs.55

Solution (c ).
Let CP of the watch be Rs. x. Then ,According to the question.
= ( 75 x) 100 =
= + 100 7500 = 0
+ 150 50 7500 = 0
= x + 150) 50 + 150) = 0)
= + 150) 50)=0 = x = Rs.50

Mixture and Alligation


186. How many kilograms of tea worth Rs.25 per kg must be blended with 30 kg of tea worth Rs.30 per
kg, so that by selling the blended variety at Rs.30 per kg , so that by selling the blended variety at Rs.30
per kg, there should be a gain of 10% ?
1) .36kg

2) .40kg

3) 32 kg

4) 42kg

5) None of these

Solution: (1)
Let the quantity of tea worth Rs.25 be x kg
According to the question,
(25x + 30 x 30) x 110/100 = 30(30 + x)
Jenisys Systems Pvt ltd www.todaysprint.com | www.ibpsexamguru.in

77

=(275x + 9900) = (9000 + 300x)


=(275x + 9900) = (9000+300x)
= 300x 275x = 900
x = 900/25
x=36kg

187. A butler stole wine from a butt of sherry which contained 80% of spirit and he replaced it by wine
containing only 32% spirit. Then, the butt was of 48% strength only. How much of the butt did he steal?
1) 1/4

2) 3/5

3) 2/5

4) 2/3

5) None of these

Solution: (4)
According to the rule of allegation

Therefore ratio = 32:16 = 2:1


Clearly, 1/3 of the butt of sherry was left and the butler stole 2/3 of the butt.

188. Tea worth Rs.126 per kg and Rs.135 per kg are mixed with a third variety in the ratio 1:1:2. If the
mixture is worth Rs.153 per kg, the price of the third variety per kg will be
1) Rs.169.5

2) Rs.170

3) Rs.175.5

4) Rs.180

5) None of these

Solution: (3)
Let the cost of third variety tea will be Rs. x per kg
According to the question,
126 + 135 + 2x / 4 = 153
= 261+2x = 4 x 153 2x = 612 -261 = 351
Therefore x = 351/2 = Rs.175.50
Jenisys Systems Pvt ltd www.todaysprint.com | www.ibpsexamguru.in

78

X = Rs.175.5
Hence, the cost of third variety tea is Rs.175.50 per kg.

189. 4 L are drawn from a container full of milk and then is filled with water. This operation is
performed three more times. The ratio of the quantity of milk left in the container and that of water is
16:65. How much milk did the container hold initially?
1) 24 L

2) 12 L

3) 15 L

4) 25 L

5) None of these

Solution: (2).
According to the formula,
After n operations, quantity of pure liquid
= [ a(1-b/a)n ] units
Here, b =4, n=4, a = ?
Quantity of milk left in the container after four operations = [ a(1-4/a)4]L
Then, according to the question,
a(1- 4/a)4 / a-a(1-4/a)4 = 16/65 = ( 1-4/a)4 / 1-(1-4/a)4 = 16/65
=(1-4/a)4 = 16/65 16/65 (1-4/a)4
=81/65(1-4/a)4 = 16/65 = (1-4/a)4 = 16/81
= 1 4/a = 2/3 = 4/a = 1/3
Therefore a = 12L

190. A vessel is filled with milk and water. 70% of milk and 30% of water is taken out of the vessel. It is
found that the vessel is vacated by 55% and has 160 L mixture. Find the quantity of milk and water in
this mixture.
1)Milk = 100 L ; Water = 60 L
2)Milk = 50 L ; Water = 110 L
3)Milk = 70 L ; Water = 90 L
4)Milk = 60 L ; Water = 100 L
Jenisys Systems Pvt ltd ww.todaysprint.com | www.ibpsexamguru.in

79

5)None of these
Solutions : (1).
Here, the percentage value of water and milk that is taken from the vessel should be taken into
consideration.

Milk : water = 25:15 = 5 : 3


Therefore Quantity of milk in the remaining mixture
=5/8 x 160
= 100L
And quantity of water = 3/8 x 160 =60L

Mixture and Alligation


191. How many kilograms of tea worth Rs.25 per kg must be blended with 30 kg of tea worth Rs.30 per
kg, so that by selling the blended variety at Rs.30 per kg , so that by selling the blended variety at Rs.30
per kg, there should be a gain of 10% ?
1) .36kg

2) .40kg

3) 32 kg

4) 42kg

5) None of these

Solution: (1)
Let the quantity of tea worth Rs.25 be x kg
According to the question,
(25x + 30 x 30) x 110/100 = 30(30 + x)
=(275x + 9900) = (9000 + 300x)
=(275x + 9900) = (9000+300x)
= 300x 275x = 900
x = 900/25
x=36kg
Jenisys Systems Pvt ltd www.todaysprint.com | www.ibpsexamguru.in

80

192. A butler stole wine from a butt of sherry which contained 80% of spirit and he replaced it by wine
containing only 32% spirit. Then, the butt was of 48% strength only. How much of the butt did he steal?
1) 1/4

2) 3/5

3) 2/5

4) 2/3

5) None of these

Solution: (4)
According to the rule of allegation

Therefore ratio = 32:16 = 2:1


Clearly, 1/3 of the butt of sherry was left and the butler stole 2/3 of the butt.

193. Tea worth Rs.126 per kg and Rs.135 per kg are mixed with a third variety in the ratio 1:1:2. If the
mixture is worth Rs.153 per kg, the price of the third variety per kg will be
1) Rs.169.5

2) Rs.170

3) Rs.175.5

4) Rs.180

5) None of these

Solution: (3)
Let the cost of third variety tea will be Rs. x per kg
According to the question,
126 + 135 + 2x / 4
= 153
= 261+2x = 4 x 153
2x = 612 -261 = 351
Therefore x = 351/2 = Rs.175.50
X = Rs.175.5
Hence, the cost of third variety tea is Rs.175.50 per kg.

Jenisys Systems Pvt ltd www.todaysprint.com | www.ibpsexamguru.in

81

194. 4 L are drawn from a container full of milk and then is filled with water. This operation is
performed three more times. The ratio of the quantity of milk left in the container and that of water is
16:65. How much milk did the container hold initially?
1) 24 L

2) 12 L

3) 15 L

4) 25 L

5) None of these

Solution: (2).
According to the formula,
After n operations, quantity of pure liquid
= [ a(1-b/a)n ] units
Here, b =4, n=4, a = ?
Quantity of milk left in the container after four operations = [ a(1-4/a)4]L
Then, according to the question,
a(1- 4/a)4 / a-a(1-4/a)4 = 16/65 = ( 1-4/a)4 / 1-(1-4/a)4 = 16/65
=(1-4/a)4 = 16/65 16/65 (1-4/a)4
=81/65(1-4/a)4 = 16/65 = (1-4/a)4 = 16/81
= 1 4/a = 2/3 = 4/a = 1/3
Therefore a = 12L

195. A vessel is filled with milk and water. 70% of milk and 30% of water is taken out of the vessel. It is
found that the vessel is vacated by 55% and has 160 L mixture. Find the quantity of milk and water in
this mixture.
1) Milk = 100 L ; Water = 60 L
2) Milk = 50 L ; Water = 110 L
3) Milk = 70 L ; Water = 90 L
4) Milk = 60 L ; Water = 100 L
5) None of these
Solutions : (1).
Jenisys Systems Pvt ltd www.todaysprint.com | www.ibpsexamguru.in

82

Here, the percentage value of water and milk that is taken from the vessel should be taken into
consideration.

Milk : water = 25:15 = 5 : 3


Therefore Quantity of milk in the remaining mixture
=5/8 x 160
= 100L
And quantity of water = 3/8 x 160
=60Ls
(Ages)

196. 7yr ago, the ages of A and B were in the ratio 4:5 and 7 yr hence, they will be in the ratio 5:6. The
present age of B is
1) 56 yr

2) 63yr

3) 70yr

4) 77yr

5) None of these

Solution: (4)
Let 7 yr ago, ages of A and B are 4x yr and 5x yr, respectively.
Then, present age of A = 4x + 7
And present age of B = 5x + 7
Now, according to the question,
Therefore (4x + 7 + 7) / (5x+7+7) = 5/6
= 24x + 84 = 25x + 70
= x = 14
Hence, Bs present age = 5 x 14 + 7 = 77 yr

Jenisys Systems Pvt ltd www.todaysprint.com | www.ibpsexamguru.in

83

197. The present ages of two persons are 36 and 50 yr, respectively . if after n yr the ratio of their ages
will be 3 : 4, then the value of n is
1) 3

2) 4

3) 7

4) 6

5) None of these

Solution: (4)
According to the question,
( 36 + n ) / (50 + n) =
= 144 + 4n = 150 + 3n
Therefore n = 6

198. 10 yr before, the ratio of ages of A and B was 13:17. After 17 yr from now, the ratio of their ages
will be 10:11 . The present age of B is
1) 23yr

2) 40yr

3) 27yr

4) 44yr

5) None of the these

Solution (3).
Let the ages of A and B 10 yr before were 13x yr and 17x yr, respectively.
Then present age of A = 13x + 10
And present age of B = 17x + 10
According to the question ,
Since (13x + 10 + 17 ) / (17x + 10 + 17 ) = 10/11
= (13x + 27) / ( 17x + 27) = 10/11
= 143x + 297 = 170x + 270
= 27x = 27
Therefore x = 1
Hence, present age of B = 17 * 1 + 10
= 27yr

199. The age of a man after 15yr is 4 times the age of that man 15 yr before. His present age is
Jenisys Systems Pvt ltd www.todaysprint.com | www.ibpsexamguru.in

84

1) 10 yr

2) 15yr

3) 20 yr

4) 25 yr

5) None of these

Solution: (4).
Let present age of man be x yr.
According to the question ,
X + 15 = 4 (x 15 )
= x + 15 = 4x 60 = 3x = 75
Therefore x = 25
Hence, the present age of man is 25 yr.

200. Ravis brother is 3 yr elder to him. His father was 28 yr of age when his sister was born, while his
mother was 26 yr of age when he was born. If his sister was 4 yr of age when his brother was born , the
ages of Ravis father and mother respectively when his brother was born were
1) 32 yr and 23 yr

2) 32 yr and 29 yr

3) 35yr and 29 yr

4) 35yr and 33yr

5) None of these

Solution: (1).
When Ravi was born , his mothers age was 26 yr and his elder brother was 3 yr elder to him.
Therefore mothers age when brother was born
= 26 3 = 23 yr
Ravis father was 28 yr of age when his sister was born and his sister was 4 yr of age when his brother
was born.
Therefore age of father when brother was born
= 28 + 4 = 32 yr

Partnership
201. A,B and C entered into partnership in a business. A got 3/5 of the profit. B and C distributed the
remaining profit equally. If C got Rs.400 less than A, the total profit was
1) Rs.1600

2) Rs.1200

3) Rs.1000

4) Rs.800

5) None of these

Solution:(3).
Jenisys Systems Pvt ltd www.todaysprint.com | www.ibpsexamguru.in

85

Let the total profit be Rs.x


Then, As share in profit = Rs. 3/5 x
Remaining profit = x 3/5x = (5x-3x)/5 = 2x /5
Therefore Bs share in profit = Rs. x/5
Cs share in profit = Rs. x/5
According to the question,
( 3x/5 - x/5) = 400
2x/5 = 400; x = (400 x 5) / 2 = Rs. 1000

202. Sonu invested 10% more than the investment of Mona and Mona invested 10% less than the
investment of Raghu. If the total investment of all the three persons is Rs.5780, find the investment of
Raghu.
1) Rs.2010

2) Rs.2000

3) Rs.2100

4) Rs.2210

5) None of these

Solution: (2).
Let share of Raghu be 100. Then share of Mona = 90 and share of Sonu = 99
Sonu : Mona : Raghu = 99 : 90 : 100
Therefore Investment of Raghu
= 5780 / ( 99 + 90 + 100) x 100
= 5780/289 x 100 = Rs.2000

203. Anil is an active and Vimal is a sleeping partner in a business. Anil invests Rs.12000 and Vimal
invests Rs.20000. Anil receives 10% profit for managing, the rest being divided in proportion to their
capitals. Out of the total profit of Rs.9000, the money received by Anil is
1) Rs.4500

2) Rs.4800

3) Rs.4600

4) Rs.3937.50

5) None of these

Solution: (4).
For management, money received by Anil = 10% of 9000 = Rs.900
Balance = Rs.( 9000 900) = Rs.8100
Jenisys Systems Pvt ltd www.todaysprint.com | www.ibpsexamguru.in

86

Now,ratio of investments = 12000 : 20000 = 3 : 5


Therefore Anils share = Rs.8100 x 3 / 3+5
= Rs.8100 x 3/8 = Rs.3037.50
Therefore amount received by Anil = Rs.900 + 3037.50 = Rs.3937.50

204. Amitabh, Brijesh and Kamlesh enter a partnership with shares in the ratio of 7/2 : 4/3: 6/5.After 4
months, Amitabh increases his share by 50% . If the total profit at the end of the year be Rs.21600, what
will be the share of Brijesh in the profit?
1) Rs.8000

2) Rs.12000

3) Rs.4000

4) Rs.7000

5) None of these

Solution: (3).
Given, ratio = 7/2 : 4/3 : 6/5
= 7/2 x 30 : 4/3 x 30 : 6/5 x 30
= 105:40:36
Let investment of Amitabh = 105x
Investment of Brijesh = 40x
Investment of Kamlesh = 36x
Ratio of their investments
= [ 105x * 4 +150% * 8 of 105x] : 40x * 12) : (36x * 12)
= ( 420x + 150/100 * 105x * 8) : ( 480x) : (432x)
= (1680x) : (480x) : (432x)
= 35 : 10:9
Therefore Brijeshs share
= ( 21600 * 10/ 35+10+9)
= (21600 * 10/54) = Rs.4000

205. A and B started a business with Rs.20000 and Rs.35000 respectively. They agreed to share the
profit in the ratio of their capital . C joins the partnership with the condition that A,B and C will share
Jenisys Systems Pvt ltd www.todaysprint.com | www.ibpsexamguru.in

87

profit equally and pays Rs.220000 as premium for this, to be shared between A and B . This is to be
divided between A and B in the ratio of
1) 10:1

2) 1:10

3) 9:10

4) 10:9

5) None of these

Solution: (1).
Ratio of total capital of A and B
= 20000 x 12 : 35000 x 12
= 240000 : 420000
Now, C gives Rs.220000 to both make the capital equal.
Therefore As capital : Bs capital
= 240000 : 420000 + 200000 : 20000
______________
440000 : 440000
If A takes Rs.200000 and B takes Rs.20000 from C, then both have the equal capital.
Therefore required ratio of divided amount
= 200000 : 20000
20 : 2 = 10:1
Hence, A and B should divide the amount in the ratio of 10 : 1

Partnership
206. A,B and C entered into partnership in a business. A got 3/5 of the profit. B and C distributed the
remaining profit equally. If C got Rs.400 less than A, the total profit was
1) Rs.1600

2) Rs.1200

3) Rs.1000

4) Rs.800

5) None of these

Solution:(3).
Let the total profit be Rs.x
Then, As share in profit = Rs. 3/5 x
Remaining profit = x 3/5x = (5x-3x)/5 = 2x /5
Jenisys Systems Pvt ltd www.todaysprint.com | www.ibpsexamguru.in

88

Therefore Bs share in profit = Rs. x/5


Cs share in profit = Rs. x/5
According to the question,
( 3x/5 - x/5) = 400
2x/5 = 400; x = (400 x 5) / 2 = Rs. 1000

207. Sonu invested 10% more than the investment of Mona and Mona invested 10% less than the
investment of Raghu. If the total investment of all the three persons is Rs.5780, find the investment of
Raghu.
1) Rs.2010

2) Rs.2000

3) Rs.2100

4) Rs.2210

5) None of these

Solution: (2).
Let share of Raghu be 100. Then share of Mona = 90 and share of Sonu = 99
Sonu : Mona : Raghu = 99 : 90 : 100
Therefore Investment of Raghu
= 5780 / ( 99 + 90 + 100) x 100
= 5780/289 x 100 = Rs.2000

208. Anil is an active and Vimal is a sleeping partner in a business. Anil invests Rs.12000 and Vimal
invests Rs.20000. Anil receives 10% profit for managing, the rest being divided in proportion to their
capitals. Out of the total profit of Rs.9000, the money received by Anil is
1) Rs.4500

2) Rs.4800

3) Rs.4600

4) Rs.3937.50

5) None of these

Solution: (4).
For management, money received by Anil = 10% of 9000 = Rs.900
Balance = Rs.( 9000 900) = Rs.8100
Now,ratio of investments = 12000 : 20000 = 3 : 5
Therefore Anils share = Rs.8100 x 3 / 3+5
= Rs.8100 x 3/8 = Rs.3037.50
Jenisys Systems Pvt ltd ww.todaysprint.com | www.ibpsexamguru.in

89

Therefore amount received by Anil = Rs.900 + 3037.50 = Rs.3937.50

209. Amitabh, Brijesh and Kamlesh enter a partnership with shares in the ratio of 7/2 : 4/3: 6/5.After 4
months, Amitabh increases his share by 50% . If the total profit at the end of the year be Rs.21600, what
will be the share of Brijesh in the profit?
1) Rs.8000

2) Rs.12000

3) Rs.4000

4) Rs.7000

5) None of these

Solution: (3).
Given, ratio = 7/2 : 4/3 : 6/5
= 7/2 x 30 : 4/3 x 30 : 6/5 x 30
= 105:40:36
Let investment of Amitabh = 105x
Investment of Brijesh = 40x
Investment of Kamlesh = 36x
Ratio of their investments
= [ 105x * 4 +150% * 8 of 105x] : 40x * 12) : (36x * 12)
= ( 420x + 150/100 * 105x * 8) : ( 480x) : (432x)
= (1680x) : (480x) : (432x)
= 35 : 10:9
Therefore Brijeshs share
= ( 21600 * 10/ 35+10+9)
= (21600 * 10/54) = Rs.4000

210. A and B started a business with Rs.20000 and Rs.35000 respectively. They agreed to share the
profit in the ratio of their capital . C joins the partnership with the condition that A,B and C will share
profit equally and pays Rs.220000 as premium for this, to be shared between A and B . This is to be
divided between A and B in the ratio of
1) 10:1

2) 1:10

3) 9:10

4) 10:9

5) None of these

Jenisys Systems Pvt ltd www.todaysprint.com | www.ibpsexamguru.in

90

Solution: (1).
Ratio of total capital of A and B
= 20000 x 12 : 35000 x 12
= 240000 : 420000
Now, C gives Rs.220000 to both make the capital equal.
Therefore As capital : Bs capital
= 240000 : 420000
+ 200000 : 20000
______________
440000 : 440000
If A takes Rs.200000 and B takes Rs.20000 from C, then both have the equal capital.
Therefore required ratio of divided amount
= 200000 : 20000
20 : 2 = 10:1
Hence, A and B should divide the amount in the ratio of 10 : 1

Partnership
211. A,B and C entered into partnership in a business. A got 3/5 of the profit. B and C distributed the
remaining profit equally. If C got Rs.400 less than A, the total profit was
1) Rs.1600

2) Rs.1200

3) Rs.1000

4) Rs.800

5) None of these

Solution:(3).
Let the total profit be Rs.x
Then, As share in profit = Rs. 3/5 x
Remaining profit = x 3/5x = (5x-3x)/5 = 2x /5
Therefore Bs share in profit = Rs. x/5
Cs share in profit = Rs. x/5
Jenisys Systems Pvt ltd www.buy-ibpsmaster.com | www.ibpsexamguru.in

91

According to the question,


( 3x/5 - x/5) = 400
2x/5 = 400; x = (400 x 5) / 2 = Rs. 1000

212. Sonu invested 10% more than the investment of Mona and Mona invested 10% less than the
investment of Raghu. If the total investment of all the three persons is Rs.5780, find the investment of
Raghu.
1) Rs.2010

2) Rs.2000

3) Rs.2100

4) Rs.2210

5) None of these

Solution: (2).
Let share of Raghu be 100. Then share of Mona = 90 and share of Sonu = 99
Sonu : Mona : Raghu = 99 : 90 : 100
Therefore Investment of Raghu
= 5780 / ( 99 + 90 + 100) x 100
= 5780/289 x 100 = Rs.2000

213. Anil is an active and Vimal is a sleeping partner in a business. Anil invests Rs.12000 and Vimal
invests Rs.20000. Anil receives 10% profit for managing, the rest being divided in proportion to their
capitals. Out of the total profit of Rs.9000, the money received by Anil is
1) Rs.4500

2) Rs.4800

3) Rs.4600

4) Rs.3937.50

5) None of these

Solution: (4).
For management, money received by Anil = 10% of 9000 = Rs.900
Balance = Rs.( 9000 900) = Rs.8100
Now,ratio of investments = 12000 : 20000 = 3 : 5
Therefore Anils share = Rs.8100 x 3 / 3+5
= Rs.8100 x 3/8 = Rs.3037.50
Therefore amount received by Anil = Rs.900 + 3037.50 = Rs.3937.50

Jenisys Systems Pvt ltd www.buy-ibpsmaster.com | www.ibpsexamguru.in

92

214. Amitabh, Brijesh and Kamlesh enter a partnership with shares in the ratio of 7/2 : 4/3: 6/5.After 4
months, Amitabh increases his share by 50% . If the total profit at the end of the year be Rs.21600, what
will be the share of Brijesh in the profit?
1) Rs.8000

2) Rs.12000

3) Rs.4000

4) Rs.7000

5) None of these

Solution: (3).
Given, ratio = 7/2 : 4/3 : 6/5
= 7/2 x 30 : 4/3 x 30 : 6/5 x 30
= 105:40:36
Let investment of Amitabh = 105x
Investment of Brijesh = 40x
Investment of Kamlesh = 36x
Ratio of their investments
= [ 105x * 4 +150% * 8 of 105x] : 40x * 12) : (36x * 12)
= ( 420x + 150/100 * 105x * 8) : ( 480x) : (432x)
= (1680x) : (480x) : (432x)
= 35 : 10:9
Therefore Brijeshs share
= ( 21600 * 10/ 35+10+9)
= (21600 * 10/54) = Rs.4000

215. A and B started a business with Rs.20000 and Rs.35000 respectively. They agreed to share the
profit in the ratio of their capital . C joins the partnership with the condition that A,B and C will share
profit equally and pays Rs.220000 as premium for this, to be shared between A and B . This is to be
divided between A and B in the ratio of
1) 10:1

2) 1:10

3) 9:10

4) 10:9

5) None of these

Solution: (1).
Ratio of total capital of A and B

Jenisys Systems Pvt ltd www.buy-ibpsmaster.com | www.ibpsexamguru.in

93

= 20000 x 12 : 35000 x 12
= 240000 : 420000
Now, C gives Rs.220000 to both make the capital equal.
Therefore As capital : Bs capital
= 240000 : 420000 + 200000 : 20000
______________
440000 : 440000
If A takes Rs.200000 and B takes Rs.20000 from C, then both have the equal capital.
Therefore required ratio of divided amount
= 200000 : 20000
20 : 2 = 10:1
Hence, A and B should divide the amount in the ratio of 10 : 1

(Pipes and Cisterns)


216. Through an inlet, a tank takes 8 h to get filled up. Due to a leak in the bottom, it takes 2 h more to
get it filled completely. If the tank is full, how much time will the leak take to empty it ?
1) 16 h

2) 20 h

3) 32 h

4) 40 h

5) None of these

Solution: (4).
Let the leak takes x h to empty the tank.
Now, part filled by inlet in 1 h = 1/8
Part filled in 1 h when both tap and leak works together = 1/ 8+2 = 1/10
According to the question,
1/x = 1/8 1/10 = 5 4 /40 = 1/40
Therefore x = 40h

Jenisys Systems Pvt ltd www.buy-ibpsmaster.com | www.ibpsexamguru.in

94

217. Two pipes P and Q can fill a cistern in 12 and 15 min, respectively. If both are opened together and
at the end of 3 min, the first is closed. How much longer will the cistern take to fill ?
1) 8 min

2) 8 min

3) 5 min

4) 8 min

5) None of these

Solution: (a)
Part filled by pipe P in 1 min = 1/12
Part filled by pipe Q in 1 min = 1/15
Part filled by both pipes in 1 min
=1/12 +1/15 = 5 + 4 / 60 = 9/60
Now, part filled by both pipes in 3 min
= 3x 9 / 60 = 27/60 = 9/20
Therefore remaining part = 1 9 /20 = 11/20
Let the remaining part is filled by pipe Q in x min.
Then, x * 1/15 = 11/20
x = 15*11/20 = 3*11/4 = 33/4 = 8 min

218. There are three pipes connected with a tank. The first pipe can fill part of the tank in 1 h , second
pipe can fill 1/3 part of the tank in 1 h. Third pipe is connected to empty the tank. After opening all the
three pipes, 7/12 part of the tank can be filled in 1 h, then how long will third pipe take to empty the full
tank?
1) 3 h

2) 4 h

3) 5 h

4) 6 h

5)None of these

Solution: (2)
1st pipe takes 1 h to fill part of the tank. So, time taken to fill the whole tank (m) = 2h 2nd pipe takes 1
h to 1/3 part of the tank So, time taken to fill the whole tank (n) = 3h
Let 3rd pipe takes P h to empty the tank = x
Therefore 1/m + 1/n 1/x = 7/12
= + 1/3 1/x = 7/12
= 1/x = (6 + 4 - 7) /12 = 3/12 =
Jenisys Systems Pvt ltd www.buy-ibpsmaster.com | www.ibpsexamguru.in

95

Therefore x = 4 h

219. A cistern has three pipes A, B and C. Pipes A and B can fill it in 3 and 4 h, respectively, while pipe C
can empty the completely filled cistern in 1 h. if the pipes are opened in order at 3:00 pm, 4:00 pm and
5:00 pm, respectively, at what time will the cistern be empty?
1) 6:15 pm

2) 7:12 pm

3) 8:12 pm

4) 8:35 pm

5) None of these

Solution: (2).
Let the cistern gets emptied in m h after 3:00 pm.
Work done by A in m h, by B in ( m -1) h and by C in ( m-2) h = 0
= m/3 + m-1 / 4 (m-2) = 0
= 4m + 3( m-1) 12(m-2) = 0
= 5m = 21
M = 21/5 = 4.2
Therefore m = 4 h 12 min
Therefore time = 7 : 12 pm

220. Three pipes A, B and C can fill a tank in 30 min, 20 min and 10 min, respectively. When the tank is
empty, all the three pipes are opened. If A, B and C discharge chemical solutions P,Q and R respectively,
then the part of solution R in the liquid in the tank after 3 min is
1) 8/11

2) 5/11

3) 6/11

4) 7/11

5) None of these

Solution: (3).
Total quantity of solutions P, Q and R from A, B and C respectively, after 3 min
= 3/30 + 3/20 + 3/10 = 3 (2+3+6/60)
= 3 x 11 /60 = 11/20
Quantity of solution R in liquid in 3 min = 3/10
Therefore part of solution R = 3/10/11/20 = 3 x 20 / 10 x 11 = 6/11
Jenisys Systems Pvt ltd www.buy-ibpsmaster.com | www.ibpsexamguru.in

96

Time & Distance


221. A person riding a bike crosses a bridge with a speed of 54 km/h. what is the length of the bridge, if
he takes 4 min to cross the bridge?
1) 3600 m

2) 2800 m

3) 3500 m

4) 4500 m

5) None of these

Solution: (1)
Length of the bridge = Distance travelled by the person in 4 min
= Speed x Time
Speed = 54 km/h = 54 x 5/8 = 3 x 5 = 15 m/s
Time = 4 min = 4 x 60 = 240 s
Therefore required length = 15 x 240 = 3600m

222. Two men start together to walk a certain distance, one at 4 km/h and another at 3 km/h. the
former arrives half an hour before the latter. Find the distance.
1) 6 km

2) 9 km

3) 8 km

4) 7 km

5) None of these

Solution: (1).let total distance = x km


According to the question,
x/3 - x/4 = ; x/12 = = x = 6 km

223. Moving 6/7 of its usual speed a train is 10 min late. Find its usual time to cover the journey.
1) 25 min

2) 15 min

3) 35 min

4) 60 min

5) None of these

Solution: (4).
New speed = 6/7 of usual speed
Now, time taken = 7/6 of usual time
(7/6 of the usual time) (usual time) = 10 min
= 1/6 of the usual time = 10 min
Jenisys Systems Pvt ltd www.buy-ibpsmaster.com | www.ibpsexamguru.in

97

Therefore usual time = 60 min

224. A student walks from his house at 2 km/h and reaches his school late by 6 min. Next day, he
increases his speed by 1 km/h and reaches 6 min before school time. How far is the school from his
house?
1) 5/4 km

2) 7/4 km

3) 9/4 km

4) 11/4 km

5) None of these

Solution: (2).
Let the required distance = x
According to the question ,
x/ 5/2 - x / 7/2 = 12/60
(difference between two times = 6 + 6= 12min)
= 2x/5 - 2x/7 = 1/5
14x - 10x = 7
4x = 7
X = 7/4 km

225. A person can walk a certain distance and drive back in 6 h. He can also walk both ways in 10 h. how
much time will he take to drive both ways?
1) 2 h

2) 2 h

3) 5

4) 4 h

5) None of these

Solution: (1).
Since, he takes 10 h to walk both ways, the number of hours to walk one way for him will be 5. If he
walks one way and drives back the same way it takes him 6 h which means that the number of h taken o
drive one way is 6 5 = 1 h. Thus, number of hours to drive both ways will be 2.

(Wages)
226. A can do a piece of work in 9 days and B can do the same work in 15 days. If they work together, in
what ratio A and B will receive their wages?
Jenisys Systems Pvt ltd www.buy-ibpsmaster.com | www.ibpsexamguru.in

98

1) 3 :5

2) 5:3

3) 2:5

4) 5:2

5) None of these

Solution: (2).
As 1 days work = 1/9
Bs 1 days work = 1/15
As share : Bs share
= 1/9 : 1/15 = 5/45 : 3/45 = 5:3

227. A can finish a work in 15 days, B in 20 days and C in 25 days. All these three worked together and
earned Rs.4700. The share of C is
1) Rs.1200

2) Rs.1500

3) Rs.1800

4) Rs.2000

5) None of these

Solution: (1)
As 1 days work = 1/15
Bs 1 days work = 1/20
Cs 1 days work = 1/25
A, B and C worked together .
Therefore ( A+ B +C )s 1 days work
= 1/15 + 1/20 + 1/25
= (20 + 15 + 12) / 300
= 47/300
Days taken to complete work by A, B and C working together = 300/47
Therefore share of C = 1/25 x 300/47 x 4700
= Rs.1200.

228. A person can do a piece of work in 26 days and an another person can do the same work in 39 days.
If they work together, then by what per cent the wages of 1st person is more than that of 2nd person?
1) 25%

2) 35%

3) 15%

4) 50%

5) None of these

Jenisys Systems Pvt ltd www.buy-ibpsmaster.com | www.ibpsexamguru.in

99

Solution: (4).
Let 1st person be x and 2nd person be y.
Then, xs 1 days work = 1/26
Ys 1 days work = 1/39
xs share : ys share
= 1/26 : 1/39 = 3/78 : 2/78 = 3:2
Difference of ratio = 3 2 = 1
Therefore required percentage = x 100% = 50%
Therefore 1st persons wages is 50% more than the 2nd persons wages.

229. A sum of money is sufficient to pay As wages for 21 days and Bs wages for 28 days . the same
money is sufficient to pay the wages of both for
1) 24 days

2) 12 days

3) 12 days

4) 14 days

5) None of these

Solution: (2).
As 1 days work = 1/21
Bs 1 days work = 1/28
Therefore same money is sufficient to pay the wages of both for
= 1/ (1/21 + 1/28) = 21 x 28 / 21 + 28 = 21 x 28 / 49
= 3 x 4 = 12 days

230. A,B and C completed a work costing Rs.1800. A worked for 6 days, B worked for 4 days and C
worked for 9 days. If their daily wages are in the ratio of 5:6:4, how much amount will be received by A ?
1) Rs.600

2) Rs.500

3) Rs.900

4) Rs.450

5) None of these

Solution: (1).
Ratio of the wages of A, B and C = 5 : 6 : 4
As share : Bs share : Cs share
Jenisys Systems Pvt ltd www.buy-ibpsmaster.com | www.ibpsexamguru.in

100

= ( 6 x 5) : ( 4 x 6 ) : ( 9x 4)
= 30 : 24 : 36 = 5 : 4 : 6
Therefore As share = 5/15 x 1800 = Rs.600
(Problems Based on Trains)

231. 150 m long train running with the speed of 90 km/h to cross a bridge in 26s. what is the length of
the bridge ?
1) 500m

2) 600m

3) 659 m

4) 550m

5) None of these

Solution: (1).
Let length of bridge be x m.
We know, speed = Distance / Time
According to the question,
90 x 5/18 = 150 + x /26
= 25 x 26 = 150 + x 650 = 150 + x
Therefore x = 500m

232. A train crosses a bridge of length 150 m in 15 s and a man standing on it in 9 s. The train is travelling
at a uniform speed. Length of the train is
1) 225m

2) 200m

3) 135 m

4) 90 m

5) None of these

Solution: (1).
Let the length of the train be x m.
According to the question,
x/9 = Speed ------------(i)
and (x+150)/15 = Speed -------------(ii)
From Eqs.(i) and (ii), we get
x/9 = (x +150) / 15
Jenisys Systems Pvt ltd www.buy-ibpsmaster.com | www.ibpsexamguru.in

101

= x/3 = ( x+150) / 5
5x = 3x + 450
X = 225m

233. A train moving with uniform speed crosses a pole in 2 s and a 250 m long bridge in 7 s . Find the
length of the train .
1) 150m

2) 120m

3) 100m

4) 80m

5) None of these

Solution: (3). Let length of the train be L m.


According to the question,
L/2 = (L+250) / 7
7L = 2L +500
7L 2L = 500
5L =500
Therefore L = 500/5 = 100m

234. A train travelling with uniform speed crosses two bridges of lengths 300 m and 240 m in 21 s and 18
s, respectively. Find the speed of the train .
1) 72 km/h

2) 68 km/h

3) 65 km/h

4) 60 km/h

5) None of these

Solution: (1).
Let length of the train = L
According to the question,
L + 300/21 = L+ 240 / 18
L + 300 / 7 = L + 240 / 6
6L + 1800 = 7L + 1680
Therefore L = 120m
Taking the length of the 2nd bridge into consideration ,
Jenisys Systems Pvt ltd www.buy-ibpsmaster.com | www.ibpsexamguru.in

102

Speed of train = L + 240 / 18 = 120 + 240 /18 m/s


= 360/18 x 18/5 km/h
= 72 km/h

235. The relative speed of a train in respect of a car is 90 km/h when train and car are moving opposite
to each other . Find the actual speed of train , if car is moving with a speed of 15 km/h.
1) 80 km/h

2) 105 km/h

3) 75 km/h

4) 100 km/h

5) None of these

Solution: (3).
Relative speed of train
= speed of train + speed of car
90 = Speed of train +15
Therefore speed of train = 90 -15 = 75 km/h

(Boats and Streams)


236. If the speed for a swimmer in still water is 9 km/h. Find the downstream speed of the swimmer,
when the river is flowing with the speed of 6km/h.
1) 15 km/h

2) 18 km/h

3) 3 km/h

4) 12 km/h

5) None of these

Solution: (1).
Given,
Swimmers speed in still water = x = 9 km/h
Rate of stream = y = 6 km/h
Therefore speed downstream = x + y
= 9 + 6 = 15 km/h

237. A boatman rows 1 km in 5 min along the stream and 6 km in 1 h against the stream. The speed of
the stream is
1) 3 km/h

2) 6 km/h

3) 10km/h

4) 12km/h

5) None of these

Jenisys Systems Pvt ltd www.buy-ibpsmaster.com | www.ibpsexamguru.in

103

Solution: (1).
Let the speed of boat and stream be x and y km/h.
Therefore speed of boat along stream = ( x + y)km/h
and speed of boat against stream = ( x y ) km/h
According to the question,
x+y = 1 / (5/60) = 60/5
= x + y = 12
And x y = 6
On adding Eqs.(i) and (ii), we get 2x = 18
x = 18/2 = 9
On putting the value of x in Eq. (i), we get
9 + y = 12
y = 12 9 = 3
Hence, speed of boat = 9 km/h and speed of stream = 3 km/h

238. A motorboat can travel at 10 km/h in still water. It travelled 91km downstream in a river and then
returned to the same place, taking altogether 20h. the rate of flow of river is
1) 3 km/h

2) 4 km/h

3) 2km/h

4) 5 km/h

5) None of these

Solution: (1).
Given , speed of boat = 10 km/h
Let speed of flow of river = x km/h
Therefore Upstream speed of boat = (10 x )km/h and downstream speed of boat
According to question,
= (10 + x) km/h
91 / (10 - x) + 91 / (10 + x) = 20
= 91(10 + x + 10 x ) / ( 10 x) ( 10 + x) = 20
Jenisys Systems Pvt ltd www.buy-ibpsmaster.com | www.ibpsexamguru.in

104

= 91 (20) / 100 x2 = 20
= 91 = 100 x2
= x2 = 9
Therefore x = 3

239. A man can row against the current three-fourth of a kilo meter in 15 min and returns same distance
in 10 min, then ratio of his speed to that of current is
1) 3 : 5

2) 5 : 3

3) 1 : 5

4) 5 : 1

5) None of these

Solution: (4).
Let the speed of man and current be x and y km/h, respectively.
Speed upstream = (x y) km/h
Speed down stream = ( x + y) km/h
According to the question,
3 x 60 / 4 x 15 = x y
= x y = 3 and x 60/10 = x + y
= x + y = 9/2
On adding Eqs. (i) and (ii), we get 2x = 3 + 9/2
2x = 3 + 9/2
2x = 6 + 9/2
x=15/4
On putting the value of x in Eq.(ii), we get
15/4 + y = 9/2
y = 9/2 15/4 = 18 15/4
y=
Hence, speed of man x = 15/4 and speed of current y =
Hence, required ratio = 15/4 : = 5 : 1
Jenisys Systems Pvt ltd www.buy-ibpsmaster.com | www.ibpsexamguru.in

105

240. The speed of the current is 5 km/h . A motorboat goes 10 km upstream and back again to the
starting point in 50 min. The speed, (in km/h) of the motorboat in still water is
1) 20 km/h

2) 26 km/h

3) 25 km/h

4) 28 km/h

5) None of these

Solution: (3)
Let speed of boat be x km/h.
Given speed of current = 5km/h
Therefore Upstream speed of boat = ( x 5) km/h
Downstream speed of boat = ( x+5) km/h
According to the question,
(10 / x 5) + (10/ x + 5) = 50/60
10 ( x + 5 + x 5/ x2 25) = 5/6
= 12 X 2x = x2 25
= x2 24x 25 = 0
x2 25x + x 25 = 0
= ( x 25) (x+1) = 0
Therefore x = 25 [ since x - 1]

(Races)
241. In a 400 m race, A runs a t a speed of 16 m/s. If A gives B a start of 16 m and still beats him by 40 s,
what will be the speed of B ?
1) 6 m/s

2) 8 m/s

3) 15 m/s

4) 5.9 m/s

5) None of these

Solution: (4).
Time taken by A to cover 400m = Distance/Speed = 400/16 s = 65s
Therefore B covers (400 16) = 384 m in (25+40) = 65s
Therefore Bs speed = 384/65 = 5.9 m/s
Jenisys Systems Pvt ltd www.buy-ibpsmaster.com | www.ibpsexamguru.in

106

242. X covers 1 km in 8 min 40 s while Y covers the same distance in 10 min. By what distance does X
defeat Y?
1) 13 1/3 m

2) 133 2/3 m

3) 133 2/5 m

4) 133 1/3 m

5) None of these

Solution: (4)
Clearly , X beats Y by 80 s.
Distance covered by y in 600 s = 1000m ( since 10 min = 600s)
Distance covered by Y in 80 s
= 1000/600 x 80 = 400/3 m
= 133 1/3 m

243. A can run 40 m while B runs 50 m . In a km race, B beats A by which of the following distance?
1) 175 m

2) 225m

3) 335m

4) 200m

5) None of these

Solution: (4).
In a 50 m race, B beats A by 10 m . In 1 km race, B beats A by
( 10/50 x 1000)m = 200m

244. Arun and Bhaskar start from place P at 6:00 am and 7:30 am, respectively and run in the same
direction. Arun and Bhaskar run at 8 km/h and 12 km/h, respectively .Bhaskar overtakes Arun at
1) 10 : 30 am

2) 9 : 00 am

3) 11 : 30 am

4) 1 : 00 am

5) None of these

Solution: (1)
Distance between Arun and Bhaskar at 7:30 am = 8 x 1 = 12 km
Time taken by Bhaskar in covering a distance of 12 km = 12 / (12 - 8) = 3h
Therefore required time = 10 : 30 am

Jenisys Systems Pvt ltd www.buy-ibpsmaster.com | www.ibpsexamguru.in

107

245. A, B and C walk 1 km in 5 min, 8 min and 10 min, respectively . C starts walking from a point at a
certain time, B starts from the same point 1 min later and A starts from the same point 2 min later than
C. Then, A meet B and C at times
1) 2 min, 3 min

2) 4/3 min, 3 min

3) 2min, 5/3 min

4) 1 min, 2 min

5) None of these

Solution: (3).
A walks 1 km in 5 min.
Then, distance covered by B in 1 min
= 1000/5 = 200m
B walks 1 km in 8 min.
Then, distance covered by C in 1 min = 1000/8 = 125m
Let A meets B and C in X and Y min, respectively.
Then, according to the question, distance covered by C in ( x+2) min
= Distance covered by A in x min
= 100(x+2) = 200 x
= 100x + 200 = 200x
200 = 100x
Therefore x = 200/100 = 2 min

(Races)
246. In a 400 m race, A runs a t a speed of 16 m/s. If A gives B a start of 16 m and still beats him by 40 s,
what will be the speed of B ?
1) 6 m/s

2) 8 m/s

3) 15 m/s

4) 5.9 m/s

5) None of these

Solution: (4).
Time taken by A to cover 400m = Distance/Speed = 400/16 s = 65s
Therefore B covers (400 16) = 384 m in (25+40) = 65s
Jenisys Systems Pvt ltd www.buy-ibpsmaster.com | www.ibpsexamguru.in

108

Therefore Bs speed = 384/65 = 5.9 m/s

247. X covers 1 km in 8 min 40 s while Y covers the same distance in 10 min. By what distance does X
defeat Y?
1) 13 1/3 m

2) 133 2/3 m

3) 133 2/5 m

4) 133 1/3 m

5) None of these

Solution: (4)
Clearly , X beats Y by 80 s.
Distance covered by y in 600 s = 1000m ( since 10 min = 600s)
Distance covered by Y in 80 s
= 1000/600 x 80 = 400/3 m
= 133 1/3 m

248. A can run 40 m while B runs 50 m . In a km race, B beats A by which of the following distance?
1) 175 m

2) 225m

3) 335m

4) 200m

5) None of these

Solution: (4).
In a 50 m race, B beats A by 10 m . In 1 km race, B beats A by
( 10/50 x 1000)m = 200m

249. Arun and Bhaskar start from place P at 6:00 am and 7:30 am, respectively and run in the same
direction. Arun and Bhaskar run at 8 km/h and 12 km/h, respectively .Bhaskar overtakes Arun at
1) 10 : 30 am

2) 9 : 00 am

3) 11 : 30 am

4) 1 : 00 am

5) None of these

Solution: (1)
Distance between Arun and Bhaskar at 7:30 am = 8 x 1 = 12 km
Time taken by Bhaskar in covering a distance of 12 km = 12 / (12 - 8) = 3h
Therefore required time = 10 : 30 am

Jenisys Systems Pvt ltd www.buy-ibpsmaster.com | www.ibpsexamguru.in

109

250. A, B and C walk 1 km in 5 min, 8 min and 10 min, respectively . C starts walking from a point at a
certain time, B starts from the same point 1 min later and A starts from the same point 2 min later than
C. Then, A meet B and C at times
1) 2 min, 3 min

2) 4/3 min, 3 min

3)2min, 5/3 min

4) 1 min, 2 min

5) None of these

Solution: (3).
A walks 1 km in 5 min.
Then, distance covered by B in 1 min
= 1000/5 = 200m
B walks 1 km in 8 min.
Then, distance covered by C in 1 min = 1000/8 = 125m
Let A meets B and C in X and Y min, respectively.
Then, according to the question, distance covered by C in ( x+2) min
= Distance covered by A in x min
= 100(x+2) = 200 x
= 100x + 200 = 200x
200 = 100x
Therefore x = 200/100 = 2 min
Now, for A and B,
Distance covered by B in (y+1) min
= Distance covered by A in y min
= 125 ( y+1) = 200 *y
= 125y 125 = 200 y
125 = 300y 125y
125 = 75y
Therefore y = 125/75 = 5/3 min

Jenisys Systems Pvt ltd www.buy-ibpsmaster.com | www.ibpsexamguru.in

110

(Permutations and Combinations)


251. In how many ways , a committee of 3 men and 2 women can be formed out of a total of 4 men and
4 women?
1) 15

2) 16

3) 20

4) 24

5) None of these

Solution: (4).
Total number of ways = 4C3 X 4C2
= [ 4! / (3! X 1!) ] X [ 4! / (2! X 2!) ]
= (4 x 4 x 3 x 2 x 1) / (2x2)
= 4 x 6 = 24

252. In how many ways,can 15 people be seated around two round tables with seating capacities of 7
and 8 people?
1) 15! / (8!)

2) 7!/88!

3) 15C8 x 6! X 7!

4) 15C8 * 8!

5) None of these

Solution: (3).
Number of ways in which 8 persons can be selected from 15 persons = 15C8
Now, 8 persons can be seated around a circular table in 7! Ways.
Now, remaining 7 persons can be seated around a circular table in 6! Ways.
Therefore required number of ways = 15C8 x 7! X 6!

253. There are 10 stations on a railway line. The number of different journey tickets that are required by
the authorities, is
1) 92

2) 90

3) 91

4) 93

5) None of these

Solution: (2).
There are 10 stations on railway line.
So, the number of different journey tickets between two stations from given 10 stations from one
Side = 10C2 = 10 x 9 / 2 = 45
Similarly , number of different journey tickets from other side = 45
Jenisys Systems Pvt ltd www.buy-ibpsmaster.com | www.ibpsexamguru.in

111

Therefore total number of tickets to be generated by authorities.


= 45 + 45 = 90
Direction (254 - 255) :Find the number of permutations that can be made from the letters of the work
OMEGA.

254. O and A occupying end places.


1) 12

2) 14

3) 20

4) 18

5) None of these

Solution: (1)
When O and A occupy end places. Then, the three letters ( M,E,G) can be arranged themselves by 3! = 6
ways and two letters ( O,A) can be arranged among themselves in 2! = 2 ways.
Therefore Total number of ways = 6 x 2 = 12

255. E being always in the middle.


1) 18 ways

2) 24 ways

3) 48 ways

4) 20 ways

5) None of these

Solution: (2).
When E is fixed in the middle, then there are four places left to be filled by four remaining letters O,M,G
and A and this can be done in 4! Ways.
Therefore total number of ways = 4! = 24

(Probability)
256. The probability that a man will be alive for 10 more years is and the probability that his wife will
alive for 10 more years is 1/3. The probability that none of them will be alive for 10 more years, is
1) 5/12

2) 1/2

3) 7/12

4) 11/12

5) None of these

Solutions : (2)
Required probability = P () x P (B)
= ( 1 ) x ( 1- 1/3 ) = x 2/3 =

Jenisys Systems Pvt ltd www.buy-ibpsmaster.com | www.ibpsexamguru.in

112

257. In a lottery 10000 tickets are sold and ten prizes are awarded. What is the probability of not getting
a prize, if you buy one ticket?
1) 9/10000

2) 9/10

3) 999/1000

4) 9999/10000

5) None of these

Solution: (3)
Total lottery tickets = 10000
Total prize in the lottery = 10
Therefore probability of getting a prize = 10/10000 = 1/1000
Now, probability of not getting a prize = 1 probability of getting a prize
= 1 1/1000 = 999/1000

258. Two persons A and B appear in an interview for two vacancies. If the probabilities of their
selections are and 1/6, respectively, then the probability that none of them is selected , is
1) 5/8

2) 5/12

3) 1/12

4) 1/24

5) None of these

Solutions : (1)
Required probability = = P () x P (B)
= ( 1- ) ( 1 - 1/6) = x 5/6 = 5/8

259. The probabilities of solving a problem by three students A, B and C are , 1/3 and , respectively.
The probability that the problem will be solved, is
1) 1/4

2) 1/2

3) 3/4

4) 1/3

5) None of these

Solutions : (3).
First, we find the probability of not solving the problem.
P (A) x P (B) x P (C)
= ( 1 ) x ( 1- 1/3) x ( 1 )
= x 2/3 x =
Therefore required probability = 1- =

Jenisys Systems Pvt ltd www.buy-ibpsmaster.com | www.ibpsexamguru.in

113

260. A dice is rolled three times and sum of three numbers appearing on the uppermost face is 15. The
chance that the first roll was four is
1) 2/5

2) 1/5

3) 1/6

4) None of these

5) Cannot be determined

Solutions : (4)
Total number of favourable outcomes n(S) = 63 = 216
Combinations of outcomes for getting sum of 15 on uppermost face = (4,5,6),
(5,4,6),(6,5,4),(5,6,4),(4,6,5),(6,4,5),(5,5,5),(6,6,3),(6,3,6),(3,6,6)
Now, outcomes on which first roll was a four, n(E) = (4,5,6),(4,6,5)
Therefore P(E) = n(E)/n(S) = 2/216 = 1/108

(Probability)
261. The probability that a man will be alive for 10 more years is and the probability that his wife will
alive for 10 more years is 1/3. The probability that none of them will be alive for 10 more years, is
1) 5/12

2) 1/2

3) 7/12

4) 11/12

5) None of these

Solutions : (2)
Required probability = P () x P (B)
= ( 1 ) x ( 1- 1/3 ) = x 2/3 =

262. In a lottery 10000 tickets are sold and ten prizes are awarded. What is the probability of not getting
a prize, if you buy one ticket?
1) 9/10000

2) 9/10

3) 999/1000

4) 9999/10000

5) None of these

Solution: (3)
Total lottery tickets = 10000
Total prize in the lottery = 10
Therefore probability of getting a prize = 10/10000 = 1/1000
Now, probability of not getting a prize = 1 probability of getting a prize
= 1 1/1000 = 999/1000
Jenisys Systems Pvt ltd www.buy-ibpsmaster.com | www.ibpsexamguru.in

114

263. Two persons A and B appear in an interview for two vacancies. If the probabilities of their
selections are and 1/6, respectively, then the probability that none of them is selected , is
1) 5/8

2) 5/12

3) 1/12

4) 1/24

5) None of these

Solutions : (1)
Required probability = = P () x P (B)
= ( 1- ) ( 1 - 1/6) = x 5/6 = 5/8

264. The probabilities of solving a problem by three students A, B and C are , 1/3 and , respectively.
The probability that the problem will be solved, is
1) 1/4

2) 1/2

3) 3/4 4) 1/3

5) None of these

Solutions : (3).
First, we find the probability of not solving the problem.
P (A) x P (B) x P (C)
= ( 1 ) x ( 1- 1/3) x ( 1 )
= x 2/3 x =
Therefore required probability = 1- =

265. A dice is rolled three times and sum of three numbers appearing on the uppermost face is 15. The
chance that the first roll was four is
1) 2/5

2) 1/5

3) 1/6

4) None of these

5) Cannot be determined

Solutions : (4)
Total number of favourable outcomes n(S) = 63 = 216
Combinations of outcomes for getting sum of 15 on uppermost face = (4,5,6),
(5,4,6),(6,5,4),(5,6,4),(4,6,5),(6,4,5),(5,5,5),(6,6,3),(6,3,6),(3,6,6)
Now, outcomes on which first roll was a four, n(E) = (4,5,6),(4,6,5)
Therefore P(E) = n(E)/n(S) = 2/216 = 1/108
Jenisys Systems Pvt ltd www.buy-ibpsmaster.com | www.ibpsexamguru.in

115

(Circular arrangement)
Direction (266 - 270): Eight persons A, B,C, D, E,F,G and H are sitting around a circular table facing
towards the centre , not necessarily in the same order. All of them like different colours viz. yellow ,
purple, orange , black, white ,green, red and blue . B is sitting second to the left of D. D like blue colour B
does not like yellow colour .D is an immediate neighbour of that two persons who likes red and purple
colours respectively. Three persons sit between B and the person who likes green colour. F and A
immediate neighbours E does not like black, yellow or purple colour .H is an immediate neighbour of G,
A is an immediate neighbour of both who likes red and blacks respectively. E is sitting just opposite to H
who likes orange colour.

266. How many persons are seated between H and D , if we go clockwise from H and D ?
1) Two

2) One

3) Four

4) Three

5) None

Ans : (3 ) Four
267. Which of the following pairs of persons represents the immediate neighbour of G?
1) H and the person who likes black colour
2) F and the person who like green colour
3) B and H
4) The person who likes green colour
5) None of these
Ans : ( 1 ) H and the person who like black
268. What is Es opposition with respect to the person who likes orange colour?
1) 3rd to the left

2) 5th to the left

3) 6th to the right

4) 4th to right 5) 2nd to the right

Ans .(4) 4th to the right

Jenisys Systems Pvt ltd www.buy-ibpsmaster.com | www.ibpsexamguru.in

116

269. Starting with A if all the persons are made to sit in an alphabetical order, the position of how many
persons ( excluding A ) will remain unchanged ?
1) None

2) One

3) Two

4) Three

5) More than three

Ans ( 2) One
270. Who among the following is seated exactly between the person who like red colour and F?
1) G

2) H

3) B

4) D

Ans : (5).A

(Quadrilateral)
271. The diagonal of a square is 4 2 cm. The diagonal of another square whose area is double that of the
first square is
1) 8 cm

2) 82 cm

3) 42

4) 6 cm

5) None of these

Solutions: (1).
Diagonal of a square = 2 a [ a = side]
42 = 2 a
a = 4 cm
Now, area of square = a2 = (4)2 = 16
Side of a square whose area is 2 x 16
a21 = 32
a1 = 32 = a1 = 4 2
Now, diagonal of new square = 2a = 2 x 4 2 = 8 cm.

272. The diagonal of two squares are in the ratio of 3:2. Find the ratio of their areas.
1) 9:4

2) 9:2

3) 9:5

4) 9:7

5) None of these

Solution: (1).
Let the diagonals of the squares be 3x and 2x.
Jenisys Systems Pvt ltd www.buy-ibpsmaster.com | www.ibpsexamguru.in

117

Therefore ratio of their areas = (3x)2 / (2x)2 = 9/4 = 9:4

273. The perimeter of two squares is 12 cm and 24 cm . The area of the bigger square is how many times
that of the smaller?
1) 2 times

2) 3 times

3) 4 times

4) 5 times

5) None of these

Solution: (3)
We know that,
Perimeter of square = 4 x side
Therefore 4 x a = 12 [for smaller square] a=3
Therefore area of smaller square = 3 x 3 = 9 cm2 .(i)
Now, 4 x b = 24
[for bigger square] b=6
therefore area of bigger square
= 6 x 6 = 36 cm2 = 4 x 9 cm2
= 4 x Area of smaller square [ from Eq.(i)]
Hence, area of bigger square is 4 times that of smaller square.

274. Diagonals of a rhombus are 1 m and 1.5 m in length. The area of the rhombs is
1) 0.75 m2

2) 1.5 m2

3) 1.5 m2

4) 0.375 m2

5) None of these

Solution: (1)
Area of rhombus = x d1 x d2
= x 1 x 1.5 = 0.75 m2

275. Find the distance between the two parallel sides of a trapezium if the area of the trapezium is 500
sq m and the two parallel sides are equal to 30 m and 20m, respectively.
1) 20 cm

2) 15 cm

3) 18 cm

4) 25 cm

5) None of these

Jenisys Systems Pvt ltd www.buy-ibpsmaster.com | www.ibpsexamguru.in

118

Solution: (1)
According to the question,

Area = (30+20) x h
= 50h = 500 x 2
Therefore h = 20 cm

(Circle)
276. A wheel makes 4000 revolutions in moving a distance of 44 km. Fund the radius of the wheel.
1) 15m

2) 27m

3) 25m

4) 28m

5) None of these

Solution: (5)
Distance covered in 1 revolution = (44 x 1000) / 4000 = 11m
According to the question,
2 r = 11 = 44/7 x r = 11
Therefore r = (11 x 7) / 44 = 1.75m

277. The area of a sector of a circle is 77 sq cm and the angle of the sector is 45 . Find the radius of the
circle.
1) 7cm

2) 14 cm

3) 21 cm

4) 28 cm

5) None of these

Solution: (2)
Let the radius of circle = r cm
According to the question,
Area of sector = 77 cm2 = /360 x r2 = 77
= 45/360 x r2 = 77
Jenisys Systems Pvt ltd www.buy-ibpsmaster.com | www.ibpsexamguru.in

119

= r2 = (77 x 7 x 8) / 22 = r = 14cm

278. The wheel of an engine turns 350 times round its axle to cover a distance of 1.76km. The diameter
of the wheel is
1) 3 cm

2) 33

3) 9 cm

4) 3 cm

5) None of these

Solution: (4)
Distance covered in 1 round
= Total Distance / Total round
= (1.76km x 1000) / 350
= 176/35 m
2 x 22/7 x r = 176/35
r =4/5m
Then Diameter = 8/5 m
= 1.6
Since None of these option not given , the nearest option is 3 because 3 = 1.7

279. The ratio of the areas of the circumcircle and the incircle of a square is
1) 2:1

2) 1:2

3) 2 :1

4) 1: 2

5) None of these

Solution: (1)
Ratio of the areas of the circumcircle and incircle of a square
= (Diagonal)2 / (Side)2 = (Side x 2 )2 / (side)2 = 2/1 or 2:1

280. A person observed that he required 30 s time to cross a circular ground along its diameter than to
cover it once along the boundary. If his speed was 30m/min,then the radius of the circular ground is
(take = 22/7)
1) 10.5 m

2) 3.5m

3) 5.5m

4) 7.5m

5) None of these

Solution: (2)
Jenisys Systems Pvt ltd www.buy-ibpsmaster.com | www.ibpsexamguru.in

120

Let the radius of circular field = r m.


Speed of person in m/s = 30/60 = m/s
According to the question,
2r /( ) 2r / () = 30 = 4r 4r = 30
= ( 4 x 22/7 4) r = 30
(125 4)r = 30 = (8.5)r = 30
= r = 30/8.5 = 3.5m

(Circle)

281. A wheel makes 4000 revolutions in moving a distance of 44 km. Fund the radius of the wheel.
1) 15m

2) 27m

3) 25m

4) 28m

5) None of these

Solution: (5)
Distance covered in 1 revolution = (44 x 1000) / 4000 = 11m
According to the question,
2 r = 11 = 44/7 x r = 11
Therefore r = (11 x 7) / 44 = 1.75m

282. The area of a sector of a circle is 77 sq cm and the angle of the sector is 45 . Find the radius of the
circle.
1) 7cm

2) 14 cm

3) 21 cm

4) 28 cm

5) None of these

Solution: (2)
Let the radius of circle = r cm
According to the question,
Area of sector = 77 cm2 = /360 x r2 = 77
= 45/360 x r2 = 77
Jenisys Systems Pvt ltd www.buy-ibpsmaster.com | www.ibpsexamguru.in

121

= r2 = (77 x 7 x 8) / 22 = r = 14cm

283. The wheel of an engine turns 350 times round its axle to cover a distance of 1.76km. The diameter
of the wheel is
1) 3 cm

2) 33

3) 9 cm

4) 3 cm

5) None of these

Solution: (4)
Distance covered in 1 round
= Total Distance / Total round
= (1.76km x 1000) / 350
= 176/35 m
2 x 22/7 x r = 176/35
r =4/5m
Then Diameter = 8/5 m = 1.6
Since None of these option not given , the nearest option is 3 because 3 = 1.7

284. The ratio of the areas of the circumcircle and the incircle of a square is
1) 2:1

2) 1:2

3) 2 :1

4) 1: 2

5) None of these

Solution: (1)
Ratio of the areas of the circumcircle and incircle of a square
= (Diagonal)2 / (Side)2 = (Side x 2 )2 / (side)2 = 2/1 or 2:1

285. A person observed that he required 30 s time to cross a circular ground along its diameter than to
cover it once along the boundary. If his speed was 30m/min,then the radius of the circular ground is
(take = 22/7)
1) 10.5 m

2) 3.5m

3) 5.5m

4) 7.5m

5) None of these

Solution: (2)
Let the radius of circular field = r m.
Jenisys Systems Pvt ltd www.buy-ibpsmaster.com | www.ibpsexamguru.in

122

Speed of person in m/s = 30/60 = m/s


According to the question,
2r /( ) 2r / () = 30 = 4r 4r = 30
= ( 4 x 22/7 4) r = 30
(125 4)r = 30 = (8.5)r = 30
= r = 30/8.5 = 3.5m

(Circle)
286. A wheel makes 4000 revolutions in moving a distance of 44 km. Fund the radius of the wheel.
1) 15m

2) 27m

3) 25m

4) 28m

5) None of these

Solution: (5)
Distance covered in 1 revolution = (44 x 1000) / 4000 = 11m
According to the question,
2 r = 11 = 44/7 x r = 11
Therefore r = (11 x 7) / 44 = 1.75m

287. The area of a sector of a circle is 77 sq cm and the angle of the sector is 45 . Find the radius of the
circle.
1) 7cm

2) 14 cm

3) 21 cm

4) 28 cm

5) None of these

Solution: (2)
Let the radius of circle = r cm
According to the question,
Area of sector = 77 cm2 = /360 x r2 = 77
= 45/360 x r2 = 77
= r2 = (77 x 7 x 8) / 22 = r = 14cm

Jenisys Systems Pvt ltd www.buy-ibpsmaster.com | www.ibpsexamguru.in

123

288. The wheel of an engine turns 350 times round its axle to cover a distance of 1.76km. The diameter
of the wheel is
1) 3 cm

2) 33

3) 9 cm

4) 3 cm

5) None of these

Solution: (4)
Distance covered in 1 round
= Total Distance / Total round
= (1.76km x 1000) / 350
= 176/35 m
2 x 22/7 x r = 176/35
r =4/5m
Then Diameter = 8/5 m
= 1.6
Since None of these option not given , the nearest option is 3 because 3 = 1.7

289. The ratio of the areas of the circumcircle and the incircle of a square is
1) 2:1

2) 1:2

3) 2 :1

4) 1: 2

5) None of these

Solution: (1)
Ratio of the areas of the circumcircle and incircle of a square
= (Diagonal)2 / (Side)2 = (Side x 2 )2 / (side)2 = 2/1 or 2:1

290. A person observed that he required 30 s time to cross a circular ground along its diameter than to
cover it once along the boundary. If his speed was 30m/min,then the radius of the circular ground is
(take = 22/7)
1) 10.5 m

2) 3.5m

3) 5.5m

4) 7.5m

5) None of these

Solution: (2)
Let the radius of circular field = r m.
Speed of person in m/s = 30/60 = m/s
Jenisys Systems Pvt ltd www.buy-ibpsmaster.com | www.ibpsexamguru.in

124

According to the question,


2r /( ) 2r / () = 30 = 4r 4r = 30
= ( 4 x 22/7 4) r = 30
(125 4)r = 30 = (8.5)r = 30
= r = 30/8.5 = 3.5m

(Spotting Error)
Noun : A word that represents name or place or animal or thing is called noun.
Subject : a person or thing that is being described or discussed or dealing with is called subject .
Verb : a word represent action or activity is called verb.
291. 1) If Ram win the match /(2) his brother is going /(3) to get him /(4) a new bike. /(5) No error.
Solution: (1) win , wins is correct.
Subject Verb Agreement rule : If the subject (he, she, it) is singular, we should add s to the
verb. (In present form).
Hint : Ram = singular , is going = present form then win becomes wins
292. 1) Shyam counted all the books /(2) to make sure that /(3) none of the books /(4) were left behind
in the shop. /(5) No error.
Solution: (4) were , was is correct.
None of the books ( None of + Noun) is always followed by a singular verb.

293. 1) ne of my classmate /(2) Mala helps me /(3) in preparing /(4) the notes. /(5) No error.
Solution: (1) classmate , classmates is correct.
One of is always followed by a plural noun ( one of + plural noun )
Some more examples:
Eg: Titanic is one of the romantic movies .
Eg: Treasury Island is one of the fictional stories .
Jenisys Systems Pvt ltd www.buy-ibpsmaster.com | www.ibpsexamguru.in

125

294. 1) Ram and Sita /(2) was with us /(3) when we /(4) went to Agra. /(5) No error.
Solution: (2) was , were is correct.
Here in this sentence the subject is Ram and Sita , they are two different persons connected with and
. So, it is plural noun. So, verb must be plural.
Hint : when " and " connecting two different noun then the noun is consider as plural

295. 1) My friend being unwilling to attend /(2) the court at an early hour /(3) of the morning, sent a
letter explaining /(4) why could he not obey /(5) No error.

Solution: (4) why could he not obey , why he could not obey is correct.
Subject Verb inversion occurs only in questions, but not in normal statements Inversion means reversal, transposition

(Miscellaneous)
296. What is the total area of three equilateral triangles inscribed in a semi-circle of radius 2 cm ?
1) 12 cm2

2) 3 3/4 cm2

3) 9 3/4 cm2

4) 33 cm2

5) None of these

Solution: (4)
Since, s AOB, BOC and COD are equilateral
Therefore sides = 2 cm
Now, total area = 3 x 3 /4 (side)2

= 3 x 3/4 x 4 = 3 3 cm2

Jenisys Systems Pvt ltd www.buy-ibpsmaster.com | www.ibpsexamguru.in

126

297. The area of a rectangle is 1.8 times the area of a square. The length of the rectangle is 5 times the
breadth. The side of the square is 20 cm. what is the perimeter of the rectangle?
1) 145 cm

2) 144 cm

3) 133 cm

4) 135 cm

5) None of these

Solution: (2).
Area of square = (side)2 = 202 = 400 sq.cm
Therefore area of rectangle = 1.8 x 400 = 720 sq.cm
Let length and breadth of rectangle be 5x and x, respectively.
Then, according to the question ,
5x * x = 720
5x2 = 720
= x2 = 720/5 = 144
X = 144 = 12 cm
Perimeter of rectangle = 2(5x + x) = 12x
12 x 12 = 144 cm

298. The side of a square is 5 cm which is 13cm less than the diameter of a circle. What is the
approximate area of the circle?
1) 245 sq cm

2) 235 sq cm

3) 265 sq cm

4) 255 sq cm

5) None of these

Solution: (4)
Diameter of the circle = 13 + 5 = 18cm
Therefore Radius = Diameter / 2 = 18/2 = 9cm
Area of the circle = r2 = 22/7 x 92
= (22 x 81)/8 = 1782/7 = 254.57 sq.cm
= 255 sq.cm (approx)

Jenisys Systems Pvt ltd www.buy-ibpsmaster.com | www.ibpsexamguru.in

127

299. Find the area of a square inscribed in a circle of radius 4 cm.


1) 32 sq cm

2) 18 sq cm

3) 64 sq cm

4) 25 sq cm

5) None of these

Solution: (1).
Here, r = 4 cm
Required area = 2r2 = 2 x 42 = 32 sq.cm
Since Diameter = Diagonal of square

Therefore of square = x (diagonal)2


= x 82 = 64/2 = 32 sq. cm

300. A rectangle of maximum area is drawn inside a circle of diameter 5 cm. what is the maximum area
of such a rectangle ?
1) 25cm2

2) 12.5 cm2

3) 12 cm2

4) None of these

5) Cannot be determined

Solution: (3)
ABCD be the rectangle inscribed in the circle of diameter 5 cm.

Therefore Diameter = Diagonal of rectangle Now, let x and y be the lengths and breadths of rectangle,
respectively. Now, in ABD
AB2 + AD2 = (5)2
= x2 + y2 = 25
Since, they form Pythagoras theorem. So, x = 4 and y = 3
Jenisys Systems Pvt ltd www.buy-ibpsmaster.com | www.ibpsexamguru.in

128

Therefore Area of rectangle = 3 x 4 = 12 cm2

Cube, Cuboid, Room or Box


301. The surface area of a cube is 726 sq cm. Find the volume of the cube.
1) 1331 cm3

2) 1232 cm3

3) 1626 cm3

4) 1836 cm3

5) None of these

Solution: (1)
According to the question,
6a2 = 726 [ a = edge of the cube]
=a2 = 726/6 = 121
Therefore a = 121 = 11 cm
Therefore required volume = a3 = 113 = 1331 cm3

302. The maximum length of a pencil that can be kept in a rectangular box of dimensions 8 cm X 6cm X
2cm, is
1) 213 cm

2) 214 cm

3) 226 cm

4) 102

5) None of these

Solution: (3)
Length of largest pencil that can be kept in a box
= Diagonal of box = l2 + b2 + h2
Where, I = 8 cm, b = 6 cm, h = 2 cm
= 64 + 36 + 4
= 104 = 2 26 cm

303. Internal length , breadth and height of a rectangular box are 10 cm, 8 cm and 6 cm, respectively.
How many boxes are needed which can be packed in a cube whose volume is 6240 cu.cm.?
1) 12

2) 13

3) 15

4) 17

5) None of these

Solution: (2)
Jenisys Systems Pvt ltd www.buy-ibpsmaster.com | www.ibpsexamguru.in

129

Volume of rectangular box = 10 x 8 x 6 = 480cm3


Volume of cubes = 6240 cm3
Therefore required boxes = Volume of cubes / Volume of rectangular box = 6240 / 480 = 13
Hence, 13 boxes are needed.

304. A cube has each edge 2 cm and a cuboid is 1 cm long, 2 cm wide and 3 cm high. The paint in a
certain container is sufficient to paint an area equal to 54 cm2. Which one of the following is correct?
1) Both cube and cuboid can be painted
2) Only cube can be painted
3) Only cuboid can be painted
4) Neither cube nor cuboid can be painted
5) None of these
Solution: (1)
Surface area of cube which can be painted = 6(side)2 = 6 (2)2 = 24 cm2
Now, surface area of cuboid which can be painted
= 2 ( lb + bh + 1h)
= 2 (2 + 6+ 3) = 22cm
Total surface area of both cube and cuboid
= 22 + 24 = 46 cm2 < 54 cm2
Therefore , both cube and cuboid can be painted.

305. The whole surface area of a rectangular block is 8788 sq.cm. If length, breadth and height are in the
ratio of 4:3:2, then find the length.
1) 26 cm

2) 52 cm

3) 104 cm

4) 13 cm

5) None of these

Solution: (2)
Let length, breadth and height be 4x, 3x and 2x, respectively .
Jenisys Systems Pvt ltd www.buy-ibpsmaster.com | www.ibpsexamguru.in

130

Whole surface area = 2 (lb + bh + lh)


= (lb + bh + lh) = 8788/2 = 4394
= ( 4x 3+3x2+2x4)x2 = 4394
26x2 = 4394
X2 = 169
X = 13
Therefore length = 4x = 4 x 13 = 52cm

(HCF and LCM)


306. In a fire range, 4 shooters are firing at their respective targets. The first, the second, the third and
the fourth shooter hit the target once in every 5 s, 6 s, 7 s and 8 s, respectively. If all of them hit their
targets at 9:am, when will they hit their targets together again?
1) 9:04 am

2) 9:08 am

3) 9:14 am

4) None of these

Solution: (3).
Time after which they will hit the target again together = LCM ( 5,6,7 and 8) = 840 seconds
LCM of (5,6,7,8) = 840 seconds
They will hit target together = 840/60 = 14 min
They will hit together again at 9:14 am.

307. Five bells begin to toll together at intervals of 9s, 6s, 4s,10s and 8s, respectively. How many times
will they toll together in the span of 1 h (excluding the toll at the start)?
1) 5

2) 8

3) 10

4) Couldnt be determined

5) None of these

Solution: (3)
The bells will toll together after time in seconds equal to the LCM of 9,6,4, 10 and 8.
LCM of 9, 6, 4, 10 and 8 is = 360 seconds
In one hour, the rings will toll together 3600/360 times = 10 times .

Jenisys Systems Pvt ltd www.buy-ibpsmaster.com | www.ibpsexamguru.in

131

308. A General can draw up his soldiers in the rows of 10, 15 or 18 soldiers and he can also draw them
up in the form of a solid square. Find the least number of soldiers with the General.
1) 100

2) 3600

3) 900

4) 90

5) None of these

Solution: (3).
LCM of 10, 15 and 18 is
LCM = 2 x 3 x 5 x 3 = 90
To make it perfect square, we multiply it with 2 x 5 = 10
Therefore required number of soldiers
90 x 10 = 900

309. Find the greatest possible length which can be used to measure exactly the lengths 7 m, 3 m 85 cm
and 12 m 95 cm.
1) 15 cm

2) 25 cm

3) 35cm

4) 42cm

5) 45cm

Solution: (3)
Required length = HCF of 7 m, 3 m 85 cm, 12 m 95 cm
= HCF of 700 cm, 385 cm, 1295 cm is 35 cm
Hence, required length is 35 cm.

310. Find the side of the largest possible square slabs which can be paved on the floor of a room 2 m 50
cm long and 1 m 50 cm broad. Also find the number of such slabs to pave the floor.
1) 25,20

2) 30,15

3) 50,15

4) 55,10

5) None of these

Solution: (3).
HCF of 250 cm and 150cm is 50 cm
HCF = 50
Therefore Number of slabs = Area of floor / Area of slab
= (250 x 150) / (50 x 50) = 15
(Word Problems Based on Numbers)
Jenisys Systems Pvt ltd www.buy-ibpsmaster.com | www.ibpsexamguru.in

132

311. A chocolate has 12 equal pieces. Manju gave th of it to Anju, 1/3 rd of it to Sujata and 1/6 th of it
to Fiza. The number of pieces of chocolate left with Manju is
1) 1

2) 2

3) 3

4) 4

5) None of these

Solution: (3).
Number of pieces in a chocolate = 12
Number of pieces Anju got = 12 x = 3
Number of pieces Sujata got = 12 x 1/3 = 4
Number of pieces Fiza got = 12 x 1/6 = 2
Therefore Number of pieces left with Manju = 12 ( 4+ 3+2) = 3

312. A student was asked to multiply a number by 3/2 but he divided that number by 3/2. His result was
therefore 10 less than the correct answer. Find the number.
1) 10

2) 12

3) 15

4) 20

5) None of these

Solution: (2).
Let the number be x.
According to the question,
x * 3/2 - [ x/ (3/2) ] = 10
(3x/2) (2x/3) = 10
(9x 4x) / 6 = 10
5x / 6 = 10
Therefore x = ( 10 * 6)/5 = 12

313. The sum of two positive numbers x and y is 2.5 times their difference . if the product of numbers is
84, then what is the sum of those two numbers ?
1) 26

2) 24

3) 22

4) 20

5) None of these

Solution: (4)
Jenisys Systems Pvt ltd www.buy-ibpsmaster.com | www.ibpsexamguru.in

133

Let the numbers be x and y.


According to the question,
(x+ y) = 2.5 ( x y)
= x + y = 2.5x 25y
= 3.5y = 1.5x
x/y = 7/3

.. (i)

Now , xy = 84
7/3 y * y = 84
= y2 = (84 x 3) /7
y2 = 36
Therefore y = 6
Therefore x = 7/3 x 6 = 14
Sum of numbers = x + y
= 14 + 6 = 20

314. X, Y and Z had taken a dinner together. The cost of the meal of Z was 20% more than that of Y and
the cost of the meal of X was 5/6 as much as the cost of the meal of Z. If Y paid Rs.100, then what was
the total amount that all the three of them had paid?
1) Rs.285

2) Rs.300

3) Rs.355

4) None of these

5) Cannot be determined

Solution: (4).
Given that,
The cost of meal of Y = Rs.100
Now, according to the question,
The cost of the meal of Z = 20% more than that of Y
= ( 100 + 20/100 *100) = ( 100 +20) = Rs.120 and the cost of the meal of X = 5/6 as much as the cost of
the meal of Z = 5/6 x 120 = Rs.100
Jenisys Systems Pvt ltd www.buy-ibpsmaster.com | www.ibpsexamguru.in

134

Therefore total amount that all the three of them has paid
= 100 + 120 + 100 = Rs.320

315. There are 200 questions in a 3 h examination. Among 200 questions, 50 are from Mathematics, 100
are from GK and 50 are from Science. Ram spent twice as much time on each Mathematics question as
for each other question. How many minutes did he spend on Mathematics questions?
1) 36

2) 72

3) 100

4) 60

5) None of these

Solution: (2).
Let Ram spends x min on each Mathematics question.
According to the question,
50x + 100 * x/2+ 50 * x/2 = 3 * 60
= x(50 + 50+25) = 180
x = 180/125
Therefore time = 20 * 180/125 = 72 min

(Partnership)
316. A and B started a business by investing Rs.35000 and Rs.20000, respectively. After 5 months B left
the business and C joined the business with a sum of Rs.15000. The profit earned at the end of year is
Rs.84125.What is the share of B in profit?
1) Rs.14133

2) Rs.15000

3) Rs.13460

4) Couldnt be determined

5) None of these

Solution: (3)
Ratio of equivalent capitals of A, B and C = (35000 x 12) : (20000 x 5) : (15000 x 7)
= 35 x 12 : 20 x 5 : 15 x 7
= 84 :20:21
Let As share = 84x
Bs share = 20x
Cs share = 21x
Jenisys Systems Pvt ltd www.buy-ibpsmaster.com | www.ibpsexamguru.in

135

According to the question,


84x + 20x +21x = 84125
= 125x = 84125
X = 84125/125 = 673
Therefore Bs share = 20x = 673 = Rs.13460

317. X and Y make a partnership. X invests Rs.8000 for 8 months and Y remains in the business for 4
months. Out of the total profit, Y claims 2/7 of the profit. How much money was contributed by Y ?
1) Rs.5000

2) Rs.5400

3) Rs.7400

4) 6400

5) None of these

Solution: (4)
Y gets 2/7 of the profit.
Therefore X gets ( 1 2/7) = 5/7 of the profit
Therefore X : Y = 5/7 : 2/7 = 5 : 2
Let the contribution of Y be a.
Then, 8000 x 8 : a x 4 = 5:2
= 64000/4a = 5/2
20a = 128000
Therefore a = 128000/20 = Rs.6400

318. A and B invest Rs.3000 and Rs.4000, respectively in a business. A receives Rs.10 per month out of
the profit as a remuneration for running the business and the rest of the profit is divided in proportion
to the investments .If in a year A totally receives Rs.390, what does B receive ?
1) Rs.630

2) Rs.360

3) Rs.480

4) Rs.380

5) None of these

Solution: (2).
Let the annual profit be Rs.x
Then, Rs .( x 120) will be distributed between A and B as their shares of profit .
Ratio of profits = Ratio of investments
Jenisys Systems Pvt ltd www.buy-ibpsmaster.com | www.ibpsexamguru.in

136

So, A : B = 3000 : 4000 = 3 : 4


Therefore As share = 120 + ( x 120) x 3/7
= 120 + ( x 120 ) x 3/7 = 390
= ( x 120) x 3/7 = 390 120 = 270
= x 120 = 270 x 7/3 = 630
Therefore Bs share = 4/7 * (x 120 )
= 4/7 x 630 = Rs.360

319. A began a business with Rs.2250 and was joined afterwards by B with Rs.2700. If the profits at the
end of the year were divided in the ratio of 2:1, after how much time B joined the business ?
1) 5 months

2) 6 months

3) 3 months

4) 7 months

5) None of these

Solution: (4)
Let B remained in the business for x months . Then,
A : B = ( 2250 x 12) : (2700 * x)
2:1 = ( 27000 : 2700x) = ( 10 : x )
Therefore 10/x = 2/1 [ since Ratio of profit is 2:1]
2x = 10
Therefore x = 10/2 = 5
Clearly, B joined after ( 12 5) = 7 months.

320. A and B entered a partnership investing Rs.16000 and Rs.12000, respectively. After 3 months A
takes out rs.5000, while B puts in Rs.5000 more. After 3 months more, C joins the business with a capital
of Rs.21000. After a year, they earned a profit of Rs.13200. By what value does the share of B exceeds
the share of C?
1) Rs.1600

2) Rs.1800

3) Rs.2100

4) Rs.2300

5) None of these

Solution: (2)
As share : B s share : Cs share
Jenisys Systems Pvt ltd www.buy-ibpsmaster.com | www.ibpsexamguru.in

137

= [ 16000 x 3 + ( 16000 5000) x 9 ] : [ 12000 x 3 + (12000 + 5000) x 9] : (21000 x 6)


= ( 16 x 3 + 11 x 9 ) : ( 12 x 3 + 17 x 9) : ( 21 x 6)
= 147 : 189 : 126 = 7 : 9 : 6
Hence, Bs share exceeds Cs share by
(13200 / 7 + 9 + 6 ) (9-6) = 13200 x (3 / 22) = Rs.1800
(Boats and Streams)

321. Speed of motorboat in still water is 45 km/h. if the motorboat travels 80 km along the stream in 1 h
20 min, then the time taken by it to cover the same distance against the stream will be
1) 4 h 20 min

2) 3 h 40 min

3) 2h 40 min

4) 2h 55 min

5) None of these

Solution: (3).
Let speed of stream be x km/h.
Given speed of motorboat in still water = 45 km/h
Therefore speed of boat along stream = ( 45 + x ) km/h
According to the question,
45 + x = ( 80 / 1 1/3 )
= 45 + x = (80 x 3) / 4
x = 60 45 = 15
Speed of boat against stream = 45 15 = 30 km/h
Hence , required time = Distance / Speed
= 80/30 = 8/3 X 60 = 160 min = 2 h 40 min

322. The ratio of speeds of a motorboat to that of the current of water is 36 : 5. The motorboat goes
along with the current in 5 h 10 min. Find the time to come back of motorboat.
1) 5 h 50 min

2) 6h

3) 6 h 50 min

4) 12 h 10 min

5) None of these

Solution: (3)
Jenisys Systems Pvt ltd www.buy-ibpsmaster.com | www.ibpsexamguru.in

138

Let speed of a motorboat be = 36 x km/h and speed of the current = 5 X km/h


Therefore speed downstream = ( 36 - 5 ) x = 41 * km/h and speed upstream = ( 36 5) x = 31xkm/h
Let distance be a km.
According to the question,
When boat goes along with the current .
Distance = Time X Speed
a = ( 5 + 10/60 ) * 41x
a=31/6 * 41x
Again, when boat come back a = Time X Speed
From Eq. (i),
31/6 * 41x = Time * 31x
Time = 41 / 6
Time = 6 h 50 min.

323. Ishwar is rowing a boat. He takes half time in moving a certain distance downstream than
upstream. What is the ratio of the rate of boat in still water to the rate of current?
1) 2 : 1

2) 5 : 1

3) 7 : 1

4) 3 : 1

5) None of these

Solution: (4)
Let speed of Ishwars boat in still water be x km/h and speed of current = y km/h
Rate downstream = ( x + y ) km/h
Rate upstream = ( x y ) km / h
Let the distance covered in each case be a.
According to the question,
2a / x+y = a/ x-y
= 2(x-y) = x+y
2x - 2y = x + y
Jenisys Systems Pvt ltd www.buy-ibpsmaster.com | www.ibpsexamguru.in

139

x = 3y
Therefore x/y = 3/1
= x : y = 3:1

324. A motorboat travelling at the some speed, can cover 25 km upstream and 39 km downstream in 8 h
. At the same speed, it can travel 35 km upstream and 52 km downstream in 11 h . The speed of the
stream is
1) 2 km/h

2) 3 km/h

3) 4 km/h

5) 5 km/h

5) None of these

Solution: (3)
Let the speed of a boat and stream be x and y km/h.
Therefore speed of boat along stream = ( x + y) km/h and speed of boat against stream = ( x y ) km/h
According to the question , (25/ x-y) + ( 39/ x+y ) = 8 ------------(i)
And ( 35 / x y) + ( 52 / x + y) = 11 .(ii)
On multiplying Eq. (i) by 4 and Eq. (ii) by 3 , then subtract Eq. (ii) from Eq.(i), we get
(100/ x y) + ( 156/x+y) = 32
(105 / x y) + (156 / x+y) = 33
-----------------------------------------( 100 / x-y) (105/x-y) = -1
= 5/ x-y = 1
x y = 5 .. (iii)
On substituting the value of ( x y = 5 ) in Eq.(i), we get
(25/5) + (39/x+y )= 8
(39/x+y) = 8 5
X +y = 39/3
x+y = 13 ..(iv)
On solving Eqs. (iii) and (iv), we get
Jenisys Systems Pvt ltd www.buy-ibpsmaster.com | www.ibpsexamguru.in

140

x=9 and y=4


Hence, speed of stream = 4 km/h.

325. A boat covers 24 km upstream and 36 km downstream in 6 h , while it covers 36 km upstream and
24 km downstream in 6 h . The speed of the current is
1) 1 km/h 2) 2km/h 3) 1.5 km/h 4) 2.5 km/h 5)None of these
Solution: (2)
Let speed of boat and current be x and y km/h.
Therefore speed of boat Upstream = ( x y ) km/h
And speed of boat downstream = ( x + y )km/h
Therefore (24 / x y) + ( 36 / x+y) = 6 (i)
And (36/ x- y ) + (24/ x+ y) = 13/2 (ii)
On solving Eqs.(i) and (ii), we get x = 10 and y = 2

(Time & Distance)

326. The ratio of speeds of a train and a car is 16:15, respectively and a bus covered a distance of 480
km in 8 h. The speed of the bus is th of the speed of train. What distance will be covered by car in 6 h?
1) 450 km

2) 480 km

3) 360 km

4) Couldnt be determined

5) None of these

Solution: (1)
Let speed of train = 16x
Speed of car = 15x
Speed of bus = 480/8 = 60 km/h
According to the question,
16x = 60 * 4/3 = 80
X=5
Jenisys Systems Pvt ltd www.buy-ibpsmaster.com | www.ibpsexamguru.in

141

Therefore speed of car = 15x = 15 * 5 = 75 km/h


Therefore required distance = Speed X Time
= 75 * 6 = 450 km

327. The ratio of the speeds of A and B is 3:4. A takes 20 min more than the time taken by B to reach a
particular place. Find the time taken by A and B, respectively to reach that place.
1) 40 min and 30 min 2) 80 min and 60 min 3) 90 min and 45 min 4) 90 min and 50 min 5) None of these
Solution: (2)
Ratio of speeds of A and B = 3 : 4
Therefore Ratio of time taken = 4 : 3
Let time taken by A and B be 4x and 3x, respectively.
Then, according to the question,
4x 3x = 20
X = 20
Hence, time taken by A = 4 * 20 = 80 min
and time taken by B = 3 * 20 = 60 min

328. A bus travels at the rate of 54 km/h without stoppages and it travels at 45 km/h with stoppages.
How many minutes does the bus stop on an average per hour?
1) 8 min

2) 10 min

3) 12 min

4) 4 min

5) None of these

Solution: (2)
Due to stoppages, bus covers 9 km less per hour.
Time taken to cover 9 km
= ( 9/54 * 60) = 10 min
Hence, the train stops on an average 10 min per hour.

Jenisys Systems Pvt ltd www.buy-ibpsmaster.com | www.ibpsexamguru.in

142

329. If you travel 39 km at a speed of 26 km/h, another 39 km at a speed of 39 km/h and again 39 km at
a speed of 52 km/h, what is your average speed for the entire journey?
1) 39 km/h

2) 37 km/h

3) 33.33 km/h

4) 36 km/h

5) None of these

Solution: (4)
Required average speed
= Total distance covered / Total time taken
= ( 3 * 39) / [(39/26) + (39/39) + (39/52) ]
= ( 3 x 13 ) / [ (1/2) + (1/3) +(1/4)]
= ( 3 * 13*12) /13 = 36 km/h

330. Two cars A and B start simultaneously from a certain place at the speed of 30 km/h and 45 km/h,
respectively. The car B reaches the destination 2 h earlier than A. What is the distance between the
starting point and destination?
1) 90 km

2) 180 km

3) 270 km

4) 360 km

5) None of these

Solution: (2)
Let the time taken by car B to reach destination is x h.
So, the time taken by car A to reach destination is ( x + 2)h.
Now, S1T1 = S2T2
= 30 * ( x+2) = 45 * x
30x + 60 = 45x
15x = 60
Therefore x = 4 h
Now, distance between starting point and destination = S2T2 = 45*4 = 180 km

(Mixture and Alligation)


331. A mixture of certain quantity of milk with 8 L of water is worth 45 paise per litre. If pure milk be
worth 54 paise per litre, how much milk is there in the mixture?
Jenisys Systems Pvt ltd www.buy-ibpsmaster.com | www.ibpsexamguru.in

143

1) 40 L

2) 35 L

3) 25 L

4) 45 L

5) None of these

Solution: (1)
As water is available free of cost, so its cost price = Rs. 0
According to the rule of allegation,

Water : Milk = 9 : 45 = 1 : 5
Therefore Quantity of milk = 5 x 8 = 40 L

332. A trader has 50 kg of pulses, part of which he sells at 8 % profit and rest at 18% profit. He gains 14%
on the whole. What is the quantity sold at 18% profit?
1) 30 kg

2) 35 kg

3) 40 kg

4) 60 kg

5) None of these

Solution: (1)
According to the rule of alligation.

Part I (8%) : Part II (18%) = 4 : 6 = 2 : 3


Therefore Quantity sold at 18% = 3/5 * 50 = 30kg

333. In two types of stainless steel, the ratio of chromium and steel are 2 : 11 and 5: 21, respectively . In
what proportion should the two types be mixed, so that the ratio of chromium to steel in the mixed type
become 7 : 32 ?

Jenisys Systems Pvt ltd www.buy-ibpsmaster.com | www.ibpsexamguru.in

144

1) 1:2

2) 1:3

3) 2:3

4) 3 :4

5) None of these

Solution: (1)
Quantity of chromium in first kind of steel = 2/13
Quantity of chromium in second kind of steel = 5/26
Quantity of chromium in both kind mixtures = 7/39
According to the rule of alligation ,

Therefore required ratio = 1/78 : 1/39 = 1:2

334. A container is filled with liquid, 6 part of which are water and 10 part milk. How much of the
mixture must be drawn off and replaced with water so that the mixture may be half water and half milk?
1) 1/3

2) 1/7

3) 1/5

4) 1/8

5) None of these

Solution: (3)
Let a L of liquid be compressing water.
Quantity of water in the new mixture
= ( 6 6a/16 + a ) L
Quantity of milk in the new mixture = ( 10 10a/16) L
According to the question,
6 6a/16 + a = 10 10a/16
= 96 6a + 16a = 160 10a
96 + 10 a = 160 10a
20a = 64

Jenisys Systems Pvt ltd www.buy-ibpsmaster.com | www.ibpsexamguru.in

145

a = 64/20 = 16/5
Therefore part of mixture replaced = 1/6 X 16/5 = 1/5

335. 60 kg of a certain variety of rice at Rs.32 per kg is mixed with 48 kg of another variety of rice and
the mixture is sold at the average price of Rs.28 per kg. If there be no profit or loss due to the new sale
price, then the price of the second variety of rice is
1) Rs.25.60 per kg

2) Rs. 25 per kg

3) Rs.23 per kg

4) Rs.30 per kg

5) None of these

Solution: (3)
Let price of second variety of rice be Rs.x per kg.

According to the question,


28 x /4 = 60/48
x = Rs.23

(Numbers)
336. A number divided by 56 gives 29 as remainder. If the same number is divided by 8, the remainder
will .
1) 4

2) 5

3) 6

4) 7

5) None of these

Solution: (2)
Let the number be x.
According to the question,
x=56k + 29
then,
x = ( 8 x 7k) + ( 8x3) + 5

Jenisys Systems Pvt ltd www.buy-ibpsmaster.com | www.ibpsexamguru.in

146

= 8 x ( 7k + 3 ) +5
Therefore, when x is divided by 8, the required remainder = 5

337. What is the remainder when 4 1000 is divisible by 7?


1) 1

2) 2

3) 4

4) None of these

5) Cannot be determined

Solution: (3)
Remainder of
41000 / 7 = (42)500/7 = (16)500 /7
= 2500 / 7 = 22 * (23)166 /7
= 4 * (8)166 / 7
=4

338. The remainder when 919 + 6 is divided by 8 is


1) 2

2) 3

3) 5

4) 7

5) None of these

Solution: (4)
Required remainder = 919 + 6
= (1)19 + 6 = 4
[since 8 = 9 1, so replaced by 1]
We have = (9)19 + 6 / 8
= ((8+1)19 + 6) / 8 = (119 + 6) / 8
= (1+6) / 8 = 7/8
Therefore remainder = 7

339. What will be the remainder when 19100 is divided by 20 ?


1) 19

2) 20

3) 3

4) 1

5) None of these

Jenisys Systems Pvt ltd www.buy-ibpsmaster.com | www.ibpsexamguru.in

147

Solution: (4)
We have = 19100 /20
= (20 -1)100 / 20
= (-1)100 / 20 = 1/20
Therefore remainder = 1
Required remainder = 19100
= (-1)100 = 1
[ since 20 = 19 + 1 so 19 replace by (-1)]

340. Every prime number of the form 3k + 1 can be represented in the form 6m+1 (where k, m are
integers), when
1) k is odd 2) k is even 3) k can be both odd and even 4) No such form is possible 5) None of these
Solution: (2)
Every prime number of the form 3k + 1 can be represented in the form 6m + 1 only , when k is even.
(HCF and LCM)

341. If three numbers are 2a, 5a and 7a , what will be their LCM ?
1) 70a

2) 65a

3) 75a

4) 70a3

5) None of these

Solution: (1)
Required LCM = a x 2 x 5 x 7 = 70a

342. If a number is exactly divisible by 11 and 13, which of the following types the number must be?
1)Divisible by ( 11+13)
2)Divisible by ( 13 11)
3)Divisible by (11 x 13)
4)Divisible by ( 13/11)
Jenisys Systems Pvt ltd www.buy-ibpsmaster.com | www.ibpsexamguru.in

148

5)None of the above


Solution: (3)
LCM of 11 and 13 will be ( 11 x 13). Hence, if a number is exactly divisible by 11 and 13, then the same
number must be exactly divisible by their LCM or by ( 11 x 13).

343. If the HCF of a and b are 12 and a, b are positive integers and a>b>12, then what will be the values
of a and b?
1) 12,24

2) 24,12

3) 24,36

4) 36,24

5) None of these

Solution: (4)
From option (4), we can say that the HCF of 36 and 24 is 12 and it is also satisfies the given condition
A > b > 12
Therefore a = 36 and b = 24

344. The sum of HCF and LCM of two numbers is 403 and their LCM is 12 times their HCF . If one number
is 93 , then find the another number.
1) 115

2) 122

3) 124

4) 138

5) None of these

Solution: (3)
Let LCM = m, HCF = n,
According to the question,
m = 12n, .(i)
and m+n = 403 (ii)
= 12n + n = 403 [ from Eq.(i)]
13n = 403
Therefore n = 403/13 = 31
Therefore m = 12 x 31 = 372
Let the another number = k
Therefore 93 x k = 372 x 31
Jenisys Systems Pvt ltd www.buy-ibpsmaster.com | www.ibpsexamguru.in

149

[ as product of two numbers = HCF x LCM ]


k = (372 x 31) / 93 = 124

345. The LCM of two numbers is 495 and their HCF is 5. If sum of the numbers is 100, then find the
difference of the numbers.
1) 10

2) 46

3) 70

4) 90

5) None of these

Solution: (1)
Given, LCM = 495, HCF = 5
Let 1st number = x, 2nd number = y
Therefore xy = 495 x 5
[ as product of two numbers = HCF x LCM]
Xy = 2475
We know that,
( x y)2 = ( x +y)2 4xy
= (100)2 4 x 2475
= 10000 9900 = 100
Therefore ( x y) = 100 = 10

(Mensuration)
346. Three sides of a triangular field are of length 15 m, 20 m and 25 m long, respectively. Find the cost
of sowing seeds in the field at the rate of Rs.5 per sq m.
1) 750

2) 150

3) 300

4) 600

5) None of these

Solution: (1)
Since, AC2 = AB2 + BC2

Jenisys Systems Pvt ltd www.buy-ibpsmaster.com | www.ibpsexamguru.in

150

(25)2 = (15)2 + (20)2


625 = 225 + 400
625 = 625
So, the triangular field is right angled at B.
Therefore Area of the field = X AB X BC
= X 15 X 20
= 150 cm2
So , the cost of sowing seed is Rs.5 per sq.m.
Therefore Cost of sowing seed for 150 m2 = 150 X 5 = Rs.750

347. The length and perimeter of a rectangle are in the ratio of 5 : 18.What will be the ratio of its length
and breath?
1) 4:3

2) 3 :5

3) 5:4

4) 4 :7

5) None of these

Solution: (3)
According to the question,
L / 2(l+b) = 5/18
10l + 10b = 18l = 8l = 10b
l/b = 10/8 = 5/4
Therefore l : b = 5:4
Hence, ratio of length and breadth of a rectangle is 5 : 4

Jenisys Systems Pvt ltd www.buy-ibpsmaster.com | www.ibpsexamguru.in

151

348. The length and the breadth of a rectangular plot are in the ratio of 5 : 3. The owner spends Rs.3000
for Surrounding it from all the sides at the rate of Rs.7.5 per metre. What is the difference between the
length and breadth of the plot?
1) 50m

2) 100 m

3) 75m

4) 60 m

5) None of these

Solution: (1)
Perimeter of the field = 3000/7.5 = 400m
22(l + b) = 400
2(5x + 3x ) = 400
8x = 200
Therefore x = 200/8 = 25
Therefore required difference = (5x 3x)
= 2x = 2 x 25 = 50m

349. If the area of a circle is equal to the area of square with side 2 units, what is the diameter of the
circle?
1) 1 unit

2) 2 units

3) 4 units

4) 8 units

5) None of these

Solution: (3)
Area of the circle = Area of the square = (side)2
Therefore r2 = (2 )2
r2 = 4 r2 = 4 / =4
r = 4 = 2 units
Therefore Diameter of circle (d) = 2 x r = 2 x 2 = 4 units

350. What is the area between a square of side 10 cm and two inverted semi-circular, cross-sections
each of radius 5 cm inscribed in the square?
1) 17.5 cm2

2) 18.5 cm2

3) 20.5 cm2

4) 21.5 cm2

5) None of these

Solution: (4)
Jenisys Systems Pvt ltd www.buy-ibpsmaster.com | www.ibpsexamguru.in

152

Area between square and semi-circles = Area of square 2 (Area of semi-circle)

= (10)2 2/2 X 22/7 (5)2


= 100 78.5 = 21.5 cm2
(Ratio)

351. A fraction can be simplified as 9/10 if 3 is added to the numerator. It can be simplified as 3/5 when
3 is subtracted from the numerator. Find the fraction.
1) 15/20

2) 5/20

3) 25/20

4) 35/20

5) None of these

Solution: (1) .9/10


18/20 = 15+3 / 20 3/5
12/20 = 15 3 / 20
The fraction is 15/20

352. The volume of a cup of water increases by 1/10 when it turns into ice. By how much does the
volume of ice decreases after it melts completely into water again? Express your answer in fraction.
1) 11/11

2) 1/11

3) 3/11

4) 4/11

5) None of these

Solution: (2)
Water 10 units
Ice 10 + 1 = 11 units
1 10/11 = 1/11

Jenisys Systems Pvt ltd www.buy-ibpsmaster.com | www.ibpsexamguru.in

153

353. Adam, Brain and Christopher were having a 200-m race. When Adam dashed through the finishing
line, Brian was 15 m behind him and Christopher was 20 m behind. How far was Christopher behind the
finishing line when Brian completed the race?
1) 3 15/37 m

2) 5 15/37 m

3) 2 15/37m

4) 4 15/37m

5) None of these

Solution: (2)
Suppose Brian ran the last 15 metres and Christopher ran n meters.
(200 15) : (200 20 ) = 15 : n
185/180 = 15/n
15/n = 37/36
37n = 540
n=540/37 = 14 22/37
20 14(22/37) = 5(15/37)
Christopher was 5 15/37 m behind the finishing line when Brian completed the race.

354. There are 35 white and black beads altogether in a box. 2/3 of the number of white beads equals to
of the number of black beads. How many white beads are in the box?
1) 5

2) 25

3) 35

4) 15

5) None of these

Solution: (4)

7u 35
1u 35 7 = 5
3u 3 x 5 = 15
15 white beads are in the box.

Jenisys Systems Pvt ltd www.buy-ibpsmaster.com | www.ibpsexamguru.in

154

355. Sean read 1/3 of the total number of pages of a book on Day 1. The ratio of the number of pages
read on Day 1 to Day 2 was 5 : 6 . Sean had to read 84 pages of the book in order to finish it. How thick
was the book?
1) 215

2) 315

3) 415

4) 515

5) None of these

Solution: (2)
1/3 x 6/5 = 6/15 = 2/5
He read 2/5 of the book on Day 2.
1 1/3 2/5 = 4/15
4u

84 pages

1u

21 pages

21 x 15 = 315 pages
The book had 315 pages.
(Mixed Topics)

356. Average of a and b is 45 and the average of b and c is 35 then a-c =


1) 20

2) 30

3) 25

4) 15

5) None of these

Solution: (1)
Average of b and c is 35
Therefore b+c / 2 = 35
b+c = 35 x 2 = 70
b = 70 c

.(1)

Average of a and b is 45
Therefore a+b / 2 = 45
a+b = 45 x 2 = 90
b = 90 a

..(2)

Equating (1) and (2),


Jenisys Systems Pvt ltd www.buy-ibpsmaster.com | www.ibpsexamguru.in

155

70 c = 90 a
Therefore a-c = 90 70 =20

357. If x : y = 2 : 1 then (x2 y2 ) : (x2 + y2 ) is


1) 3 : 5

2) 5 :3

3) 1 : 3

4) 3 :1

5) None of these

Solution: (a)
x :y = 2: 1
Therefore x = 2y
(x2 y2 ) : (x2 + y2)
Substituting x = 2y in the above ratio, we get
[ (2y)2 y2 ] : [ (2y)2 +y2 ]
= [ (4y2) y2 ] : [ ( 4y2) + y2 ]
3y2 : 5y2 = 3:5

358. If 1 is added to the denominator of a fraction, the fraction becomes and if 1 is added to the
numerator the fraction becomes 1. The fraction is
1) 4/7

2) 5/9

3) 2/3

4) 10/11

5) None of these

Solution: (3).
Let the fraction be x/y
If 1 is added to the denominator, the fraction becomes x/ y+1
x / ( y+1) =
2x = y+1
2x-y = 1 ..(1)
If 1 is added to the numerator, the fraction becomes (x+1) / y
(x + 1) / y = 1
x+1 = y
Jenisys Systems Pvt ltd www.buy-ibpsmaster.com | www.ibpsexamguru.in

156

x-y =-1 (2)


2x y = 1 (1)
x y = -1 .(2)
(-)(+)(+)
----------------------------Sub x = 2
----------------------------x=2
x-y = -1
(x=2)2-y = -1
-y = -3
Y=3
Therefore the fraction is 2/3

359. The product of two co prime numbers is 117. Their L.C.M. should be
1) 1

2) 117

3) 1/117

4) None of these

5) Cannot be determined

Solution: (2)
13 117 9
13 and 9 are co-primes because, they have no common divisor except 1 (or) they cannot divide one by
other. Hence, their L.C.M. should be 13 x 9 = 117

360. Seven men can complete a work in 12 days. They started the work and after 5 days, two men left.
In how many days will the work be completed by the remaining men?
1)5.8 days 2)6.8 days 3)9.8 days 4) 8 days 5)None of these
Solution: (3)
Seven men can complete a work in 12 days
Jenisys Systems Pvt ltd www.buy-ibpsmaster.com | www.ibpsexamguru.in

157

Therefore 1 day work of them = 1/12


Work done by them for 5 days = 5 x 1/12 = 5/12
Remaining work to be done = 1 5/12 = 7/12
No. of men
7
5

Part of the work


1
7/12

No. of days
12
?

7/5 x 7/12 x 12 = 49 /5 = 9.8 days


Therefore the work will be completed in 9.8 days.
(Mixed topics)

361. If a square and a rhombus stand on the same base, then the ratio of the area of the square and the
rhombus is
1) Greater than 1

2) equal to 1

3) equal to

4) equal to

5)None of these

Solution: (2).
The areas of the square and the rhombus standing on the same base must be equal.
Their ratio will be = 1/1 = 1

362. A sum of money doubles itself in 20 years in simple interest. Then the rate of interest per annum is
1) 5%

2) 4%

3) 5.5%

4) 4.5%

5) None of these

Solution: (1)
Let the principal be Rs.100
Amount in 20 years will be 2 times = Rs.200
Therefore S.I. = Rs.100
n = 20
Rate of interest r = 1001/Pn = (100 x 100) / (100 x 20)
r = 5%
Jenisys Systems Pvt ltd www.buy-ibpsmaster.com | www.ibpsexamguru.in

158

363. A sphere is placed inside a right circular hollow cylinder so as to touch the top, base and the lateral
surface of the cylinde. if the radius of the sphere is r, the volume of the cylinder is
1) 4r3

2) 8 /3 r3

3) 2r3

4) 8r3

5) None of these

Solution: (3)
Since the sphere touches the top, base and the lateral surface of the hollow cylinder, the radius of the
cylinder is r, the height of the cylinder is 2r.
Volume of the cylinder = r2 h
= r2 x (2r) = 2r3

364. 12 spheres of the same size are made from melting a solid cylinder of 16 cm. diameter and 2 cm
height then the diameter of each sphere is
1) square root of 3 cm

2) 2 cm

3) 3 cm

4) 4 cm

5) None of these

Solution: (4)
Diameter of the cylinder = 16 cm.
Therefore Radius = 8 cm.
Height of the cylinder = 2cm.
Volume of the cylinder = r2h
= x 8x 8 x 2 cm3
Volume of 12 spheres = Volume of the cylinder
12 x 4/3 x r3 = x 8 x 8 x2
r3 = ( x 8 x 8 x2 x3) / ( x 12 x 4)
r3 = 2 x 2 x 2
r= 2 2 2 = 2cm.
Therefore the diameter of the sphere = 2r = 4cm.

Jenisys Systems Pvt ltd www.buy-ibpsmaster.com | www.ibpsexamguru.in

159

365. A man performs 3/5 of the total journey by rail, 7/20 by bus and the remaining 6.5 km. on foot .His
total journey distance is
1) 65 km.

2) 100 km.

3) 120 km.

4) 130 km.

5) None of these

Solution: (4)
Distance travelled by rail = 3/5 part
Distance travelled by bus = 7/20 part
Total of them = 3/5 + 7/20 = (12 + 7) / 20 = 19/20 part
Therefore the distance travelled by the man on foot is
= 1 19/20 = (20-19)/20 = 1/20
For the part of 1/20 the distance travelled is 6.5 km.
Therefore the total distance travelled is
= 20/1 x 6.5 = 130 km.

(Mixed Topics)
366. Three numbers are in the ratio 3:4:5 and their LCM is 240. Then the HCF of these number is
1) 4

2) 8

3) 12

4) 20

5) None of these

Solution: (1)
The ratio of three numbers are = 3 : 4 : 5
Let the numbers be 3x, 4x and 5x
LCM of 3x, 4x, 5x = 60x
HCF = x
LCM = 60x = 240
Therefore HCF = x = 240 / 60 = 4

367. LCM of two numbers is 14 times their HCF . The sum of LCM and HCF is 600. If one number is 280
then the other number is
Jenisys Systems Pvt ltd www.buy-ibpsmaster.com | www.ibpsexamguru.in

160

1) 40

2) 60

3) 80

4) 100

5) None of these

Solution: (3)
Let the HCF be x, LCM = 14x
Sum of LCM and HCF = 600
x + 14x = 600
15x = 600
x = 600/15 = 40
Therefore HCF = 40
LCM = 14x = 560
One number is = 280
Therefore the other number
= (LCM x HCF) / One number = (560 x 40) / 280 = 80

368. If x+y = 12 and xy = 32 then 1/x + 1/y is


1) 1/8

2) 1/2

3) 1/4

4) 3/8

5) None of these

Solution: (4)
1/x + 1/y = (y+x)/xy = (x+y)/xy
(x+y = 12 and xy = 32)
(x+y) /xy = 12/32 = 3/8

369. If 2x + y = 2x - y = 16 then y is
1) 2

2) 4

3) 0

4) 1

5) None of these

Solution: (3)
2x+y = 2x-y = 16
2x+y = 2x-y = 24

Jenisys Systems Pvt ltd www.buy-ibpsmaster.com | www.ibpsexamguru.in

161

x+y = x-y
x-x+y+y =0
2y = 0 = y = 0
2x+y = 24
x+y = 4
x+0 = 4
x = 4, y = 0

370. If 8 -5 x 4 = 44 and 15 - 3 x 3 = 48 then, 16 4 x 5 =?


1) 0

2) 69

3) 20

4) 25

5) None of these

Solution: (2)
1) 8 5 x 4 = 44
2) 15 3 x 3 = 48
In the data given above, - is treated as x and x is treated as +
1)8 -5 x 4 = 44 (given)
It should be calculated as,
8 x 5 + 4 = 44 (Equation is satisfied)
2)15 -3 x 3 = 48
15 x 3 + 3 = 48 [Equation is satisfied]
Hence, 16 4 x 5 should be calculated as,
16 x 4 + 5 = 69.

(Mixed Topics)
371. The mean of 5 observation is 25, if one of the observation is excluded the means becomes 20. The
excluded number is
(1) 45

(2) 40

(3) 20

(4) 10

(5) None of these

Jenisys Systems Pvt ltd www.buy-ibpsmaster.com | www.ibpsexamguru.in

162

Solution: (1)
The mean of 5 observations = 25
Therefore the total of 5 observations ( 5x 25) = 125
The mean of 4 observation = 20
Therefore the total of 4 observation ( 4 x 20 ) = 80
Therefore the excluded number is ( 125 80 ) = 45

372. If P/Q = 1/3 then (27P 34Q) /(36P 3Q) is


(1) 14/3

(2) -14/3

(3) -25/9

(4) 25/9

(5) None of these

Solution: (3)
P/Q = 1/ 3 therefore Q = 3P
(27P 34Q) / (36P 3Q) = ( 9Q 34Q ) / (12Q 3Q) [ substituting Q = 3P ]
= -25Q / 9Q
= -25/9

373. If (3x + 2y) : (3x - 2y) = 5 : 2 then x : y is


(1) 5 : 2

(2) 14 : 9

(3) 9 : 14

(4) 2 : 5

(5) None of these

Solution: (2)
(3x + 2y ) : (3x 2y) = 5:2
2 (3x + 2y) = 5 (3x - 2y)
6x + 4y = 15x 10y
4y + 10y = 15x 6x
14y = 9x
14/9 = x/y
Therefore x : y = 14 : 9

Jenisys Systems Pvt ltd www.buy-ibpsmaster.com | www.ibpsexamguru.in

163

374. The number of small cubes with edge 10 cm. that can be accommodated in a cubical box of edge
1m. is.
(1) 10

(2) 100

(3) 1000

(4) 10000

(5) None of these

Solution: (3)
Volume of the cubical box a3 = 1m3
= ( 100 cm)3 = 1000000 cm3
[ since 1m = 100 cm ]
Volume of 1 cube = 103 = 1000 cm3
Therefore the number of small cubes that can be accommodated in the box
= 1000000/ 1000 = 1000

375. The diameter of a sphere is 6 cm. It is melted and drawn into a wire of diameter 2 mm. The length
of the wire is :
(1) 12 m

(2) 18m

(3) 36m

(4) 66m

(5) None of these

Solution: (3)
Diameter of a sphere = 6 cm.
Radius = 3 cm.
Diameter of the wire = 2 mm.
Radius = 1 mm.
Volume of a sphere = 4/3 R3
Volume of a cylinder = r2h
The length of the wire
= (4/3 R3) / r2 = ( 4/3 * * 33) * 12
= 4/3 * ( * 3 *3*3*1000) / * 1 * 1
1000 * 4 * 3 * 3 = 36000 mm = 36m.
Jenisys Systems Pvt ltd www.buy-ibpsmaster.com | www.ibpsexamguru.in

164

(Mixed Topics)

376. The sides of a triangle are in the ratio : 1/3 : and its perimeter is 104 cm. The length of the
longest side is
(1) 52 cm,

(2) 48 cm,

(3) 32 cm,

(4) 26 cm,

(5) None of these

Solution: (2)
The ratio of the sides of the triangle = : 1/3 : = ( 6:4:3) / 12 = 6: 4 : 3
Let the sides be 6x, 4x and 3x
Perimeter = 104 cm.
6x + 4x + 3x = 104
13x = 104
x = 104 / 13 = 8
Longest side is 6x = 6 * 8 = 48cm.

377. Find x in 4x * 642x = 16 x+5


(1) 1

(2) 5

(3) 4

(4) 2

(5) None of these

Solution: (4)
4x * 642x = 16 x+5
4x *(43)2x = (42)x+5
4x * 46x = 42x + 10
4x+6x = 42x + 10
47x = 42x + 10
7x = 2x + 10
7x 2x = 10
5x = 10
X = 10/5 = 2
Jenisys Systems Pvt ltd www.buy-ibpsmaster.com | www.ibpsexamguru.in

165

378. A ladder of 25ft. length reaches a window which is 24 ft above the ground level on one side of the
street. Keeping its foot at the same point the ladder is turned the other side of the street and now
reaches a window of 7 ft high. Then the width of the street.
(1) 30

(2) 32

(3) 29

(4) 31

(5) None of these

Solution: (4)

In the diagram, AC and AC are the positions of the ladder.


In the right angled ABC,
AC2 = AB2 + BC2
BC2 = AC2 AB2
BC =

=7

Similarly, in the right angled ABC


BC =
=
=

=
= 24

The width of the street (BC + BC)


= 7 + 24 = 31 ft.

379. _________is the binary equivalent of (12)10


(1) 00010010

(2) 1100

(3) 1101

(4) 1010

(5) None of these

Solution: (2)
Jenisys Systems Pvt ltd www.buy-ibpsmaster.com | www.ibpsexamguru.in

166

Therefore the binary equivalent of 12 10 = 1100

380. A horse and two cows together cost Rs.680. If a horse cost Rs.80 more than a cow then the ratio of
cost of horse and cow is
(1) 7:5

(2) 5:7

(3) 8:9

(4) 9:8

(5) None of these

Solution: (1)
Let the cost price of the cow be x
Therefore Cost price of two cows will be 2x
Therefore Cost price of horse will be (680 2x)
680 2x = x + 80
680 80 = x+2x
3x = 600
x = 600/3 = 200
Cost price of the cow is Rs.200
Therefore the cost price of the horse is (200 + 80) = Rs.280
The ratio of the cost of horse and cow
= 280 : 200 = 7:5

(Mixed Topics)
381. If 50% of ( x- y ) = 30% of ( x + y ) then what percent of x is y ?
1) 25%

2) 50%

3) 75%

4) 100%

5) None of these

Solution: (1)
50% of ( x y) = 30% of (x+y)
50/100 * (x - y) = 30/100 * ( x+y)
( x y) = 3/10 ( x+y)
2 * 3 (x+y) = 10 ( x-y)
Jenisys Systems Pvt ltd www.buy-ibpsmaster.com | www.ibpsexamguru.in

167

6x + 6y = 10x 10y
10y + 6y = 10x 6x
16y = 4x
x/y = 16/4 = 4/1
Therefore x : y = 4:1 = 100% : 25%
Therefore y is 25% of x

382. How many straight lines are there in the figure below?

1) 10

2) 12

3) 16

4) 8

5) None of these

Solution: (2)
If we separate the given figure like these figures, we get 4+4+2+2= 12 straight lines

383. Find the missing number in the diagram.


4
20

1) 25

5
9

10
80

2) 30

3) 96

8
18

12
?

8
20

4) None of these

5) Cannot be determined

Solution: (3)
Jenisys Systems Pvt ltd www.buy-ibpsmaster.com | www.ibpsexamguru.in

168

(1)In the first square, if we multiply the numbers in the first row such as (4 x 5) we get the product as 20
and if we add the numbers (4+5) we get the sum 9. And these numbers are given in the second row.
(2)In the second square also we get,
10 x 8 =80
10 + 8 = 18
10 and 8 are in the first row.
80 and 18 are in the second row.
(3) Similarly in the third square, we get
12 * 8 = 96
12 + 8 = 20
Hence, 96 is the answer.

384. The average salary of all workers in the factory Rs.60.The average salary of 12 officers is Rs.400. The
average salary of rest is Rs.56.Find the total no.of workers in the factory
1) 1116

2) 1032

3) 1212

4) 1132

5) None of these

Solution: (2)
The average salary of all workers in the factory = Rs.60
The average salary of 12 officers is Rs.400
Therefore the total salary of 12 officers ( 12 x 400) = rs.4800
Let the total number of workers be x
Therefore the total salary ( x*60) = 60x
The average salary of the rest = Rs.56
Therefore the total salary of the rest = ( x -12) *56 = 56 (x-12)
4800 + 56(x-12)=60x
4800 + 56x 672 = 60x
60x 56x = 4128
Jenisys Systems Pvt ltd www.buy-ibpsmaster.com | www.ibpsexamguru.in

169

4x = 4128
X = 4128/4 = 1032

385. If 7 spiders make 7 webs in 7 days, then 1 spider will make 1 web in how many days?
1) 1

2) 7/2

3) 7

4) 49

5) None of these

Solution: (3)
Spiders Webs
7
7
1
1

Days
7
?

7/1 * 1/7 *7 = 7 days


Therefore 1 spider will make 1 web in 7 days.
Data Sufficiency
(A) Statement (1) Alone is sufficient, but statement (2) alone is not sufficient.
(B) Statement (2) Alone is sufficient, but statement (1) alone is not sufficient
(C) Both statements together are sufficient, but neither statement alone is sufficient.
(D) Each statement alone is sufficient
(E) Statements (1) and (2) together are not sufficient to answer the question asked, and additional data
are needed.

386. If 90/k is an integer, is k an integer?


(i) k >1 (ii) k is a multiple of a prime number
Solution: (B)
Statement (1) is not sufficient . If k = 2, then the condition in the question is satisfied and k is an integer.
If k = 1.5, then the condition in the question is still satisfied but k is not an integer. Eliminate choices (A)
and (D). Statement (2) says that k is a multiple of a prime number, so k equals the product of two
integers and must therefore be an integer. This is sufficient. Thus, since only statement (2) is sufficient
alone, the answer is choice (B).

Jenisys Systems Pvt ltd www.buy-ibpsmaster.com | www.ibpsexamguru.in

170

387. How many different positive prime factors does integer x have?
(i) x/12 = 3 (ii) x/36 is an integer
Solution: (A).
Statement (1) states that x/12 = 3. Multiply both sides by 12 to get x = 36. Because the prime
factorization of 36 is 2 x 2 x 3 x3, this is sufficient to answer the question. Eliminate choices (B), (C), and
E).Statement (2) states that x/36 is an integer, so x has at least the prime factors of 36. Because it might
also have more, this is not sufficient. Thus, since statement (1) is sufficient alone, the answer is choice
(A).

388. Are positive numbers a and b integers?


(i) a/b is an integer (ii)ab is an integer
Solution: (E).
Statement (1) states that the quotient of the two numbers is an integer, but this does not imply that the
two numbers are integers. This information only implies that a is a multiple of b, so it is not sufficient
Eliminate choices (A) and (D). Statement (2) states that the product of the two numbers is an integer,
but this does not imply that the two numbers are integers. For example, if ab = 15, then one possible
solution is a = 3 and b = 5, but another is a = 2 and b = 7.5, so this is not sufficient . Eliminate choice (B).
Taking both statements(1) and (2) together does not give enough information to determine whether a
and b are integers. If a = 4 and b = 0.5 , then both statements are satisfied. However, the statements ar
still satisfied if a = 4 and b = 2. Thus, since both statements together are still not sufficient , the answer is
choice (E).

389. What is the range of Set S?


(i) The median of Set S is 12. (ii)The lowest term in Set S is the smallest prime number and the largest
term in Set S is equal to the square of the first term multiplied by 7.
Solution: (B) To solve for the range, it is necessary to know the smallest and largest terms in Set
S.Statement (1) provides the middle term, which by itself does not provide either the smallest or the
largest term. Eliminate choices (A) and (D). Statement (2) tells you that the lowest term is 2, and the
largest term is 28. The answer choice is (B).

390. If a, b, c and d are distinct even integers and a is prime, is the four digit number abcd divisible by 8 ?
(i)The two digit number ba is a multiple of 21, and 40 is a factor of the two digit number dc.
Jenisys Systems Pvt ltd www.buy-ibpsmaster.com | www.ibpsexamguru.in

171

(ii) 10 is a divisor of the two digit number bc and the product of a and d is a perfect square.
Solution: (D).
The question tells you the possible values for the variables are 0,2,4,6, or 8; the only prime even number
is 2, so a = 2. Statement (1) tells you ba is either 21,42,63 or 84; since you already know a is 2, then b
must be 4. The statement also tells you dc is either 40 or 80, and because you just figured out that b is 4,
d must be 8 and c is 0. The four digit number abcd is therefore 2,408, which gives an answer to the
Yes/No question (in this case, yes). Eliminate choices (B), (C), and (E). In statement (2), if 10 is a divisor
of bc, and you know from the question already that a is 2, then bc is 40,60, or 80 ; and since a is 2, the
only perfect square you can create with a and d is 16, so d is 8 (the number 4 is another perfect square,
but the question says a and d are distinct, so they cant both be 2 ). So the number abcd is either 2, 408
or 2, 608, and with a little bit of arithmetic you can determine that both possible values are divisible by 8
, so statement (2) provides another yes answer to the Yes/No question. The answer is choice (D).

Numbers
391. If a is an even integer, b is an integer and 2 is not a factor of a+b, which of the following could be
the value of b?
(1) 0

(2) 2

(3) 4

(4) 10

(5) 15

Solution: (5)
If 2 is not a factor of a+b, then a +b is odd. Because a is even , the only way to make the sum odd is to
make b odd. The numbers in choices (1), (2), (3) and (4) are all even. The answer is choice (5).

392. If a is an integer and b = 5a + 3 , which of the following can be a divisor of b ?


(1) 5

(2) 10

(3) 13

(4) 17

(5) 25

Solution: (3) Note that 5a is divisible by 5 but that 3 is not. Therefore, 5a + 3 cannot be divisible by 5 or
any multiple of 5.That eliminates choices (1),(2) and (5). Try Plugging in a few values for a. If a = 1, then b
= 8.However, none of the answers is a divisor of 8. If a = 2, then b = 13, which is divisible by 13.The
answer is choice (3).

393. If x and y are integers and x2 y2 is odd, which of the following must be odd ?
(1) x

(2) y

(3) x2

(4) x2 + 1

(5) x + y

Solution: (5)
Jenisys Systems Pvt ltd www.buy-ibpsmaster.com | www.ibpsexamguru.in

172

Try to find counterexamples for each answer choice. For choices (1) and (3), if x = 4 then y could
equal3.Since x does not have to be odd, choices(1) and (3)can be eliminated. For choice (2), if y =4, then
x could equal 5. Since y does not have to be odd, choice (2)can be eliminated .For choice (4), if x = 5,
then x2 + 1 = 26 and y could equal 3. Eliminate choice (4). So, choice (5) must be correct. Moreover, note
that x2 y2 = (x+y) (x-y) which means that both (x+y) and (x-y) must be odd. The answer is choice (5).

394. When m is divided by 5, the quotient is 2 and the remainder is b. When n is divided by 7, the
quotient is 2 and the remainder is also b. If mn = 221,what is the value of b ?
(1) 2

(2) 3

(3) 4

(4) 13

(5) 27

Solution: (2)
Since the remainder cannot be larger than the divisor, choices (4) and (5) can be eliminated. Since
choice (2) is the middle value of the remaining answers, start there. If b = 3, then m = ( 5x2) + 3 = 13 and
n=(7x2)+3 =17.Because mn = 13 x 17 = 221, b= 3. The answer is choice (2).

395. Which of the following is a multiple of 4! + 6?


(1) 2! + 3

(2) 4! + 12

(3) 5! + 6

(4) 5! + 30

(5) 8! + 12

Solution: (4)
A multiple of 4! + 6 can be factored into 4! + 6 and an integer. So, one way to determine a multiple is to
divide the answer choices by 4! + 6. For choice (4), 5!+30 / 4!+6 = 5(4!+6) / 4! +6) = 5. Be careful of
choice (5) : 4! X 2 8 ! The answer is choice (4).
Decimal Numbers
Direction (396 399):
(A)Statement (1) Alone is sufficient ,but statement (2) alone is not sufficient.
(B) Statement (2) Alone is sufficient, but statement (1)alone is not sufficient.
(C)Both statements Together are sufficient, but Neither statement alone is sufficient
(D)Each statement alone is sufficient
(E)Statement (1) and (2) Together are not sufficient to answer the question asked and additional data
are needed.

Jenisys Systems Pvt ltd www.buy-ibpsmaster.com | www.ibpsexamguru.in

173

396. What number is 0.03 of z ?


(1) 0.0008 is 0.02 of z.
(2) 0.01 of z is 0.0004
Solution: (D).
If you can solve for z , you can answer the question. Translate statement (1) to get 0.0008 = 0.02 x z and
you can solve for z. Eliminate choices (B), (C) and (E). Translate statement (2) to get 0.01 x z = 0.0004 and
again you can solve for z. Since both statement (1) and (2) are sufficient on their own , the correct
answer is choice (D).

397. If b = 0 . cdef where c, d, e and f represent nonzero digits, what is b?


(1) 1,000 x b is divisible by 4,938
(2) c>d>e>f and f = 6.
Solution: (B).
Statement (1) tells you that the number is divisible by 4,938, which leaves two possibilities; 4,938
or9,876, so there are two possible values for b. Statement (1) is not sufficient , so eliminate choices
(A)and (D). In statement (2) you get the inequality c>d>e>6, which tells you that e = 7, d = 8,and c=9 and
b=0.9876. Since statement (2) is sufficient , the correct answer is choice (B).

398. If k and / are positive and / is a single digit, what is the value of k?
(1) 1/3 0.01 = 0.3/ rounded to the nearest hundredth digit
(2) k = 3.927/
Solution: (C)
Statement (1) is not sufficient .From statement (1), you can find that l =2, but there is no information
regarding k. Eliminate choices (A) and (D). Statement (2) gives k as a decimal with a variable in it, so you
cannot find k from that alone. Eliminate choice (B).Combine both statements together and you have a
value for l and can find the value for k. The correct answer is choice (C).

399. X and Y are single digits in the decimal 7.381 x Y9. What is X ?
Jenisys Systems Pvt ltd www.buy-ibpsmaster.com | www.ibpsexamguru.in

174

(1) When the decimal is rounded to the nearest ten thousandth, the result is 7.3814.
(2) When the decimal is rounded to the nearest thousandth, the result is 7.381.
Solution: (E)
Statement (1) is not sufficient. From statement (1),you can find two possible values for X. X is 3 if Y is
5,6,7,8 or 9 and X is 4 if Y is 1,2,3 or 4. Eliminate choices (A) and (D). Statement (2) is not sufficient. From
statement (2), there are 4 possible values for X; 1,2,3 or 4. Eliminate choice (B). Combine both
statements together and X can still be 1,2,3 or 4. Since the statements together are also not sufficient,
the correct answer is choice (E).

400. When 3/22 is written in decimal form, which of the following is the 19th digit to the right of the
decimal point?
(1) 2

(2) 3

(3) 4

(4) 5

(5) 6

Solution: (E)
The decimal form of 3/22 is 0.1363636., so every even place to the right of the decimal point is 3, and
every odd place, after the first digit, is 6. The correct answer is choice (E).
(Numbers Factors)

401. If x and y are positive integers such that x is a factor of 10 and y is a factor of 12, all of the following
could be the value of xy EXCEPT
(A) 1

(B) 4

(C) 15

(D) 36

(E) 40

Solution: (D)
If x is a factor of 10, x could equal 1,2,5 and 10. If y is a factor of 12, y could equal 1,2,3,4,6 and 12.
Choice (A) is the result of multiplying 1 and 1. Choice (B) is the result of multiplying 1 and 4 or 2 and
2.Choice (C) is the result of multiplying 5 and 3 .Choice (E) is the result of multiplying 10 and 4. The
answer is choice (D).

402. How many distinct prime factors does 14! Have?


(A) 2

(B) 5

(C) 6

(D) 7

(E) 9

Solution: (C)

Jenisys Systems Pvt ltd www.buy-ibpsmaster.com | www.ibpsexamguru.in

175

14! equals 14 x 13 x 12 x 11 x 10 x 9 x 8 x 7x 6x 5x 4 x 3 x 2x 1. Its distinct prime factors are the prime


numbers from 1 to 14, which are 2,3,5,7,11 and 13. The answer is choice (C).

403. If the positive integers a and b are distinct factors of 9, then the smallest possible value for ab is
(A) 1

(B) 3

(C) 9

(D) 81

(E) 162

Solution (B)
The distinct factors of 9 are 1, 3 and 9. The smallest product of any two of these is 1 x 3 = 3. The answer
is choice (B).

404. If integer x has at least three distinct prime factors, then which of the following could be an
integer?
(A) 3/x

(B) 15/x

(C) 24/x

(D) 35/x

(E) 42/x

Solution: (E)
For a number divided by x to be an integer, that number must share the factors of x. Choice (A) has only
one prime factor, 3.Choice (B) has only two prime factors, 3 and 5. Choice (C) has only two distinct prime
factors, 2 and 3. Choice (D) has only two prime factors, 5 and 7. Choice (E) has three prime factors, 2,3
and 7. The answer is choice (E).

405. The product of distinct integers a,b and c is 1,001. IF a,b and c are each greater then 1, what is the
smallest value any one of the integers can have?
(A) 3

(B) 7

(C) 11

(D) 13

(E) 77

Solution: (B)
The prime factorization of 1,001 is 7 x 11 x 13 , so each integer a,b and c must equal one of these values.
The smallest value any of the integers can have, then is 7. The answer is choice (B).

(Numbers Fractions)
Direction (406 - 408):
(A) Statement (1) Alone is sufficient, but statement (2) alone is not sufficient
(B) Statement (2) Alone is sufficient, but statement (1) alone is not sufficient
Jenisys Systems Pvt ltd www.buy-ibpsmaster.com | www.ibpsexamguru.in

176

(C) Both Statements together are sufficient, but neither statement alone is sufficient.
(D) Each Statement alone is sufficient
(E) Statement (1) and (2) together are not sufficient to answer the question asked, and additional data
are needed

406. If Marvin saves x/10 of his income each month for savings, what is x ?
(1) Marvin saves 10% of his income
(2) Marvins income is Rs.2,500 a month.
Solution: (A)
A fraction is equivalent to part over whole, and a percentage is another way of giving a fraction.
Statement (1) gives you a percentage : 10/100, which can be reduced to 1/10. So x = 1. Statement (1) is
sufficient, so eliminate choices (B),(C) and (E).Statement (2) is not sufficient. It only provides income
without saying how much goes into savings. The answer is choice (A).

407. What is the value of x/3 + y/6 ?


(1) x/y = 2/3
(2) (y-x)/9 = 1/3
Solution: (C).
You can plug In for statement (1) and find different values of x and y to fit the equation, such as 2 and 3
or 4 and 6, that will yield different values for the question. Statement (1) is not sufficient , so eliminate
choices (A) and (D).The equation in statement (2) can be cross-multiplied to give 3y 3x = 9, and then
dividing both sides by 3 gives you y x = 3 , which by itself is not enough. Eliminate choice (B).
Combining both statements together yields simultaneous equations with two variables, two equations.
These can be solved for both x and y (but dont actually do the work), so the answer is choice (C).

408. If q and r are positive integers and qr = 64, what is q ?


(1) r/8 is an integer
(2) q/4 is an integer
Solution: (E)
Jenisys Systems Pvt ltd www.buy-ibpsmaster.com | www.ibpsexamguru.in

177

The possible pairs of factors for q and r according to the question are 1 and 64, 2 and 32, 4 and 16, and 8
and 8. Statement (1) tells you that r can be 8, 16, 32 or 64 and q could be 1,2,4 or 8. That isnt sufficient ,
so eliminate choices (A) and (D). Statement (2) tells you q can be 4,8,16,32 or 64 and r can be 16,8,4,2 or
1. That isnt sufficient, so eliminate choice (B). Both statements combined allow two possible pairs of
factors: 4 and 16, or 8 and 8 . The trap answer here is choice (C) if you assume the two variables are
distinct, which the question never states. The statements combined are still not sufficient, so the answer
is choice (E).

409. Each of the following could be a value of 2 / 10 x EXCEPT


(A) 1/10

(B) 1/5

(C) 0

(D) 2

(E) 10

Solution: (C)
Two divided by a number will never equal 0. Alternatively, you can try to set 2/ (10 x) equal to each
answer choice. The answer is choice (c)

410. At a dinner party, 2/3 of the diners ordered salad, and of those who ordered salad also ordered
soup. What fraction of diners at the party did not order both salad and soup?
(A) 1/6

(B) 1/4

(C) 1/3

(D) 3/4

(E) 5/6

Solution: (E)
If 2/3 ordered salad and, of those, ordered soup, then the fraction of diners who ordered both is 1/4 x
2/3 = 2/12 = 1/6. The fraction of diners who did not order both is 1 1/6 = 5/6. The answer is choice (E).

(Ratios)
Direction (411 412):
(A) Statement (1) Alone is sufficient, but statement (2) alone is not sufficient
(B) Statement (2) Alone is sufficient, but statement (1) alone is not sufficient
(C) Both statements together are sufficient, but neither statement alone is sufficient
(D) Each statement alone is sufficient
(E) Statements (1) and (2) together are not sufficient to answer the question asked, and additional data
are needed

Jenisys Systems Pvt ltd www.buy-ibpsmaster.com | www.ibpsexamguru.in

178

411. The ratio of x to y is less than 1. What is the ratio of x to y?


(1) 3/5 x = 9
(2) If one of the values is doubled, the new ratio is 3/2
Solution: (B)
The first statement provides information about the value of x, but not the value of y, so it is insufficient
to determine their ratio. Eliminate choices (A) and (D). Statement (2) seems ambiguous, but in order for
a ratio less than 1 to become greater than 1, the smaller value must be the one that is doubled. Thus,
statement (2) says 2x/y = 3/2, which can be solved for x/y to determine their ratio , .

412. A music store sells used and new CDs, at x dollars per used CD and y dollars per new CD. If Adrian
and Betty each made a purchase at the store, what is the value of x?
(1) Adrian paid $33.75 for 1 used CD and 1 new CD.
(2) Betty paid $24 for 3 used CDs.
Solution: (B)
Statement (1) is not sufficient to determine the price of a single used CD, since it is not possible no
separate the contribution of the used and new CDs to Adrians total. Eliminate choices (A) and (D).
Statement (2) is sufficient, because Betty bought only used CDs; if she bought 3 used CDs at x dollars
each, then her total must be 3x, so x can be found.

413. If a and b are positive integers such that the ratio of a to b is 5 to 3, which of the following could be
the remainder when a is divided by b?
(A) 3

(B) 5

(C) 6

(D) 9

(E) 15

Solution: (C)
If the ratio of a to b is 5 to 3, a could equal 5 when b equals 3, or a could equal 10 when b equal 6, or a
could equal 15 when b equals 9, and so on . When a is divided by b in any case, the remainder is going to
be an even number. The only even number in the answer choices is 6.

414. For all numbers x and y , the ratio x to y is equivalent to which ratio?

Jenisys Systems Pvt ltd www.buy-ibpsmaster.com | www.ibpsexamguru.in

179

(A) x to 2y

(B) 2x to 2y

(C) x+ 3 to y +3

(D) xy to xy

(E) 3x to x/3

Solution: B
Multiplying both values in a ratio by the same number yields an equivalent ratio. So, the ratio of x to y is
equal to the ratio of 2x to 2y.

415. A math professor maintains the same ratio of the number of students who pass to the number who
fail every semester. The numbers of students who passed the last four semesters, and the total numbers
for each, are as follows: 76 out of 80, 57 out of 60, 38 out of 40, and 95 out of 100 .If the professor
currently has 120 students, how many students are expected to fail this semester ?
(A) 6

(B) 7

(C) 8

(D) 112

(E) 114

Solution: (A)
The ratio of the number of students who pass to the number who fall remains the same every semester,
so use the numbers for one semester to determine the ratio. If 76 out of 80 students passed, then 4
students failed. The ratio of the number who pass to the number who fail is 76 to 4, or 19 to 1. So for
every 20 students, 19 students pass and 1 student fails. Apply this to the current semester; Let s be the
number of students who fail. Then s/120 = 1/20 and s equals 6.
(Equations and Inequalities)

416. If x and y are integers , and 1<x<7 and -3<y<4, what is one possible value of x/y ?
(A) -1/3

(B) -2/5

(C) 1

(D) 7/4

(E) 7

Solution: (C)
Using x = 2 and y = 2, for instance, gives x/y = 1.
Direction (417 - 420):
(A) Statement (1) Alone is sufficient, but statement (2) alone is not sufficient
(B) Statement (2) Alone is sufficient, but statement (1) alone is not sufficient
(C) Both statements together are sufficient, but neither statement alone is sufficient
(D) Each statement alone is sufficient
(E) Statements (1) and (2) together are not sufficient to answer the question asked, and additional
data are needed
Jenisys Systems Pvt ltd www.buy-ibpsmaster.com | www.ibpsexamguru.in

180

417. If 3a + 2b = 21, what is the value of b?


(1) b is a positive integer
(2) 3a = 5b
Solution: (B)
Statement (1) doesnt give enough information about b to find b . Eliminate choices (A) and (D).
Statement (2) is sufficient because, with manipulation, it allows you to put in 3/5 a for b and then
solve for b.

418. Is x>0 ?
(1) 6x > 1
(2) 5x2 45 = 0
Solution: (A)
Statement (1) states that 6x is greater than 1, so x must be positive. Note that x-y = 1/xy , so if x were
negative , 6x would be a fraction between 0 and 1. This is sufficient. Eliminate choices (B),(C), and
(E).Statement (2) states that 5x2 45 = 0. Add 45 to both sides of the equation to get 5x2 = 45, so x2 = 9
and x = 3 or x = -3 , and this is not sufficient. Thus, because only statement (1) is sufficient alone, the
answer is choice (A).

419. If r>s +t, is r positive?


(1) s>t
(2) r / s+t >1
Solution: (B)
Statement (1) gives no information about the values of s and t, and therefore gives no information about
r, so it is not sufficient. Eliminate choices (A) and (D) . Statement (2) states that r/s+t >1, so s+t must be
positive. If r>s+t and s+t<0, then r/s+t would be less than 1, because the direction of the inequality sign
switches when dividing or multiplying by a negative number. Because r>s+t, r must also be positive; so
this is sufficient. Thus, since only statement (2) is sufficient alone.

Jenisys Systems Pvt ltd www.buy-ibpsmaster.com | www.ibpsexamguru.in

181

420. If a> 0, is b <0?


(1) a/b > -6 (2) a + b > -2
Solution: (E).
When statements (1) and (2) are taken together, there is still not enough information to determine
whether b is negative. Either scenario, b<0 or b>0, can satisfy both statements. Thus, since both
statements together are still not sufficient, the answer is choice (E).

(Quadratic Equations)
421. ( 1/ 11 + 1/ 11 )2 =
(A) 1/121

(B) 1/44

(C) 4/11

(D) 1/22

(E) 4/ 11

Solution: (C)
Remember that ( 1/ 11 + 1/ 11 )2 is equivalent to ( 1/ 11 + 1/ 11 ) ( 1/ 11 + 1/ 11 ). Use FOIL to multiply
out the equation . The first two terms give ( 1/ 11 ) (1/ 11 ), or 1/11 . Multiplying the outer terms also
results in ( 1/ 11 ) (1/ 11 ) , or 1/11. Multiplying the inner terms results in ( 1/ 11 ) (1/ 11 ) , or 1/11, and
finally multiplying out the last terms results again in ( 1/ 11 ) (1/ 11 ) , or 1/11. Add up all the results, and
you have 1/11 + 1/11 + 1/11 + 1/11 = 4/11.

422. In the equation x2 + 4x c=30, 5 is one of the roots of the equation , and c is a constant . What is
the value of the other root?
(A) -9

(B) 5

(C) 4

(D) 9

(E) 15

Solution: (A).
First, solve for c by Plugging In 5 for x in the equation. This gives you 25+20 c = 30, so c must equal 15.
Now you can rewrite the equation as x2+4x-15 = 30.Now Plug in your answer choices, starting with
choice (C) to see which choice makes this equation true. 16+16-15 does not equal 30, so choice (C) is
incorrect. Choice (D) give 81 + 36 15, which still does not equal 30. Because choice (D) yielded such a
big number, try a negative number to get the equation closer to equaling 30. Choice (A) correctly gives
81-36-15 = 30.Because this is a Plugging in the answers question, stop when you find a number that
works. An alternative approach to this problem would be to solve for c as above, and then simplify the
equation x2+4x-15 = 30 by subtracting 30 from both sides, so that the equation becomes x2+4x-45
=0.Now factor the equation so that you have (x-5)(x+9). Thus the roots of the equation must be 5 and -9.
Note that choice (D) is a trap

Jenisys Systems Pvt ltd www.buy-ibpsmaster.com | www.ibpsexamguru.in

182

423. If r+15/ r+3 = r, what is the value of r2+2r 15 ?


(A) -5

(B) -3

(C) 0

(D) 3

(E) 5

Solution: (C)
Start with the first equation, which is r+15/r+3 = r. Multiply both sides by the denominator r+3 in order
to get rid of the fraction. Now the equation is r+15 = r(r+3), or r+15 = r2+3r.Now subtract r and 15 from
both sides of the equation so that you have r2+2r-15 = 0. The question asks for the value of the equation
r2+2r-15, so the answer is 0.

424. If 3 is one root of an equation, and the second root is a negative even number , which of the
following expressions might represent the equation ?
(A) x2 8x + 15

(B) x2 9x + 15

(C) x2 3x 15

(D) x2 + x 15

(E) x2 3x 15

Solution: (D).
The answer, when factored, should have positive 3 as one of the roots, and a negative even number as
the second root. When you factor choice (D) you have (x-3)(x+4), so one of the roots is positive 3, and
the other root is negative 4. Choice (D) is the answer. Factoring choice (E) gives (x+3)(x-6), so the roots
are negative 3 and positive 6. In this case, both roots have the wrong sign.

425. If x2 + bx + 26 = 8, what is the value of b?


(1) (x+9) is one of the factors of x2 + bx + 26 = 8
(2) -2 is one of the roots of x2 +bx + 26 = 8
(A) Statement (1) Alone is sufficient, but statement (2) alone is not sufficient.3
(B) Statement (2) Alone is sufficient, but statement (1) alone is not sufficient
(C) Both statements together are sufficient, but neither statement alone is sufficient.
(D) Each statement alone is sufficient
(E) Statements (1) and (2) together are not sufficient to answer the question asked, and additional data
are needed
Solution: (D)
Statement (1) is sufficient .If x2+bx+26 =8, then x2+bx+18=0. When you factor the equation, it will take
the form (x)(d)=0.The two numbers in the parentheses must multiply to 18 and add to b. Fact 1 tells you
Jenisys Systems Pvt ltd www.buy-ibpsmaster.com | www.ibpsexamguru.in

183

that (x+9) is one of the factors, so the other factor must be (x+2) because 2 x 9 = 18. The value of b
would then be 11. Eliminate choices (B), (C) and (E), statement (2) is also sufficient. Roots of quadratic
equations have the opposite sign of the numbers in the parentheses when you factor the equation. Thus
if -2 is a root of the equation x2+bx+18 = 0, then (x+2) must be one of the factors. From here you can
find the other factor (x+9) as you did above. Alternatively, since the root is just another name for the
solution of a quadratic equation, you could take -2, plug it into the equation for x, and solve for b.

(Numbers)
426. A filing cabinet has of drawers . Each drawer contains h hanging folders, and each hanging folder
holds 20 manila folders. How many manila folders are contained in 3 cabinets?
(A) 30d/h

(B) 30dh

(C) 60/dh

(D) 60dh

(E)60d/h

Solution: (D)
There are variables in the answers, so this is a good opportunity to Pilug, In . Lets say d = 4 and h = 5.
Then each cabinet has 4 drawers and each drawer has 5 hanging folders so thats total of 20 hanging
folders. Each hanging folder has 20 manila folders, so that gives you 400 manila folders in each cabinet.
There are 3 cabinets, so your target answer is 1200. Only choice (D) yields 1200 when you plug your
numbers into the answer choices, so the correct answer is choice (D).

427. Miriam sells homemade jewelry. If she sells 3/5 as many necklaces in May as she does in April, and
1/6 as many necklaces in June as she does in May, then the number of necklaces she sold in April was
how many times the average (arithmetic mean) number of necklaces she sold in May and June ?
(A) 7/20

(B) 10/7

(C) 5/3

(D) 20/7 (E)10

Solution: (D)
You dont know the number of necklaces for any of the months, so this is a good opportunity to Plug In.
Lets say the number of necklaces sold in April is 60. Then the number of necklaces sold in May is 36 and
the number sold in June is 6 . You can use an average pie to figure out that the average number of
necklaces sold in May and June is 21. 60 divided by 21 is 20/7, so the correct answer is choice (D).

428. Noah bought 30 horses at a total cost of g dollars. He later sold each of the horses at 25 percent
above the original cost per horse. If all the horses originally cost the same price, then , in terms of g, for
what dollar amount did each horse sell ?
(A) 30/g

(B) g/30

(C) 4g/ 120

(D) 75/g

(E) g/24

Jenisys Systems Pvt ltd www.buy-ibpsmaster.com | www.ibpsexamguru.in

184

Solution: (E)
The question says in terms of, so assume a value for G = 180 , then each horse was bought for 6
dollars and sold for 7.5 dollar so your target answer is 7.5. When you assume 180 into the answers , only
choice E gives you 7.5 so the correct answer is E.

429. At Lizs Bakery 2/7 of the cupcakes sold during August had chocolate frosting and of the other
cupcakes sold had sprinkles. If s cupcakes with sprinkles were sold in August, then , in terms of s ,how
many cupcakes with chocolate frosting were sold ?
(A) 5s/8

(B) s/25

(C) 8s/5

(D) 25/s

(E) 28s/5

Solution: (C)
Plug In, but rather than Plugging In for s, Plug In for the total number of cupcakes sold. Lets use 28 for
that total. That means 8 cupcakes had chocolate frosting, and of the remaining 20 cupcakes, 5 cupcakes
had sprinkles. Therefore s = 5 and your target answer is 8. Only choice (C) yields 8, so the correct answer
is choice (C).

430. If x and y are positive integers , is xy an odd integer?


(1) x/3 is an even integer
(2) x+y is an even integer
(A) Statement (1) Alone is sufficient, but statement (2) alone is not sufficient.
(B) Statement (2) Alone is sufficient, but statement (1) alone is not sufficient
(C) Both statements together are sufficient, but neither statement alone is sufficient.
(D) Each statement alone is sufficient
(E) Statements (1) and (2) together are not sufficient to answer the question asked, and additional data
are needed
Solution: (A)
Plug in for x according to the information in statement (1). If x = 6, then x/3 = 2. Now Plug In for y. If y =
3, then xy = 18, so xy is even . This yields a no answer. Plug in again , but choose an even number for y
this time . If x = 12 and y = 4 , then xy = 48, so xy is even .This also yields a no answer . In fact, if x/3 is
an even integer, then x must be even. The product of an even number and another number will always
be even , so statement (1) is sufficient.
Jenisys Systems Pvt ltd www.buy-ibpsmaster.com | www.ibpsexamguru.in

185

(Numbers)

431. At a baseball game, each player on the red team hit 2/3 as many balls as each player on the blue
team did. If the red team has 5/6 as many players as the blue team, then approximately what percent of
all the balls hit at the game were hit by the blue team ?
(A) 25%

(B) 56%

(C) 64%

(D) 66 2/3%

(E) 83%

Solution: (C)
In Lets say there are 6 players on the blue team and each player on the blue team hit 12 balls. That
means there are 5 players on the red team and each player on the red team hit 8 balls. Therefore, the
blue team hit a total of 72 balls and the red team hit a total of 40 balls for a grand total of 112 balls hit.
72 out of 112 balls is approximately 64% , so the correct answer is choice (C).

432. Issen is trying to select an assistant from a pool of a applicants. If 1/5 of them really want to be
actors, and of those 2/3 really want to be directors, then, in terms of a, how many of the applicants do
not want to be both actors and directors?
(A) 2/15a

(B) a/5

(C) a/3

(D) 7/8a

(E) 13/15a

Solution: (E).
Lets say a = 15. Then 3 applicants want to be actors, and of those, 2 also want to be directors. Therefore
the other 13 dont want to be both actors and directors. Thats your target answer. Only choice (E) yields
13 when 15 is plugged in for a, so the correct answer is choice (E).

433. At the opening night performance of a new play, 5 percent of the theatre patrons are late and are
seated immediately, but 30 percent of the patrons who are late are not seated until the end of the first
act. Approximately what percent of the theatre patrons that evening are late?
(A) 5%

(B) 7%

(C) 35%

(D) 70%

(E) 80%

Solution: (B)
This is a hard question so get rid of trap answer choices (A) and (C). Now, Plug In, but rather than
Plugging in for the total number of theatre patrons, Plug in 100 for the number of theatre patrons who
are late. That means 30 are late and arent seated until the end of the first act, and the other 70 are late
and seated immediately .Because 5 percent of the total patrons are late and seated immediately, you
can solve for the total number of patrons, which is 1,400. 100 out of 1,400 or approximately 7% of the
total patrons are late, so the correct answer is choice (B).
Jenisys Systems Pvt ltd www.buy-ibpsmaster.com | www.ibpsexamguru.in

186

434. Rebecca wants to climb a mountain, but she has to return to her camp at the base of the mountain
in r hours. If she climbs up at w miles per hour and climbs down at z miles per hour, how far can she
climb up so that she spends a total of r hours for the round trip ?
(A) w+z+r/wz

(B) rwz/w+z

(C) wr/z

(D) z+r/w - r/z

(E) r+w/wz

Solution: (B)
This ones a little tricky. Lets begin by Plugging In w = 2 and z = 4. Now lets Plug In a one-way distance
up the mountain of 12 miles. With those numbers it will take Rebecca 6 hours to climb up the mountain
and 3 hours to climb down for a total of 9 hours. Thus r= 9 and the target is 12. Only choice (B) yields 12
when you plug your numbers into the answers.

435. The amount of money Michael makes in a year is represented by m. The amount of money Gary
makes in a year is represented by g. If Michaels yearly salary is p percent of Garys yearly salary, then, in
terms of p, Garys yearly salary is what percent of Michaels yearly salary ?
(A) 10/p

(B) p/100

(C) 100p

(D) 10,000/p

(E) p

Solution: (D)
Plug In , starting with Garys salary. Lets say g = 200 and p =50. That makes m = 100. 200 is 200% of 100,
so your target answer is 200. Only choice (D) yields 200 when you Plug In 50for p, so the correct answer
is choice (D).

(Mixed Topics)
436. At a particular zoo, 2/5 of all the animals are mammals, and 2/3 of the mammals are allowed to
interact directly with the public. If 24 mammals are allowed to interact directly with the public, how
many animals in this zoo are NOT mammals?
(A) 36

(B) 48

(C) 54

(D) 60

(E) 72

Solution: (C)
This problem is a bit tricky because at first it looks like a hidden Plug In. The question does not tell you
the number of animals in the zoo and gives you a bunch of fractions. But in fact, this is a Plug in the
answer problem, because the question asks for the total number of non-mammals in the zoo and the
answer choices are real numbers, not fractions or percents. Start by Plugging in choice (C ), 54, for the
number of non mammals. Now, use the information in the problem to find the number of mammals.
According to the problem, 24 mammals are allowed to interact with the public, and this is 2/3 of all the
Jenisys Systems Pvt ltd www.buy-ibpsmaster.com | www.ibpsexamguru.in

187

mammals. Thus, there must be 36 total mammals in the zoo (because 24 is 2/3 of 36). If there are 36
mammals and 54 non mammals, then there are 90 animals in the zoo. Now, check this number against
the information in the problem. The problem says that 2/5 of all the animals are mammals and 36 is 36
is 2/5 of 90.

437. Reservoir A contains 450 million gallons of water more than does Reservoir B . If 100 million gallons
of water were drained from Reservoir A into Reservoir B, then Reservoir A would contain twice as much
water as would Reservoir B. How many million gallons of water does Reservoir A currently contain?
(A) 500

(B) 600

(C) 700

(D) 800

(E) 900

Solution: (B)
If Reservoir A contains 600 million gallons of water, then Reservoir B has 450 million gallons less, or 150
million gallons. When 100 million gallons are drained from Reservoir A to Reservoir B , then the
reservoirs will hold, respectively, 500 million and 250 million gallons of water. Thats the relationship
youre looking for Reservoir A has twice as much water as Reservoir B so the answer is choice (B).

438. A sports league encourages collaboration by awarding 3 points for each goal scored without
assistance and 5 points for each goal scored with assistance .A total of 48 points were scored by a team
in a single game. Which of the following CANNOT be the number of goals scored without assistance by
this team in this game?
(A) 1

(B) 6

(C) 11

(D) 12

(E) 16

Solution: ( D)
If 12 goals for 3 points are scored, then 36 points have been scored and there are 12 points remaining.
This is not divisible by 5, so choice (D) does not work.

439. Benji is booking an airline flight, and the airline is running a special . Benji can choose either to
receive 15,000 air miles immediately, or to receive 7,000 air miles, plus 20 percent of the miles that he
flies over the next six months. How many miles must Benji fly in order to receive equal miles from either
option?
(A) 11,000

(B) 15,000

(C) 22,000

(D) 40,000

(E) 80,000

Solution: (D)
Plug in the answers, starting with choice ( C). If Benji purchase 22,000 miles, then he receives 4,400
miles plus the original 7,000 miles, so the total is 11, 400 miles. This number is too small, so go to choice
Jenisys Systems Pvt ltd www.buy-ibpsmaster.com | www.ibpsexamguru.in

188

(D). If Benji purchase 40,000 miles, then he will receive 8,000 miles, plus the original 7,000 miles. The
total is 15000, so the answer is choice (D).

440. At his weekly poker game, Ron bought $200 worth of chipls in $5 and $10 denominations. By the
end of the evening , Ron had lost all but 12 of his chips. If the number of $10 chips Ron had left was two
more or two less than the number of $5 chips he had left, what is the minimum possible value of the
chips that Ron lost?
(A) $140

(B) $105

(C) $95

(D) $85

(E) $80

Solution: (B)
The question says that Ron lost all but 12 chips and the number of $10 chips Ron had left was two more
or two less than the number of $5 chips he had left. So Ron either had five $5 and seven $10 chips left,
totaling $95, or seven $5 and five $10 chips left , totaling $85. For choice (B) , Ron has $95 worth of
chips left. That matches , so the answer is choice (B).

(Numbers)
441. If x is a positive integer and is a multiple of both 3 and 11, then x must also be a multiple of which
of the following ?
I.14 II.33 III.66
(A) I only

(B) II only

(C) I and II

(D) II and III

(E) None of the above

Solution: (B)
Plug in a number for x that satisfies the conditions of the problem. If x = 66, then II and III work, but I
does not. Eliminate choice (A). Then the answer is choice (B). Alternatively, you can solve this by using
the properties of numbers. Both 3 and 11 are prime numbers, so any number that is a factor of both
must also be divisible by (3)(11), or 33.

442. For all real numbers w, which of the following must be equal to 0?
I. w0 II. III. 1/w + - 1/w
(A) I,II and III

(B) II and III only

(C) I and II only

(D) III only

(E) I only

Solution: (D)

Jenisys Systems Pvt ltd www.buy-ibpsmaster.com | www.ibpsexamguru.in

189

This is a must be true question , so you may have to Plug In more than once. If w = 2, statement I is not
true because 20 = 1. In fact, anything to the zero power equals 1 . Get rid of all the answers that have I in
them , so eliminate choices, (A), (C) and (E). That leaves choices (B) and (D). Both have III in them , so
theres no need to check III. For statement II, 2 does not equal 0, so eliminate choice (B).

443. If x = - 0.5, which of the following must be true?


I. x < x2 II. x3 < x III. X3 < x2
(A) I only

(B) I and III

(C) II and III

(D) I,II and III

(E) None of the above

Solution: B
If x = -0.5, then x<x2 can be written as 0.5 < 0.25, which is true. Eliminate choices and (E). The
expression x3 < x can now be written as 0.125 < -0.5. This is false, so eliminate choice (D). The
expression x3 < x2 becomes - 0.125 < 0.25 . This is true, so the answer is choice (B).

444. If positive integer r is divisible by both 6 and 8, then r must also be divisible by which of the
following?
I.48 II.15 III.96
(A) None

(B) I only

(C) II only

(D) I and III

(E) I, II and III

Solution: (A)
This is a must be question, so Plug In. If r = 48, statement I is true, but you have to Plug In again to
make sure its always true. Statement II is not true so eliminate choices and (E). Statement III is not
true, so eliminate choice (D). At this point your choices are I only and None. Is there a number you
can Plug In that is divisible by both 6 and 8 , but not divisible by 48? Yes. 24. Thus, you can eliminate
choice (B) and the answer is choice (A).

445. If both x and y are positive integers, and 8 is a factor of x, and 9 is factor of y, which of the following
must be a factor of xy?
I.12 II.16 III.24
(A) I only

(B) II only

(C) I and II

(D) I and III

(E) I, II and III

Solution: (D).

Jenisys Systems Pvt ltd www.buy-ibpsmaster.com | www.ibpsexamguru.in

190

Plug in for x and y. If x = 8, and y = 9 , then xy = 72.12 is a factor of 72, so keep choices (A), (C), (D) and
(E) for now. 16 is not a factor of 72, so eliminate choices (B), (C) and (E). 24 is a factor of 72, so keep
choice (D). Plug in again to confirm that both 12 and 24 work for all possible values of xy. If x = 16 , and y
=18, then xy=288. Both 12 and 24 are factors of 288, so choice (D) is the answer. Alternatively , you can
use the properties of numbers to solve this problem. If x is a multiple of 8, and y is a multiple of 9, then
xy must be a multiple of 72 and any of its factors.

(Numbers)
446. If 2r 3s = 0 and s<2, which of the following must be true?
(A) r = 3

(B) r>3

(C) r<2

(D) r> -3

(E)r<3

Solution: (E)
This is a must be true question , so Plug In. First, though, solve the given equation for r because all the
answer choices refer to r. That gives you r = 3/2 s. Now Plug In for s. If s = 1, then r = 3/2 . Eliminate
choices (A) and (B). Plug in again ; if s = -2, then r = -3 ; eliminate choice (D).Plug In again : if s = 3/2, then
r = 9/4 = 2 . Eliminate choice (C).

447. If p and r are distinct, odd integers , and p<r, then which of the following must be true?
I.p+r is even II.pr is positive III. p/r <1
(A) I only

(B) II only

(C) I and II

(D) II and III

(E) I,II and III

Solution (A).
Plug In values for p and r . If p = -5, and r = -3 , then p+r is -8 . Because -8 is even , I works for now. Try II
with those numbers : ( -5) x (-3) = 15, and 15 is positive , so II works for now. Try III with those numbers :
-5/-3 = 5/3 , which is greater than 1, so eliminate choices (D) and (E). Now try another set of values for r
and p . If p = -3, and r = 5 , then p+r = 2, so I still works. (-3) x 5 = -15. Because the result is a negative
number , you can eliminate choices (B) and (C). The only choice left, and therefore the answer, is choice
(A).

448. Olivia has twice as many marbles as Josh and Josh has 3 fewer marbles than Hugo. If Olivia , Josh
and Hugo all receive 4 more marbles, which of the following must be true ?
I. Olivia has more marbles than Hugo.
II. Olivia has twice as many marbles as Josh.
Jenisys Systems Pvt ltd www.buy-ibpsmaster.com | www.ibpsexamguru.in

191

III. Josh has 3 fewer marbles than Hugo.


(A) I only

(B) III only

(C) I and III

(D) II and III

(E) I,II and III

Solution: (B)
This is a must be true question so you may need to Plug In more than once. Say that Olivia has 10
marbles. That means Josh has 5 and Hugo has 8. If they each get 4 more marbles, Olivia will have 14,
Josh will have 9 and Hugo will have 12. Looking at statement I, Olivia does have more marbles than
Hugo. For statement II, Olivia does not have twice as many marbles as Josh, so cross off answers that
include II. Eliminate choices (D) and (E). For statement III Josh does have 3 fewer marbles than Hugo.
Now Plug In with smaller numbers. If Olivia has 2 marbles, Josh has 1 and Hugo has 4. If they each get 4
more marbles, then Olivia has 6, Josh has 5 and Hugo has 8. Statement I is no longer true. Eliminate
choices (A) and (C). That leaves III only, so the answer is choice (B).

449. If 0<x<y<1, then which of the following must be true?


I.x-2 >y2 II.xy>1 III.x2<y3
(A) I only

(B) II only

(C) I and III

(D) II and III

(E) None of the above

Solution: (A)
Plug In for x and y . Let x = 1/3 and y = . Now the expression x-2 > y2 becomes (1/3)-2 > (1/2)2,which
simplifies to 9> . That is a true statement , so I works for now . Try II with those numbers: xy > 1 is now
(1/3) x (1/2) >1. (1/3) x (1/2) = 1/6, and 1/6 is not greater than 1 , so eliminate choices (B) and (D) Try III
with those numbers :x2 < y3 now becomes (1/3)2 < (1/2)3, or 1/9 <1/8 . This is a true statement , so III
works for now. Plug In again to see if both I and III are true in all circumstances. Let x = 1/5 and y = .
Now x-2 >y2 becomes (1/5)-2 > (1/4)2. This simplifies to 25> 1/16. This is true . For III , the expression x2
<y3 becomes (1/5)2<(1/4)3 , or 1/25 < 1/64 . This expression is not true, so eliminate choice (D).

450. If 0 <p<1, which of the following must be true?


I.p4-p5<p2-p3 II.p5<p3 III.p4 +p5 <p3+p2
(A) I, II and III

(B) I and II only

(C) II only

(D) I only

(E) None

Solution: (A).
All the three statements will work. When a fraction is raised to a positive power greater than 1, the
result is a number that is smaller than the original fraction. Thus, all three statements will always work
and the answer is choice (A).

Jenisys Systems Pvt ltd www.buy-ibpsmaster.com | www.ibpsexamguru.in

192

(Numbers)
451. If 5x + 3y = 0, and x<3 , then which of the following must be true?
(A) y < -5

(B) y = -5

(C) y > - 5

(D) y>3

(E) y>5

Solution: (C)
Plug in values that fit both the equation and the inequality in order to eliminate answer choices. If x = -3,
then y = 5. Eliminate choices (A),(B) and (E). If x = 1, then y = - 5/3. This eliminates choice (D), leaving
only choice (C). Alternatively you can solve with problem algebraically. The first equation indicates that
5x + 3y = 0. Rewrite the equation in terms of x by first subtracting 3y from both sides, so that you have
5x = -3y. Now divide both sides by 5, so that the equation is now x = -3/5y. Now you can replace x with 3/5y in the inequality x<3, so that it becomes -3/5 y <3 . Now solve for y by first multiplying both sides
by 5, so that you have -3y < 15. Next divide both sides by -3, and dont forget to flip the sign in the
inequality so that you now have > -5.

452. The crew of the Voelke Mountain Observatory records the outdoor temperature rounded to the
nearest whole degree for x days. If the temperature increases by exactly 1 degree per day, and the sum
of all of the temperatures is 0, then which of the following must be true?
I.x represents an even number of days.
II. x represents an odd number of days
III.The average temperature for the x days is 0.
(A) I only

(B) II only

(C) I and III

(D) II and III

(E) None of the above

Solution: (D)
Plug in an even number for x. Let x = 4. The question says that each day was I degree warmer than the
previous one, so you need 4 consecutive integers. If the daily temperatures were -2, -1 , 0 and 1 , the
sum is -2, not 0. Notice, however, that the three numbers, -1, 0 and 1 effectively cancel each other out,
so that the sum is 0. This suggests that odd numbers may work , whereas even ones may not. Because
the information in I is not true, eliminate choices (A) and (C). Now try an odd number for x . If x = 5 ,
then you can use -2 , -1, 0, 1 and 2 for your consecutive numbers. Each number is canceled out by its
opposite, so the sum is 0. The information in II does work, so eliminate choice (E). The average of these
numbers is 0, so III works also. The answer is choice (D).

Jenisys Systems Pvt ltd www.buy-ibpsmaster.com | www.ibpsexamguru.in

193

453. a,b,c,d,e A series of numbers in which the difference between any two successive members of the
sequence is a constant is called an arithmetic sequence. Which of the following must also be an
arithmetic sequence if the series above is an arithmetic sequence ?
I. (a-2),(b-2),(c-2),(d-2),(e-2)
II.3a,3b,3c,3d,3e
III. square root of a , square root of b, square root of c, square root of d, square root of e
(A) I only (B)II only (C)I and II only (D)II and III only (E)I,II and III only
Solution: (C)
Start by plugging in something simple for the original sequence : 2,3,4,5,6. For statement I the sequence
will be 0,1,2,3,4. This is an arithmetic sequence, so you cant cross off any answer choices yet. For
statement II the sequence will be 6,9,12,15,18. This is also an arithmetic sequence so you cant cross off
any answer choices. For statement III you get square root of 2, square root of 3, 2, square root of 5,
square root of 6. You know that square root of 2 1.7 so the difference between the first three terms in
statement III is 1,.4,.3. The sequence in statement III is not an arithmetic sequence so get rid of all the
answers that contain III. That eliminates choices (D) and (E) . Plug in again for statements I and II. Try
0,1,2,3,4 this time. The sequence for statement I is now -2,-1,0,1,2. This is an arithmetic sequence. In
fact, anytime you add or subtract a constant (in this case, subtracting 2) from an arithmetic sequence,
you will still have an arithmetic sequence, so statement I will always be true. Eliminate choice (B). Using
0, 1,2,3,4 for statement II gives you 0, 3,6,9, 12 which is also an arithmetic sequence.

454. Tom is preparing for a race, and each day he runs one mile farther than he did on the previous day.
If he runs w miles one day , x miles the next day, y miles the day after that, and on the next and final day
he runs z miles, and w,x,y and z are all integers, then which of the following must be true?
I. z - w = 3 II. wxyz is an even number III. w+x+y+z / 4 is an integer.
(A) I only

(B) II only

(C) I and II only

(D) I and III only

(E) I,II and III only

Solution: (C) .
Plug in for w,x,y and z. Let w = 3, x =4 , y =5 and z =6. That means that z - w is equal to 6 3, or 3.
Additionally, the numbers are consecutive, so the difference between z and w will always be 3, so I will
always be true. Eliminate choice (B). According to II , wxyz is an even number. 3x4x5x6 = 360, which is an
even number. There are four consecutive numbers, so you will always have two odd numbers multiplied
by two even numbers, so the result will always be even and II is also true. Eliminate choices (A) and (D).
Finally , III indicates that (w+x+y+z)/4 is an integer. (3+4+5+6)/4 = 18/4 , which is not an integer, so this is
not an integer, so this is not true. Eliminate choice (E).

Jenisys Systems Pvt ltd www.buy-ibpsmaster.com | www.ibpsexamguru.in

194

455. If 0 <1 f/g < 1, and g>0, which of the following must be true?
I.f2 +g2 >1 II.f >0 III. f/g < 1
(A) I only

(B) III only

(C) I and III only

(D) II and III only

(E) I,II and III only

Solution: (D).
This is a must be true question so you may need to Plug in more than once. If f = 2 and g=3, all three
statements work so you dont get to cross off anything. Try some weird numbers to see if you can
disprove any of the statements. When you are dealing with fractions, it is often a good idea to Plug in
fractions. Make f = and g = 2/3 . For statement I then , you get + 4/9 = 25/36, which is less than 1 .
Eliminate choices (A), (C) and (E). The remaining choices both have III in them so theres no need to
check III. For statement II, you get > 0, which is true. You cannot Plug in a negative value for f , since
g>0, so statement II will always be true.

456. Neeraj ,Manoj and Geeta start running around a circular stadium and complete one round in 10
seconds, 6 seconds and 14 seconds respectively. In how much time will they meet again at the starting
point?
1) 3 minutes 30 seconds

2) 2 minutes 28 seconds

3) 4 minutes 45 seconds

4) 1 minutes 40 seconds

5) None of these
Solution: (1) it is given Neeraj ,Manoj and Geeta finish running a round in 10 seconds 6 seconds and 14
seconds. So every 10, 6,14 seconds each one will be in the starting point respectively.
That is
10,20,30,40,.
6,12,18,24,..
14,28,42,56,.
To know the meeting time of all three together, we should take LCM
LCM of (10,6,14 ) = 210
Which is 210 seconds, since all answers are in minutes, convert seconds into minutes by dividing
210/60 = 3.5 minutes = 3 minutes and 30 seconds.
Jenisys Systems Pvt ltd www.buy-ibpsmaster.com | www.ibpsexamguru.in

195

457. Balaji, Dinesh ,Partha begin to jog around a circular stadium. They complete their revolutions in 2/3
minutes, 3/5 minutes, 4/7 minutes respectively . After how many minutes will they meet together at
starting point.
1) 4/35

2) 12

3) 1/35

4) cannot be determined

5) None of these

Solution: (2). Each finishes one round in 2/3, 3/5, 4/7 minutes. So every 2/3, 3/5, 4/7 minutes each will
be in starting point. To know the meeting time of all three together we need to calculate LCM for
fraction by following the below steps.
1. 2/3 , 3/5, 4/7 among these numbers numerators are 2,3,4 and denominators are 3,5,7
2. take LCM for numerators and HCF for denominators
3. LCM of fraction = (LCM of Numerator) / (HCF of Denominator) = LCM of ( 2,3,4) / HCF of (3,5,7) = 12/1
= 12
So in 12 minutes they will meet each other at starting point .

458. SBI, BOB and RBI gave flash advertisements such that each advertisement flashes for every 10
seconds, 20 seconds and 30 seconds respectively. If all these ads flash together at 3 pm. Then when will
all ads flash together immediately after 3 pm.?
1) 3 .01 pm

2) 3.02 pm

3) 3.03 pm

4) 3.04 pm

5) None of these

Solution: (1). To find the meeting time of all the ads together after 3 pm, simply find the LCM.
So LCM of (10,20,30) = 60
60 is 60 seconds that is one minute,. So after 3 pm advertisement of all three banks will flash together at
3.01pm.

459. Meera sets an alarm in two different clocks. The first alarm rings after every 2 hours and the
second alarm rings after every 1 hour and thirty minutes. If both the alarm rang together at 1pm what
will be the time when they will next ring together?
1) 5.30pm

2) 7pm

3) 7.30pm

4) 5pm

5) None of these

Solution: (2). To find the two clocks alarming together after 1 pm, simply find LCM of 2hours and 1
hours.

Jenisys Systems Pvt ltd www.buy-ibpsmaster.com | www.ibpsexamguru.in

196

2 hour = 120 minutes


1 hour = 90 minutes
LCM of ( 120,90 ) = 360.
So 360 minutes, which is 6 hours .
So after 6 hours from 1 pm is 7pm.

460. The salary of A and B are in the ratio 2:3 and their LCM is 48,then the sum of their salary is?
1) 28

2) 32

3) 40

4) 64

5) None of these

Solution: (3). This is an interesting sum where two concepts are tested
1.Ratio
2.LCM
It is given ratio of salary is 2:3 that is 2x and 3x is the salary
LCM of ( 2x, 3x ) = 6x
So 6x is given as 48
6x = 48 . hence X=8
Then salary of A = 2x = 2 x 8 = 16.
Similarly salary of B = 3x = 3 x 8 = 24.
So total salary is 16 + 24 = 40

461. A & B can finish a job in 10 and 15 days. In how many days they will finish, if they work together ?
1) 6

2) 8

3) 10

4) 12

5) None of these

Solution: (1). Let us take LCM (10,15) = 30


Then consider 30 chair is the work to be done.
A alone will make 3 (30/10), chairs in a day.
Similarly B alone will make 2 (30/15) chairs in a day

Jenisys Systems Pvt ltd www.buy-ibpsmaster.com | www.ibpsexamguru.in

197

Then if they work together in one day they will make 5 chairs (2+3)
If total number of chairs to be done is 30 then A&B will finish the work in 6 (30/5) days.

462. A construct a wall in 10 days B destroy the same wall in 15 days . In how many days the wall is
constructed if they work together?
1) 10 days

2) 20 days

3) 30 days

4) 40 days

5) None of these

Solution: (3). LCM (10,15) = 30


Let us consider height of wall is 30 meters
A will construct 3 metres (30/10) in a day
B will destroy 2 metres (30/15) in a day
On the whole if A & B work together only 1 metre (3 - 2) can be constructed in a day .
So to construct 30 metres it will take 30 days (30/1).

463. A & B finish a work in 30 and 20 days if they work in alternate days . In how many days work is
completed?
1) 12

2) 18

3) 24

4) Cannot be determined 5)None of these

Solution: (3). LCM (30,20) = 60


Let us consider 60 chair to be made
A will make 2 chair (60/30) in a day
B will make 3 chair (60/20) in a day
A + B + A + B + = 60 chairs
2 + 3 + 2 + 3.= 60 chairs
So in 2 days 5 chairs
Then in 24 days 60 chairs

464. A & B complete the work in 15 and 10 days , if they work together , A receives Rs.400 then how
much B receive?
Jenisys Systems Pvt ltd www.buy-ibpsmaster.com | www.ibpsexamguru.in

198

1) 800

2) 600

3) 400

4) cannot be determined

5) None of these

Solution: (2). Working days for A & B is 15 & 10 days then ratio is 3 : 2 days is inverse proportion to
efficiency then efficiency is 2 : 3
Then salary of A & B is 2x, 3x
A = 2x = 400
x = 200
then B = 3x = B x 200 = 600
So B receives Rs.600.

465. If the efficiency of A & B are in the ratio 2:3 .If A required 5 days more than the B, then in how
many days A & B finish the job individually?
1) 10,15

2) 15,10

3) 20,30

4) 30,20

5) None of these

Solution: (2) .Efficiency of A & B is 2 : 3 then day ratio of A & B is 3 : 2 efficiency 2(1/ days)
Days required for A & B is 3x, 2x difference is 5 days
3x 2x = 5
A = 3x = 15, B = 2x = 10
So 15,10.

466. a and b are real numbers. The relation = X is


1) Always true

2)Never true

3) True when a and b are positive


4) True when either a or b is non negative
5) None of these
Solution: (3). a= -ve , b= -ve
Then square root of ab = +ve
But square root of a = imaginary , square root of b = imaginary
Jenisys Systems Pvt ltd www.buy-ibpsmaster.com | www.ibpsexamguru.in

199

A = +ve , b= -ve
Then square root of ab = imaginary
Square root of a = +ve , square root of b = imaginary
a=+ve, b = +ve
the square root of ab = +ve
square root of a = +ve , square root of b = +ve
then square root of ab = square root of a x b
so a & b must be +ve

467. If A is rational and B is irrational then AB is


1) Rational

2) Irrational

3) An integer

4) Real

5) None of these

Solution: (2). Rational (A) = Natural nos , whole nos, Integers, fractions, Decimals
Eg : 1, -5 , , 0.5
Irrational (B) = square root of not perfect square
Eg : square root of 5 , square root of 10 etc.,
So A X B
2 X square roof of 5 = 2 square root of 5
= square root of 20
= irrational

468. Between any two integers, there are k integers , k is


1) Finite 2) Infinite 3) Finite under some conditions 4) Infinite under certain conditions 5) None of these
Solution: (1).Integer is -2, -3, -2, -1, 0, +1 , +2, +3 . Without decimal or fraction
Between 2 & 7
3,4,5,6 is available i.e.

Jenisys Systems Pvt ltd www.buy-ibpsmaster.com | www.ibpsexamguru.in

200

4 integers i.e. k = 4
So finite under some condition

469. If p is an integer and p2 is divisible by 3, then p is divisible by 3. This statement is


1) Always true 2) Never true 3) True when p is positive 4) True when p is negative 5) None of these
Solution: (1). Given p2 is divisible by 3
Since 3 is a prime no, p must be a multiple of 3 .
So it is always divisible by 3.

470. A number is multiplied by 9 and 9 is added to the product. If the resulting number is divisible by 17,
the smallest of such number is
1) 9

2) 16

3) 17

4) 21

5) None of these

Solution: (3). If x = 16,


Then (9x+9) = 144+9 = 153
So 153 is divisible by 17.

471. Average age of 20 students is 9 years. If the age of the teacher is included the average increases by
2. Find the age of the teacher?
1) 51 years

2) 50 years

3) 52 years

4) 53 years

5) None of these

Solution: (1).Total age of 20 students = 20 X 9 = 180 years. When the teacher is included, there are 21
people in all. Thus, after including the teachers age, average age of the 21 people = 11years.
So, total age of the 21 persons = 21 X 11 = 231.
Therefore age of the teacher = 231 180 = 51 .

472. Average age of three brothers is 10 years . If the age of the father and mother is also considered,
the average increases by 13 years. If the father is 5 years elder to the mother , what is the age (in
years)of the father?

Jenisys Systems Pvt ltd www.buy-ibpsmaster.com | www.ibpsexamguru.in

201

1) 42

2) 44

3) 45

4) 40

5) None of these

Solution: (3). Total age of the three brothers = 10 X 3 = 30. When the father and mother are also
considered in this group, the average age increases by 13 i.e. it becomes 10 +13 = 23
Therefore total age of the 5 family members = 23 X 5 = 115
So, total age of father and mother = 115 30 = 85
Let the age of the father be x years
So, age of the mother = x 5 years.
Therefore x +x -5 = 85
Therefore x = 45
Thus, the father is 45 years old

473. The average age of a couple married five years back was 24 years at the time of their marriage. The
average age of the family now is 20 years. It is known that the couple has one child. What is the age of
the child in years?
1) 2

2) 3

3) 4

4) 1

5) 5

Solution: (1). Five years ago, average age of the couple = 24. So, total age of the couple five years age =
24 x 2 = 48.
Therefore total age of the couple at present = 48 + 10 = 58
Here,the age of the child is not considered .
But, present average age of the family of three members = 20
Therefore total age of the family at present = 20 x 3 = 60
Therefore age of the child at present = 60 58 = 2 years.

474. The average runs scored by 10 players of a cricket team are 26 . If the captains runs are included,
the average increases by 4. What is the captains score?
1) 60

2) 50

3) 80

4) 40

5) 70

Solution: (5). Average score of 10 players = 26

Jenisys Systems Pvt ltd www.buy-ibpsmaster.com | www.ibpsexamguru.in

202

Total score of the 10 players without the captain = 26 x 10 = 260


When the captains score is added, the average increases by 4 i.e. it becomes 30
So, total score of the 11 players including the captain = 30 x 11 = 330
Therefore runs scored by the captain = 330 260 = 70

475. If the average of three different positive integers is 70, what is the greatest possible value of one of
the integers?
1) 207

2) 208

3) 209

4) 210

5) Cannot be determined

Solution: (1). Average of three distinct positive integers = 70


Therefore sum of three integers = 3 x 70 = 210
If the largest of the three integers is 210, the other two integers have to be 0 or negative.
However, it is given that both the other integers are positive and distinct.
So, those two integers have to be the smallest possible integers i.e. 1 and 2
So, the largest number required = 210 ( 1+2) = 207

Ratio & Proportion


476. In a class of 90 students, boys and girls are in the ratio 5:4. Now 2000 books were distributed
among them. If each girl get five books more than a boy, how many books will each girl get?
1) 25

2) 30

3) 35

4) 45

5) None of these

Solution: (1). It is given that ,boys and girls are in the ratio 5:4.
So, number of boys and girls are 5x and 4x
It is given total students are 90 .
Then 5x + 4x = 90
x = 10
Then boys = 5x = 5 (10) = 50
Girls = 4x = 40
Jenisys Systems Pvt ltd www.buy-ibpsmaster.com | www.ibpsexamguru.in

203

It is given, each girl gets five books more than a boy


So the total number of extra books given to all the girls are,
5 * 40 = 200
So the remaining books are equally distributed to boys and girls (2000 - 200) = 1800
Now this 1800 books are distributed to 90 students.
So each student is getting 20 books
So the girls will get = 20 + 5 = 25 books

477. A bag contains 2 rupee, 1 rupee, 50 paise and 25 paise coins in the ratio 1:3:4:8 respectively. The
total amount in the bag is Rs.36. Find the number of coins in the bag?
1) 84

2) 64

3) 74

4) 94

5) None of these

Solution: (2)
Given - 2 rupee, 1 rupee, 50 paise and 25 paise coins are in the ratio 1:3:4:8
To get uniformity convert all denominations into paise.
i.e., 2 rupee = 200 paise
1 rupee = 100 paise
50 paise
and 25 paise
Since these coins are in the ratio 1:3:4:8 respectively,
And the sum of the value these coins = 3600 paise (36 Rs.)
Now equating according to the ratio we can write it as,
1x*200 + 3x*100 + 4x*50 + 8x*25 = 3600
900 x = 3600
x=4
Total ratio = 1:3:4:8
= 1x + 3x + 4x + 8x = 16x
Jenisys Systems Pvt ltd www.buy-ibpsmaster.com | www.ibpsexamguru.in

204

= 16 (4) = 64 coins

478. A tank contains 75 liters of milk and water in the ratio 4:1. Find the quantity of water to be added
to the tank to make the ratio as 3:1?
1) 20

2) 30

4) 40

4) 50

5) None of these

Solution: (5)
Milk : Water = 4x : 1x
So, 4x + 1x = 75
X = 15
Since Milk = 4x and x = 15
Milk = 60 liters
Water = 1x
= 1 x 15 = 15 liters
New ratio of Milk : Water should be 3:1
So in the new ratio quantity of milk will remain same i.e. 3x = 60 liter , x = 20 liter
According to the new ratio of 3:1 water is 1 part that is 20 liter , in the previous mixture water was 15
liters so to convert 15 to 20 the required amount of water to be added is 5 liters

479. A watermelon contains 80% of water whereas a dry Grape contains 20% of water. Now how many
kg of dry grapes are required to equalize the water content of 20kg of watermelon?
1) 36kg

2) 46kg

3) 16kg

4) 26kg

5) None of these

Solution: ( 5). Let us solve this sum by equating :


x % of y = y % of x
80 % water * weight of watermelon = 20% water * x kg of grapes
So x =80 kg

Jenisys Systems Pvt ltd www.buy-ibpsmaster.com | www.ibpsexamguru.in

205

480. In a town of 50000 people, 42% of male and 28% of female (who belong to the same town) were
married. Find the number of males in the town.
1) 64

2) 54

3) 44

4) 84

5) None of these

Solution: (5). Let us solve this sum in a tricky approach


The fact is if 42 % of male is married to 28% of female, indirectly it says that number of male and
number of female are equal
That is 42 % of male = 28% female
42 / 100 * total number of male = 28/100 * total number of females
By solving we get (male / female) = 28/42
i.e. 2/3 .
so male and female are in the ratio of 2:3 = 2x :3x
2x+3x = 50000
So x=10000
So number of male = 20000 and number of female = 30000
So as per question number of males in the town = 20000
(So answer is none of these)

Jenisys Systems Pvt ltd www.buy-ibpsmaster.com | www.ibpsexamguru.in

206

Вам также может понравиться